Hepatobiliary Flashcards

1
Q

A 66-year-old Japanese female is referred by her
primary physician for long-standing biliary colic symptoms. She describes 10 to 15 years of intermittent right upper quadrant (RUQ) pain with nausea that typically resolves after 1 to 2 hours.

She went to the emergency room once 6 years ago and had an ultrasound that showed gallstones. Her medical history is significant for hypertension and osteoporosis. Her vitals and exam are unremarkable.

A repeat RUQ ultrasound ordered by her primary care physician now shows a large 3 cm gallstone, As well as a fixed mass in the fundus, 2 cm in diameter, that appears to originate from the gallbladder wall. The immediate surrounding gallbladder wall is thickened to 8 to 11 mm.

A CBC, basic chemistry, and liver function tests are all within normal limits.

Which of the following is a risk factor for gallbladder carcinoma?

A. Hemolytic anemia

B. Biliary dyskinesia

C. Clonorchis sinensis infection

D. Anomalous union of the pancreaticobiliary ductal
system or pancreaticobiliary maljunction (PBM)

E. Auto-immune diseases

A

D.

A history of gallstones is common, and 65% to
90% of those with biliary carcinoma, have a history
of gallstones. The relation of gallstones to gallbladder
cancer (GBC) is thought to be mediated by chronic
inflammation.

There is a relatively well-defined sequence of flat-epithelial premalignant changes leading to GBC. Chronic inflammation leads to intermediate low-grade dysplastic changes. Dysplastic progression over time leads to carcinoma-in-situ and finally invasive carcinoma.

Anomalous union of the pancreaticobiliary ductal system, where the pancreatic duct and common bile duct merge outside the wall of the duodenum and form a long common channel, is also associated with an increased risk of GBC. This pancreaticobiliary maljunction (PBM)
leads to chronic reflux of pancreatic enzymes.

The progression to GBC is likely mediated through
an epithelial hyperplasia with resultant papillary or
villous epithelial changes progressing to GBC.

Further evidence that this is a distinct pathway from
chronic inflammation is that the gene alterations of
cancers arising in the setting of cholelithiasis differ
from anomalies of the duct system associated cancers.

Adenomas do occur in the gallbladder and can
progress to cancer. However, this likely occurs much
less commonly than the other two pathways, given
a lack of cancer-related molecular changes in most
of these lesions.

Inflammatory bowel disease is also associated with increased risk for gallbladder carcinoma. Clonorchis infection is associated with risk for cholangiocarcinoma, but it has not been linked to carcinoma of the gallbladder.

Chronic Salmonella typhi or paratyphi infection is, however, associated with increased risk for gallbladder cancer.

Hemolytic anemia may be a cause of bilirubin type gallstones but is not a risk factor for gallbladder cancer, neither is biliary dyskinesia or auto-immune diseases.

How well did you know this?
1
Not at all
2
3
4
5
Perfectly
2
Q

Which of the following radiographic findings is associated with the highest incidence of gallbladder carcinoma?

A. Pancreaticobiliary maljunction (PBM) without biliary dilatation identified on magnetic resonance cholangiopancreatography (MRCP).

B. Strongly enhancing thick inner layer and a weakly enhancing or non-enhancing outer layer of the gallbladder wall on the portal phase of a multi-detector CT scan (MDCT).

C. Gallbladder polyp 10-20mm on ultrasound.

D. Gallbladder wall calcifications; “porcelain gallbladder”

E. Asymptomatic gallstone greater than 3 cm in size.

A

B.

A recent retrospective study of findings on MDCT associated with gallbladder cancer found two patterns most associated with finding malignancy at the time of surgery.

A strongly enhancing thick inner layer and a weakly enhancing or non-enhancing outer layer of the gallbladder wall on multi-detector CT scan was shown to have a 52% to 55% incidence for gallbladder cancer.

A single thick layer with heterogenous enhancement on MDCT had an incidence of 35% to 38% for gallbladder cancer.

Pancreaticobiliary malformation (PBM) without bile duct dilatation have an incidence of biliary tract cancer of 37.9%, of which 93.2% of these were gallbladder cancer.

Based on this data, prophylactic cholecystectomy is recommended for these patients.

The incidence rates of malignancy in gallbladder polyps varies widely in published reports but ranges from 9.6% to 40% for polyps 10 to 20 mm.

The wide variance is related to various imaging modalities used and populations studied.

Polyps >10mm, sessile polyps and rapidly growing polyps are all recommendations for gallbladder removal.

The finding of gallbladder calcifications or a porcelain gallbladder was found to be associated with a malignancy in 6% of cases in a recent systematic literature review. This is a far lower number than the historically quoted figure of approximately 25%.

Attempts in the review were made to limit inherent biases in a review of retrospective studies that favor overestimation but this value likely still overestimates the true incidence.

Given the incidence, the decision to perform a prophylactic cholecystectomy should not be absolute and should be weighed against the risks of surgery for the individual patient. The presence of gallstones is associated with an increased risk of gallbladder cancer.

The size and volume of stone burden have been identified as potential risk factors for developing gallbladder cancer.

However, there is no direct evidence of a causal relationship between gallstones and gallbladder cancer. The risk for gallbladder cancer development with a 3 cm or greater stone has been estimated to be a 2% risk over a 20 year period.

How well did you know this?
1
Not at all
2
3
4
5
Perfectly
3
Q

Review of all imaging studies shows a gallbladder tumor invades into the muscularis propria. There is no lymphadenopathy or distant metastases seen. What is the most appropriate next step in management?

A. Referral for definitive treatment with chemotherapy and radiation.

B. Referral for neoadjuvant chemotherapy and radiation.

C. Schedule for cholecystectomy.

D. Schedule for cholecystectomy with removal of
regional lymph nodes and en-bloc hepatic resection.

A

D.

The patient has a T1b lesion or early gallbladder
cancer.

Surgical resection is the only curative therapy
for gallbladder cancer.

The general consensus is for radical cholecystectomy for T1b or greater lesions. T1b tumors have been shown to have lymph node metastasis in 24% of cases. Lower recurrence rates and improved survival have been observed with radical resection including lymph nodes when compared to simple cholecystectomy.

A review of the SEER database showed that the evaluation of even a single lymph node improved overall survival and that radical resection without lymph node assessment was no better than cholecystectomy alone for early stage gallbladder cancer.

Neoadjuvant therapy has been evaluated in the setting of borderline and unresectable extra-hepatic biliary malignancies with good results in survival and obtaining negative margins but none of these addressed gallbladder cancer specifically and were confined to advanced disease.

Adjuvant chemoradiation does have a role in select
gallbladder cancer patients, especially with positive nodes or margins to improve local control.

How well did you know this?
1
Not at all
2
3
4
5
Perfectly
4
Q

The same patient with the same history of present
illness and past medical history instead presents
to the emergency room with RUQ pain.

The ultrasound in this case is read as 2 large gallstones
with diffuse gallbladder wall thickening up to 11 mm, pericholecystic fluid, and a normal common bile duct. Labs show white blood cell count of 13 and normal liver function. You take the patient for laparoscopic cholecystectomy.

There was no concern for malignancy during the
procedure. The gallbladder was removed without
spillage and with a retrieval bag. You see her 2 weeks
later in clinic, and review of the pathology report
shows T2 adenocarcinoma.

All surgical margins, including cystic duct margin, are reported as clear. What is the most appropriate course of action?

A. No additional surgery, surveillance with imaging
every 6 months.

B. Staging with imaging followed by radical cholecystectomy to include: liver resection with at
least 3 cm of margin around gallbladder bed and regional lymphadenectomy.

C. Staging with imaging followed by radical cholecystectomy and in addition, excision of the
previous laparoscopic port sites.

D. Staging with imaging followed by radical cholecystectomy and in addition, excision of the
common bile duct.

E. Staging with imaging followed by radical cholecystectomy and in addition, excision of both the
common bile duct and the laparoscopic port sites.

A

B.

Prognosis of gallbladder carcinoma is determined
by the depth of tumor infiltration and the ability to
obtain a tumor-free resection margin (R0).

For T2 or greater the definitive resection should include a minimal hepatic resection centered on the gallbladder
bed and a regional lymphadenectomy.

The majority of experts also extend this recommendation to T1b tumors but some controversy persists. There is uniform agreement that cholecystectomy alone is sufficient for Tis and T1a tumors. There is also general
agreement regarding several other technical points, such as resection of the common bile duct, and the need for port site excision. Excision of the common bile duct is only necessary for a positive cystic duct margin or direct invasion of the hepatoduodenal ligament.

Regarding routine common bile duct excision with radical cholecystectomy unless there is direct invasion of the hepatoduodenal ligament and/or of the cystic duct, bile duct resection does not result in decreased recurrence or better overall survival and does not increase the number of nodes in the specimen.

Peritoneal involvement with gallbladder cancer is common and there is theoretical adverse impact on this with pneumoperitoneum. However, the risk of port site recurrence is based on perforation of the gallbladder or extraction without a retrieval bag rather than the pneumoperitoneum.

Port-site excision does not improve overall or disease-free survival in large retrospective series. Port site excision does not need be routinely performed during secondary procedures for gallbladder cancer discovered
after laparoscopic cholecystectomy.

How well did you know this?
1
Not at all
2
3
4
5
Perfectly
5
Q

Which of the following is true regarding the surgical management for T2 or T3 gallbladder cancers?

A. Formal segmentectomy (4b + 5) improves overall survival over wedge resection.

B. Formal segmentectomy reduces local recurrence
rates compared to wedge resection, but overall survival is the same.

C. Formal segementetomy improves disease free survival, but not overall survival.

D. A clear survival benefit for formal segmentectomy
over wedge resection has not been demonstrated.

A

D.

Although some studies have reported anatomical resection improves survival and RO resection rate when compared to wedge resection, other reports have not demonstrated a benefit.

The majority of these studies dealt with liver resection for liver metastases. There are a few studies looking at gallbladder cancer specifically.

Pawlik et al. found that patients who underwent a major hepatic resection (e.g., formal segmentectomy of 4b + 5 or hemi-hepatectomy) had a similar risk of disease-specific death compared with patients who underwent a hepatic wedge resection. Horiguchi et al. found the overall survival rate and disease-free survival rate at 5 years did not differ significantly between wedge resection and 4a + 5 resection group for T2 tumors.

The available evidence in gallbladder cancer does not show a clear benefit to anatomic resection.

As such, the surgeon’s goal should be to resect all disease with negative histologic margins and chose the appropriate operation to achieve this with the fewest complications.

How well did you know this?
1
Not at all
2
3
4
5
Perfectly
6
Q

A 67-year-old female presents with sharp, burning right upper quadrant abdominal pain that radiates to her back. The pain awakens her from sleep. She has experienced nausea and three episodes of emesis. She reports burning substernal and epigastric pain. She denies any fevers or chills. She is hemodynamically normal and afebrile. Laboratory workup reveals a normal hepatic function panel, a normal basic chemistry, and a normal white blood cell count. Her abdominal exam is remarkable for mild tenderness in the right upper quadrant without peritoneal signs. After three hours, her pain resolves.

  1. What is the next best test in this patient scenario?

A. Plain films of the abdomen

B. Cholecystokinin stimulated cholescintigraphy

C. Right upper quadrant ultrasound

D. Computed tomography (CT) of the abdomen

E. Esophagogastroduodenoscopy (EGD)

  1. The appropriate imaging is ordered and shows no abnormality. Which is the next most appropriate
    step in management?

A. Cholecystectomy

B. Esophagogastroduodenoscopy

C. Right upper quadrant ultrasound in two weeks

D. Proton pump inhibitor administration

E. Bile microscopy

A

1) C. This scenario describes symptomatic cholelithiasis, or biliary colic.

Plain films of the abdomen would not be a high yield study as less than 10% of gallstones are radio-opaque, nor would it offer specific imaging of the biliary system. Other yields from a plain film would be presence of nephrolithiasis or free air (Choice A).

A hepatobiliary (HIDA) scan is the gold standard for the diagnosis of acute cholecystitis—a clinical scenario in which she does not fit criteria (Choice B). With the cholecystokinin injection, the nuclear imaging study can be diagnostic for biliary dyskinesia.

CT can often visualize gallstones but is not the recommended imaging modality of choice for cholelithiasis as the sensitivity is 55% to 80% (Choice D); cholelithiasis is often found incidentally on CT scan when the imaging is performed for other reasons.

Esophagogastroduodenoscopy could be helpful to rule out peptic ulcer disease that can sometimes mimic symptoms of biliary colic, but this test would not be the ideal next choice in this situation (Choice E).

The most common initial imaging modality in biliary disease is the right upper quadrant ultrasound. Ultrasound has a sensitivity of 84% and specificity approaching 99% for the diagnosis of cholelithiasis (Choice C). Simultaneously, assessment for signs of acute cholecystitis, choledocholithiasis, and hepatic pathology can be sought.

2) C.

This patient should have had a right upper quadrant ultrasound performed in the first question above. If this imaging modality shows no abnormality, but the patient has biliary symptoms, she should have a repeat right upper quadrant ultrasound performed in two weeks with focus on areas that may have missed stones smaller than
3 mm (Choice C).

Esophagogastroduodenoscopy is an appropriate diagnostic modality to rule out peptic ulcer disease and can be utilized if the repeat ultrasound is still negative. Bile microscopy has been shown as an adjunct study to assess for microlithiasis and can be helpful (Choice E), although is not routinely performed; bile microscopy also requires endoscopy for collection. The sensitivity of bile microscopy for microlithiasis is 65% to 90%.

Cholecystectomy is not recommended at this time as other causes of the patient’s symptoms have not been ruled out. Cholecystokinin stimulated cholescintigraphy can help identify a functional gallbladder problem that
may warrant cholecystectomy if positive (Choice A).

Proton pump inhibitor therapy may help with reflux
symptoms but does not specifically address biliary
symptoms (Choice D).

How well did you know this?
1
Not at all
2
3
4
5
Perfectly
7
Q

A 67-year-old female presents with sharp, burning right upper quadrant abdominal pain that radiates to her back. The pain awakens her from sleep. She has experienced nausea and three episodes of emesis. She reports burning substernal and epigastric pain. She denies any
fevers or chills. She is hemodynamically normal and afebrile. Laboratory workup reveals a normal hepatic function panel, a normal basic chemistry, and a normal white blood cell count. Her abdominal exam is remarkable for mild tenderness in the right upper quadrant without peritoneal signs. After three hours, her pain resolves.

The patient returns to the emergency department one month later with similar symptoms. A right upper quadrant ultrasound is performed that shows gallstones without signs of acute cholecystitis. Her pain again resolves and she is referred to your clinic the following week. You discuss performing a cholecystectomy with the patient, which she refuses. Which of the following
is true regarding medical therapy of cholelithiasis?

A. Ursodiol therapy typically resolves biliary symptoms within one month.

B. Complete resolution of cholelithiasis is successful in less than 40% of patients.

C. If biliary symptoms persist while on ursodiol, cholecystectomy is indicated.

D. The mechanism of action of ursodiol includes reduction in bile synthesis.

E. Extracorporeal shockwave lithotripsy (ESWL)
has a symptomatic recurrence rate of 30%.

A

B.

Ursodiol may take up to 3 months to show
improvements in biliary symptoms, and can takeup to
3 years to completely dissolve gallstones (Choice A).

A meta-analysis of treatment with ursodiol showed
that only 37% of patients had complete resolution
of biliary symptoms; cholecystectomy remains the
preferred choice in surgical candidates (Choice B).

Many patients who undergo initial medical therapy
with ursodiol do progress to surgical treatment, but
they tend to show initial improvement in symptoms.

If biliary symptoms progress while being treated
with ursodiol, other causes must be ruled out, such
as sphincter of Oddi dysfunction or peptic ulcer
disease (Choice C).

The mechanism of action of ursodiol involves reducing absorption in the duodenum, resulting in disruption of micelles and reduced cholesterol absorption, thereby decreasing cholesterol concentration; ursodiol is not involved with the synthesis of bile (Choice D).

Extracorporeal shockwave lithotripsy can be utilized as nonsurgical therapy, but has a recurrence rate of approximately 20%.

Indications include patients with single stones, between the sizes of 5 mm and 2 cm.

ESWL is not commonly offered as medical therapy due to the efficacy and commonplace practice of cholecystectomy (Choice E).

How well did you know this?
1
Not at all
2
3
4
5
Perfectly
8
Q

A 67-year-old female presents with sharp, burning right upper quadrant abdominal pain that radiates to her back. The pain awakens her from sleep. She has experienced nausea and three episodes of emesis. She reports burning substernal and epigastric pain. She denies any fevers or chills. She is hemodynamically normal and afebrile. Laboratory workup reveals a normal hepatic function panel, a normal basic chemistry, and a normal white blood cell count. Her abdominal exam is remarkable for mild tenderness in the right upper quadrant without peritoneal signs. After three hours, her pain resolves.

The patient’s ultrasound, in addition to cholelithiasis, is also notable for a 1.5 cm non-mobile polypoid lesion in the fundus of the gallbladder. Which of the following is true?

A. Polyps greater than 5 mm are a risk factor for
gallbladder cancer.

B. Patients with large (> 2.5 cm) gallstones are more likely to develop gallbladder cancer than those without.

C. Laparoscopic cholecystectomy should be performed, a frozen analysis should be done, and if
positive, an oncologic resection should be done
laparoscopically.

D. Multiple pedunculated subcentimeter lesions are also a risk factor for gallbladder adenocarcinoma.

E. Extended cholecystectomy is not required for gallbladder adenocarcinomas with up to T3 lesions.

A

B.

This patient should undergo open cholecystectomy with a symptomatic, visualized preoperative intraluminal gallbladder lesion that is 1.5 cm in size.

The open cholecystectomy is preferred, because
gallbladder perforation or bile spillage during laparoscopic cholecystectomy can potentially seed the peritoneal cavity.

Polypoid lesion size greater than 1 cm is an independent risk factor for gallbladder adenocarcinoma, along with stone size greater than 2.5 cm (Choice A, B).

Gallbladder cancer has a 3:1 ratio of incidence in women and typically presents after age 60.

Extended cholecystectomy is not indicated if the lesion is confined below the muscle layer of the gallbladder.

If the lesion is T2 or greater, resection usually includes segments IVb and V of the liver (Choice E).

Choice D is descriptive of cholesterol polyps, which are not a risk factor for gallbladder adenocarcinoma.

How well did you know this?
1
Not at all
2
3
4
5
Perfectly
9
Q

A 70-year-old male presents to the emergency department with altered mental status. Family reports he was complaining of right upper quadrant abdominal pain prior to becoming altered mentally. On arrival, he is found to have a temperature of 102.5°F, heart rate of 112, and systolic blood pressure of 80 despite 2 liters of crystalloid infusion. On exam, he is visibly jaundiced with tenderness in the right upper quadrant. He has 3 out of 4 systemic inflammatory response syndrome
(SIRS) criteria. He is started on pipericillin/tazobactam and admitted to the intensive care unit for invasive monitoring and vasopressor support.

  1. Regarding this patients constellation of symptoms, what is the most common cause?

A. Gallstones

B. Biliary stricture

C. Malignancy

D. Genetic disorder

A

A.

Cholangitis is caused by obstruction of the biliary tree eventually leading to bile stasis and bacterial infection. The most common cause being gallstones, which account for around half of cases.

Other causes include stenosis/biliary stricture, malignancy and biliary stents.

Stents can cause obstruction from migration, occlusion, or colonization by bacteria leading to bacterial overgrowth and translocation into the bloodstream.

How well did you know this?
1
Not at all
2
3
4
5
Perfectly
10
Q

Regarding the pathophysiology of cholangitis, which of the following is correct?

A. Increased biliary pressure leads to decrease in production of IgG in the biliary mucosa leading to increased translocation of duodenal bacteria.

B. Intra-portal toxins and bacteria can cross through the biliary system due to stasis leading to infection.

C. Stones do not colonize with bacteria.

D. Biliary stents are not felt to contribute to or cause
cholangitis as they help decompress the biliary tree.

A

B.

Obstruction of the biliary tree via stricture/stenosis, stones, malignancy, or stent occlusion leads to increased biliary tract pressure. This pressure promotes stasis of bile and decreases production of IgA in the bile tract mucosa.

The lack of continuous bile flow, coupled with decrease mucosal protection allows for bacterial translocation from the duodenum through the biliary tract. This static bile and gallstones provide a healthy growth medium for bacteria.

The elevated intra-biliary pressure allows for translocation of these pathogens into the systemic
circulation causing septicemia. Less commonly, bacteria and toxins can enter through the portal circulation into the bile due to increase biliary pressure.

The most common bacteria are gram negative enteric pathogens: E coli, klebsiella, and enterobacter. They carry LPS that promotes cytokine release and leads to septic shock. Enterococcus is seen in a smaller set of cases.

How well did you know this?
1
Not at all
2
3
4
5
Perfectly
11
Q

Regarding the proper diagnostic workup, which noninvasive test has highest sensitivity and specificity for detecting the most common cause?

A. Abdominal ultrasound

B. Magnetic resonance cholangiopancreatography (MRCP)

C. CT Scan

D. Hepatobiliary (HIDA) scan

A

B.

MRCP is the best non-invasive test to confirm the presence of choledocholithiasis due to its high sensitivity (some studies quoting 100%) and nearly 100% specificity.

If the test is positive then this confirms diagnosis. It also is helpful in evaluating for stricture and ampullary masses.

Ultrasound is a good screening tool and can evaluate for common bile duct dilatation to perhaps lend clinical suspicion to presence of biliary obstruction. It is best at identifying the presence of cholelithiasis/cholecystitis. However, it has lower accuracy in identifying the presence of a choledocholith, roughly 80%.

A CT scan is less useful than ultrasound in detecting cholecystitis/common bile duct dilatation, but is helpful at evaluating for ampullary masses as a cause of a dilated common bile duct.

HIDA scans are not useful in the setting of cholangitis as the biliary tract infection reduces secretion of the radio nucleotide labeled marker into the biliary tree. It may show, however, obstruction with lack of flow into the duodenum.

How well did you know this?
1
Not at all
2
3
4
5
Perfectly
12
Q

Regarding the management of septic cholangitis caused by choledocholithiasis, which of the following is correct?

A.	Urgent biliary tract decompression via endoscopic
retrograde cholangiopancreatography (ERCP) is successful 60% of the time.

B. Percutaneous transhepatic cholangiography (PTC) is feasible for stone extraction and stent placement.

C. ERCP with a sphincterotomy is equivalent to cholecystectomy for reducing recurrence rates.

D. Should ERCP and PTC fail or are not feasible, operative choledochotomy and T-tube placement should be avoided because of the risk of surgery.

E. Broad spectrum antibiotic therapy alone will generally provide adequate treatment.

A

B.

In cases of cholangitis without septic shock, a trial of antibiotic therapy is recommended as this can resolve symptoms and ensure stability in up to 80% of patients.

Routine ERCP can be performed in this setting assuming the patient remains stable.

This patient displays Reynold’s pentad of fever, right upper quadrant pain jaundice, altered mental status, and hypotension, the first three signs constituting Charcot’s triad. This lends suspicion to suppurative cholangitis due to the patient’s state of septic shock.

Antibiotics, though required as initial therapy, are unlikely to complete resolve this patient’s septic physiology. Emergent/urgent biliary tree decompression is warranted and must be performed to prevent excessive morbidity/mortality.

ERCP with sphincterotomy has shown upwards of a 95% success rate in stone extraction and decreasing the rate of recurrence of cholangitis. It is, however, not superior to cholecystectomy in decreasing rates of recurrence and thus cholecystectomy is recommended after the ERCP/sphinceterotomy once the septic physiology has resolved. Arguments for early cholecystectomy have been made as waiting 6 to 8 weeks runs the risk of 20% recurrence rate of a gallstone related event.

Should ERCP fail, PTC is warranted as both these
procedures decrease the morbidity/mortality risk of a common bile duct exploration. PTC can be challenging if there is little intra-hepatic ductal dilatation and it also does not allow sphincterotomy.

Common bile duct exploration is warranted should ERCP and PTC fail at decompressing the biliary tree. In the setting of a patient who is in septic shock, choledochotomy with stone extraction and T-tube placement is recommended as this allows for decompression of the biliary tree and allows for sepsis to resolve prior to performing cholecystectomy to limit the morbidity and mortality associated with both procedures.

The exploration is performed through a choledochotomy on the common bile duct distal to the insertion of the cystic duct. Stay sutures are placed on either side of the choledochotomy and using balloon catheters, fluoroscopy with basket retractors, and flushing, stone extraction is performed. In general, the choledochotomy should be roughly the size of the largest stone. It is best done in dilated ducts as the risk of stenosis is high in
the setting of common bile duct size <6 mm.

A large bore T-tube is placed and the choledochotomy repaired over the T-tube with 4-0 absorbable sutures.

The tube is externalized and bile allowed to drain into an external bag. Due to lack of re-absorption of bile, a patient with this procedure is prone to being deficient in fat soluble vitamins (A, D, E, and K). It is the vitamin K deficiency which is most worrisome as it can lead to a coagulopathy.

How well did you know this?
1
Not at all
2
3
4
5
Perfectly
13
Q

A 32-year-old gravida 2 para 1 female at 28 weeks gestation presents with acute onset of right upper quadrant and right upper flank pain with associated nausea and vomiting over the preceding 24 hours. She has no significant medical or surgical history. She has had 1 uncomplicated vaginal delivery. At the time of her evaluation, her temperature is 99.8°F, heart rate is 110, and respiratory rate is 24. Her exam documents a positive Murphy’s sign and guarding in the right upper quadrant. Laboratory studies show the following: WBC - 20,000, H/H- 9/29, Platelets 130,000, ALT-60, and AST 90. Her bilirubin, lipase, and amylase levels are normal. Her urinalysis is within normal limits.

  1. Which of the following diagnoses is the least common disease in pregnancy presenting with right upper quadrant pain?

A. Acute fatty liver of pregnancy

B. Cholecystitis

C. Cholelithiasis

D. HELLP syndrome

E. Appendicitis

A

A.

The most common surgical disease in pregnancy is appendicitis with an incidence of 1 in 1000 to 2000 pregnancies.

Gallbladder disease is the second most common surgical disease in pregnancy with an incidence of 1 in 1200 to 1 in 10,000. Theoretically, the incidence of gall bladder disease including cholelithiasis, cholecystitis, and cholangitis should be increased in pregnancy.

The elevated level of serum estrogen seen in pregnancy increases cholesterol secretion, whereas the elevated level of progesterone reduces soluble bile acid secretion and slows emptying of gallbladder. Despite the predilection toward biliary sludge and stone formation, cholecystitis does not occur more frequently during pregnancy. Appendicitis occurs with equal frequency in each trimester and the incidence is not increased in the gravid patient.

The differential diagnosis for RUQ abdominal pain is expanded in pregnancy. It includes gastrointestinal disorders such as pancreatitis, peptic ulcer disease, hepatitis, and appendicitis, due to a superiorly displaced cecum, as well as pyelonephritis, nephrolithiasis, right lower lobe pneumonia, peptic ulcer disease, and myocardial infarction.

Obstetric specific diagnoses must also be included in the differential to include preeclampsia, HELLP (Hemolysis, Elevated Liver enzymes. Low Platelets syndrome and acute fatty liver of pregnancy (AFLP).

HELLP syndrome is a severe form of preeclampsia occurring in up to 8 of 1000 pregnancies presenting most commonly in the third trimester of pregnancy. This syndrome generally involves the characteristic hypertension and proteinuria seen with preeclampsia with evidence of liver dysfunction and a consumptive coagulopathy which can rapidly progress to fulminant
DIC.

Patients with preeclampsia may present with right upper quadrant or epigastric pain due to liver involvement and in the most severe cases subcapsular hemorrhage or hepatic rupture. AFLP is a rare diagnosis, seen in 1:20,000 pregnancies.

This patient presents with findings consistent with an inflammatory intra-abdominal process. Cholecystitis, choledocholithiasis, and cholangitis lead the differential diagnosis. The physical exam findings are highly suggestive of gallbladder disease. WBC counts and alkaline phosphatase levels are routinely elevated during pregnancy and therefore may not be as specific for inflammation during the assessment of the gravid patient.

How well did you know this?
1
Not at all
2
3
4
5
Perfectly
14
Q

Which of the following statements is correct concerning Acute Fatty Liver of Pregnancy (AFLP)?

A. AFLP presents most commonly in the second trimester of pregnancy.

B. AFLP commonly presents with serum aminotransferase levels similar to those found in gallbladder disease.

C. AFLP can present with hypoglycemia and occasionally renal failure which can help distinguish
it from HELLP (hemoconcentration, elevated liver enzymes, low platelet) syndrome and gallbladder disease.

D. In a preterm pregnancy, it is considered safe to
continue the pregnancy in a patient who has been diagnosed with AFLP.

A

C.

The AFLP syndrome almost always presents in the third trimester with serum aminotransferase elevations up to 1000IU/L, which is generally higher than those found in gallbladder disease.

AFLP can also present with hypoglycemia and renal failure, which is not characteristic of either HELLP or gallbladder disease.

Findings of AFLP can still significantly overlap with those of HELLP, making it very difficult to distinguish these two syndromes.

The treatment for both is the emergent delivery of the fetus.

How well did you know this?
1
Not at all
2
3
4
5
Perfectly
15
Q

Which of the following statements regarding radiographic imaging of biliary disease in pregnancy is
correct?

A. Classic sonographic signs of biliary disease are
altered in pregnancy.

B. The risk of radiation exposure to the fetus with ERCP (endoscopic retrograde cholangiopancreatography) is high throughout pregnancy.

C. The neuronal development of the fetus is most
sensitive to radiation between 20 to 28 weeks gestation.

D. Exposure to less than 5 rad of ionizing radiation has not been associated with an increased risk of fetal anomalies or pregnancy loss.

E. MR imaging has a higher sensitivity and specificity in the diagnosis of cholecystitis than ultrasonography.

A

D.

Due to the acuity of presentation and the myriad of diagnoses in the differential, imaging is an essential component in the diagnostic evaluation.

Risks of radiologic studies to the fetus must therefore be considered.

Sonography is the appropriate first line diagnostic modality in pregnancy for both biliary disease and appendicitis as this modality has a high diagnostic accuracy (90% to 100% for both diagnoses) and has no known risk to the fetus.

Classic ultrasound (US) findings to include wall edema, pericholecystic fluid, calculi, and sonographic Murphy’s sign maintain their sensitivity and specificity in pregnancy.

If ultrasound studies arc non-diagnostic, MR imaging without contrast has become the confirmatory test for appendicitis.

For biliary disease, an MRCP can be used in equivocal eases or in suspected eases of choledocholithiasis or cholangitis. It is not as sensitive as US for cholecystitis.

Intraoperative cholangiogram in combination with cholecystectomy is an option for diagnostic evaluation after fetal organogenesis is complete in the second trimester and does not appear to increase the risk for preterm delivery or adverse fetal outcomes.

If MRCP documents stone disease in the biliary tree, ERCP is considered a viable therapeutic option after the first trimester. The risks to the fetus with cholangiogram and ERCP can be reduced with shielding.

CT scan of the abdomen, which is the preferred imaging modality for appendicitis in the non-pregnant patient, confers radiation levels of 5 to 10 rads which approach the maximum permissible radiation dose for fetal exposure during pregnancy.

Fetal exposure to ionizing radiation increases risks of microcephaly, micropthalmia, mental retardation, growth restriction, and cataracts.

The concern of ionizing radiation is greatest during organogenesis which falls between 3-20 weeks of gestation.

The above patient is at 28 weeks and therefore the
risk of serious complications with ionizing radiation is limited. CT generally remains behind US and MR on the imaging algorithm for both appendicitis and cholecystitis even in the patient with a fetus of advanced gestational age due to the disputed twofold
increased risk of carcinogenesis ( 1:1000) in the fetus.

CT imaging should, however, not be abandoned as a diagnostic modality, as the risk of delay in diagnosis far outweighs the risk of radiation.

The consulting radiologist can design CT protocols to minimize the associated risks and counseling can minimize the associated anxiety of the patient.

How well did you know this?
1
Not at all
2
3
4
5
Perfectly
16
Q

In this patient, acute cholecystitis is diagnosed by ultrasound. Which of the following is correct regarding treatment of this patient?

A. The risk of adverse effects of laparoscopy is high
even with maximal intra-abdominal pressures limited to 9 mm Hg.

B. If left untreated, the most common complication
of acute cholecystitis in pregnancy is gangrenous
cholecystitis.

C. Available studies have shown significant differences regarding preterm delivery rates, birthweights or neonatal outcomes when comparing laparoscopic versus open cholecystectomies.

D. Nonsteroidal anti-inflammatory drugs (NSAID)
treatment for pain expected to last more than
48 to 72 hours is the pharmacologic option of
choice after 30 weeks gestation to avoid fetal
complications.

E. Beta-lactam antibiotics such as ampicillin-sulbactam or piperacillin-tazobactam are contraindicated in pregnant patients.

A

B.

Initial non-surgical management can be considered in hemodynamically normal pregnant patients experiencing cholelithiasis. This management plan generally involves bowel rest, intravenous hydration, and NSAID therapy.

A short course (< 48 to 72 h) of indomethacin treatment can provide effective analgesia but is generally avoided in late pregnancy due to the potential adverse fetal effects. Use in the third trimester increases the risk of premature closure of the patent ductus arteriosis and oligohydramnios.

Intravenous antibiotics to include ampicillin-sulbactam, piperacillin-tazobactam, and ticarcillin-clavulanate are not contraindicated in pregnant patients who need antibiotics for acute cholecystitis or choledocolithiasis.

Early surgery has been advocated for all types of biliary disease in pregnancy. If not treated, cholecystitis can lead to life threatening complications, the most common of which is gangrenous cholecystitis followed by abscess formation, perforation, fistula, ileus, or emphysematous cholecystitis. For symptomatic cholelithiasis with no evidence of cholecystitis, surgery can be delayed.

However the literature reports that surgical management of symptomatic cholelithiasis in pregnancy is safe, decreases hospital days, reduces emergency room visits, and the rate of preterm deliveries.

The second trimester (from 13 to 27 weeks gestation) is considered the best timeframe for cholecystectomy as the uterus is not obstructing the view of the surgical field, and the risk of miscarriage or preterm birth is lowest.

In this patient, with clear evidence of cholecystitis, surgical intervention is warranted to reduce risk of serious complications. If complications such as cholangitis or gallstone pancreatitis develop in a pregnant patient, maternal mortality may approach 15% and fetal loss up to 60%.

Surgical management of biliary disease has been revolutionized with the advent of laparoscopy. Laparoscopic technique can be utilized safely in pregnancy across all trimesters depending on the comfort level of the surgeon.

Although data is limited, laparoscopy does not confer an increased risk of adverse pregnancy outcomes to include preterm delivery as compared to laparotomy.

Proper positioning in left lateral tilt is important to reduce venal caval compression and maintain adequate placental blood flow, and open entry technique is recommended to prevent injury to the enlarged gravid uterus. Intraabdominal pressure with pneumoperitoneum should be limited to 10 to 12 mm Hg to reduce the theoretical concern of fetal acidosis associated with the effect of CO2.

There is no indication for intraoperative fetal monitoring.

Early surgery for appendicitis is also recommended in pregnant patients, as the consequences for both the mother and fetus can be catastrophic. Abbasi et al., in the largest case series to date involving 7000 patients, documents a markedly increased risk of severe complications with conservative management to include miscarriage and maternal sepsis. As with cholecystitis, non-operative management of appendicitis is contraindicated in pregnancy.

How well did you know this?
1
Not at all
2
3
4
5
Perfectly
17
Q

This patient undergoes an uncomplicated laparoscopic cholecystectomy. On postoperative day 2, she develops increasing pain in her right upper quadrant (RUQ) with fever and recurrent leukocytosis as well as elevated total bilirubin, transaminase, lipase and amylase levels. RUQ ultrasound documents dilated biliary ducts. Which of the following would be the most appropriate next step?

A. MRCP (magnetic resonance cholangio-pancreaticogram)

B. ERCP

C. Continued observation with antibiotic therapy

D. Repeat surgery with bile duct exploration

E. Delivery of the fetus

A

B.

Following cholecystectomy, this patient presents with findings consistent with choledocholithiasis, with associated gallstone pancreatitis and possible cholangitis.

The best option in this case is ERCP with the option of sphincterotomy to decompress the biliary tract. This approach appears safe in pregnant patients with early onset cholangitis with lower morbidity than conservative management.

In this patient, repeat surgery with intraoperative cholangiography or bile duct exploration would be a backup option, if the stones cannot be removed via ERCP.

Percutaneous biliary tract decompression would be another option in a high risk patient. MRCP is an excellent and safe diagnostic test in pregnancy and is a viable option in patients where the diagnosis is uncertain.

In a case with a high suspicion of cholangitis, this step could delay therapy which could have severe consequences for both fetus and mother.

More aggressive therapy for cholangitis is therefore indicated in pregnancy.

Although conservative treatment with continued intravenous antibiotics and observation may be appropriate in the non-gravid patient, the risks of this non-surgical approach are higher in the gravid patient and predispose her to grave complications.

Premature delivery of the fetus is not indicated for the treatment of biliary disease.

How well did you know this?
1
Not at all
2
3
4
5
Perfectly
18
Q

One year after an apparently uncomplicated cholecystectomy for chronic cholecystitis and cholelithiasis, a 37/F presents epigastric pain and jaundice of 2 wk duration. Ultrasonography reveals a dilated CBD along with dilated IHDs. To establish the nature of the obstruction, which diagnostic procedure is best:

a. ERCP
b. Percutaneous transhepatic cholangiography
c. CT scan
d. HIDA scan

A

a. ERCP

How well did you know this?
1
Not at all
2
3
4
5
Perfectly
19
Q

A 40/M from Davao presents with RUQ pain and fever. UTZ reveals a 6 cm complex mass in segment 7 of the liver. The treatment of choice is:

a. Percutaneous UTZ-guided aspiration
b. Laparoscopic surgery for drainage
c. Extra-peritoneal approach for drainage
d. Appropriate doses of metronidazole

A

d. Appropriate doses of metronidazole

How well did you know this?
1
Not at all
2
3
4
5
Perfectly
20
Q

A patient undergoes cholecystectomy, CBDE, T-tube choledochostomy for cholecystodocholithiasis. One week post-op, a T-tube cholangiogram shows multiple retained extrahepatic bile duct stones. Ideally this patient is best managed by:

a. choledochoscopic lithotripsy
b. re-exploration
c. extracorporeal shock wave lithotripsy
d. oral chenodeoxycholic acid

A

a. choledochoscopic lithotripsy

How well did you know this?
1
Not at all
2
3
4
5
Perfectly
21
Q

55/F consulted at the ER due to progressive painless jaundice, 1 day PTC, developed on and off of high grade fever associated with RUQ pain, on HBT-UTZ there was a cut off at the confluence of left and right IHDs, what will be your best treatment option at this time?

a. proceed with biliary exploration
b. ERCP with stenting
c. Bilateral percutaneous transhepatic biliary drainage
d. Conservative medical (antiobiotic / hydration) management

A

c. Bilateral percutaneous transhepatic biliary drainage

How well did you know this?
1
Not at all
2
3
4
5
Perfectly
22
Q

34/M underwent cholecystectomy, CBDE, t-tube choledochostomy. On follow up, the tube cholangiogram showed an impacted stone at the distal CBD. What is the best management for this patient?

a. flushing the t-tube with saline
b. choledochoscopy with stone extraction
c. ERCP, sphincterotomy with stone extraction
d. Re-exploration of the CBD

A

b. choledochoscopy with stone extraction

How well did you know this?
1
Not at all
2
3
4
5
Perfectly
23
Q

65/M came in at the ER due to abdominal pain and fever. On PE, the patient was noted to be jaundiced, tachycardic and hypotensive (70/40). Preliminary UTZ done at the ER showed dilated intrahepatic ducts. The best approach for this patient after resuscitation is:

a. Definitive surgery
b. Antibiotics alone
c. Immediate decompression
d. Conservative treatment

A

c. Immediate decompression

How well did you know this?
1
Not at all
2
3
4
5
Perfectly
24
Q

A 52-year-old male with a history of poorly controlled diabetes presents to the emergency department complaining of right upper quadrant pain and vomiting. An ultrasound is performed that shows gallstones with air in the gallbladder wall. Which of the following is the BEST next step in management of this patient?

a. IV antibiotics and cholecystectomy in the next 24-48 hours
b. Schedule elective cholecystectomy as an outpatient
c. ERCP
d. Emergent cholecystectomy

A

d. Emergent cholecystectomy

How well did you know this?
1
Not at all
2
3
4
5
Perfectly
25
Q

List the ligaments of the liver:

A

1) The falciform ligament,
2) coronary ligaments, and
3) the right and left triangular ligaments

How well did you know this?
1
Not at all
2
3
4
5
Perfectly
26
Q

What structures are contained within the hepatoduodenal ligament?

A

Porta hepatis:

1) proper hepatic artery
2) portal vein
3) common bile duct (CBD)

How well did you know this?
1
Not at all
2
3
4
5
Perfectly
27
Q

What line divides the liver into left and right lobes?

A

Cantlie line (runs from the middle of the gallbladder fossa anteriorly to the inferior vena cava posteriorly)

How well did you know this?
1
Not at all
2
3
4
5
Perfectly
28
Q

How many Couinaud segments are in the liver?

A

8

How well did you know this?
1
Not at all
2
3
4
5
Perfectly
29
Q

Which segments are removed in a right hepatectomy?

A

Segments 5 to 8

How well did you know this?
1
Not at all
2
3
4
5
Perfectly
30
Q

Which segments are removed in a right trisegmentectomy?

A

Segments 4 to 8

How well did you know this?
1
Not at all
2
3
4
5
Perfectly
31
Q

Which segments removed are in a left hepatectomy?

A

Segments 2 to 4

How well did you know this?
1
Not at all
2
3
4
5
Perfectly
32
Q

Which segments are removed in a left lateral trisegmentectomy?

A

Segments 2 and 3

How well did you know this?
1
Not at all
2
3
4
5
Perfectly
33
Q

Which segments are removed in a left trisegmentectomy?

A

Segments 2, 3, 4, 5, and 8

How well did you know this?
1
Not at all
2
3
4
5
Perfectly
34
Q

How much percentage of blood flow does the portal vein supply to the liver?

A

70%

How well did you know this?
1
Not at all
2
3
4
5
Perfectly
35
Q

What percentage of cardiac output accounts for hepatic blood flow?

A

25%

How well did you know this?
1
Not at all
2
3
4
5
Perfectly
36
Q

How much of the liver’s oxygen supply is provided by the hepatic artery?

A

50%

In Schwartz: The hepatic artery delivers approximately 25% of the blood supply, and the portal vein 75%.

How well did you know this?
1
Not at all
2
3
4
5
Perfectly
37
Q

How many hepatic veins are there?

A

Three (right, middle, left)

How well did you know this?
1
Not at all
2
3
4
5
Perfectly
38
Q

What is the approximate length of the CBD?

A

Approximately 7 cm

How well did you know this?
1
Not at all
2
3
4
5
Perfectly
39
Q

What is the normal CBD diameter?

A

0.5 to 1.5 cm. Depends on age and if gallbladder is still present.

Age: 0.2 cm to 1 cm (age above seventies).

If gallbladder removed, then CBD enlarges few mm.

How well did you know this?
1
Not at all
2
3
4
5
Perfectly
40
Q

What is the arterial supply of the bile ducts?

A

Primarily the right hepatic artery

How well did you know this?
1
Not at all
2
3
4
5
Perfectly
41
Q

Where does the sympathetic innervation of the gallbladder originate?

A

Celiac axis

How well did you know this?
1
Not at all
2
3
4
5
Perfectly
42
Q

What is the parasympathetic innervation of the gallbladder?

A

Vagus nerve

How well did you know this?
1
Not at all
2
3
4
5
Perfectly
43
Q

What are ducts of Luschka?

A

Accessory ducts directly from the liver bed into the gallbladder

How well did you know this?
1
Not at all
2
3
4
5
Perfectly
44
Q

How much bilirubin is produced daily in normal adults?

A

About 250 to 350 mg

How well did you know this?
1
Not at all
2
3
4
5
Perfectly
45
Q

How much bilirubin is produced from turnover of senescent red blood cells?

A

Approximately 85%

How well did you know this?
1
Not at all
2
3
4
5
Perfectly
46
Q

What is the most common cause of hyperbilirubinemia in adults?

A

Cholelithiasis

How well did you know this?
1
Not at all
2
3
4
5
Perfectly
47
Q

What is the major metabolite of heme?

A

Bilirubin

How well did you know this?
1
Not at all
2
3
4
5
Perfectly
48
Q

What is the name of sinusoidal macrophages?

A

Kupffer cells

How well did you know this?
1
Not at all
2
3
4
5
Perfectly
49
Q

What is the half-life of albumin?

A

20 days

How well did you know this?
1
Not at all
2
3
4
5
Perfectly
50
Q

What is the half-life of transferrin?

A

6days

How well did you know this?
1
Not at all
2
3
4
5
Perfectly
51
Q

Which major protein is produced by the liver?

A

Albumin

How well did you know this?
1
Not at all
2
3
4
5
Perfectly
52
Q

What are bile adds conjugated with to form bile salts?

A

Glycine and taurine

How well did you know this?
1
Not at all
2
3
4
5
Perfectly
53
Q

What is the major site of bile acid reabsorption?

A

Distal ileum

How well did you know this?
1
Not at all
2
3
4
5
Perfectly
54
Q

What autosomal recessive disorder causes an increase in conjugated bilirubin without elevation of liver function tests (LFTs)?

A

Dubin-Johnson syndrome

How well did you know this?
1
Not at all
2
3
4
5
Perfectly
55
Q

What percentage of the population has gallstones?

A

Approximately 10%

How well did you know this?
1
Not at all
2
3
4
5
Perfectly
56
Q

What group of Indians in southern Arizona are at high risk for gallstones?

A

Pima Indians

How well did you know this?
1
Not at all
2
3
4
5
Perfectly
57
Q

What population is at lowest risk for developing gallstones?

A

Sub-Saharan Africans

How well did you know this?
1
Not at all
2
3
4
5
Perfectly
58
Q

How are gallstones classified?

A

Cholesterol, black pigment, or brown pigment stones

How well did you know this?
1
Not at all
2
3
4
5
Perfectly
59
Q

What is the main composition of black pigmented stones?

A

Calcium bilirubinate

How well did you know this?
1
Not at all
2
3
4
5
Perfectly
60
Q

What is the main composition of brown pigmented stones?

A

Calcium salts of unconjugated bilirubin. Incidence increased with systemic infections.

How well did you know this?
1
Not at all
2
3
4
5
Perfectly
61
Q

What causes gallstone formation?

A

Cholesterol supersaturation, accelerated crystal nucleation, and gallbladder hypomotility

How well did you know this?
1
Not at all
2
3
4
5
Perfectly
62
Q

What are the 2 primary bile acids?

A

Cholate and chenodeoxycholate

How well did you know this?
1
Not at all
2
3
4
5
Perfectly
63
Q

What is the most common inherited cause of hyperbilirubinemia?

A

Gilbert syndrome (decreased activity of the enzyme glucuronyl transferase)

How well did you know this?
1
Not at all
2
3
4
5
Perfectly
64
Q

Which enzyme found in the cells of the bile duct rises with bile duct obstruction?

A

Alkaline phosphatase

How well did you know this?
1
Not at all
2
3
4
5
Perfectly
65
Q

The liver is the site of synthesis of all of the coagulation factors except:

A

von Willebrand factor

How well did you know this?
1
Not at all
2
3
4
5
Perfectly
66
Q

What is type I Crigler-Najjar syndrome?

A

Severe unconjugated hyperbilirubinemia, kernicterus, and bilirubin deposits in the brain causing severe motor dysfunction and retardation

How well did you know this?
1
Not at all
2
3
4
5
Perfectly
67
Q

What is type II Crigler-Najjar syndrome?

A

Less severe form, enzyme activity is 10% of normal

How well did you know this?
1
Not at all
2
3
4
5
Perfectly
68
Q

List the secondary bile salts:

A

Lithocholate, deoxycholate, and ursodeoxycholate

How well did you know this?
1
Not at all
2
3
4
5
Perfectly
69
Q

What hormone is a potent stimulator of gallbladder contraction?

A

Cholecystokinin

Study tip: CCK-HIDA is ordered for the workup of biliary dyskinesia.

EF <35% confirms the diagnosis of biliary dyskinesia.

How well did you know this?
1
Not at all
2
3
4
5
Perfectly
70
Q

Is prophylactic cholecystectomy recommended in diabetics?

A

No

How well did you know this?
1
Not at all
2
3
4
5
Perfectly
71
Q

What is the incidence of endoscopic retrograde cholangiopancreatography (ERCP)- induced pancreatitis?

A

About 5%

How well did you know this?
1
Not at all
2
3
4
5
Perfectly
72
Q

Which is a rapid, noninvasive imaging study that provides detailed biliary tree and pancreatic duct images equal to those of ERCP?

A

Magnetic resonance cholangiopancreatography (MRCP)

How well did you know this?
1
Not at all
2
3
4
5
Perfectly
73
Q

What percentage of pigmented stones and cholesterol stones are seen on plain abdominal films?

A

50% and 20%, respectively

How well did you know this?
1
Not at all
2
3
4
5
Perfectly
74
Q

What is the principal imaging modality for the diagnosis of cholelithiasis?

A

Ultrasonography

How well did you know this?
1
Not at all
2
3
4
5
Perfectly
75
Q

What is the sensitivity of ultrasonography for stones greater than 2 mm?

A

> 95%

How well did you know this?
1
Not at all
2
3
4
5
Perfectly
76
Q

What is the probability of complications requiring surgery in a patient with a history of biliary colic?

A

1% to 2% per year

How well did you know this?
1
Not at all
2
3
4
5
Perfectly
77
Q

What is the surgical management of uncomplicated biliary colic and documented gallstones?

A

Elective laparoscopic cholecystectomy

How well did you know this?
1
Not at all
2
3
4
5
Perfectly
78
Q

What percentage of patients with acute cholecystitis have positive enteric bacteria culture from the bile?

A

50%

How well did you know this?
1
Not at all
2
3
4
5
Perfectly
79
Q

What is hydrops of the gallbladder?

A

Chronic cystic duct obstruction and gallbladder distention with clear mucoid fluid

How well did you know this?
1
Not at all
2
3
4
5
Perfectly
80
Q

What is Mirizzi syndrome?

A

Impacted stone in the gallbladder neck causing extrinsic compression of the common (hepatic) duct

How well did you know this?
1
Not at all
2
3
4
5
Perfectly
81
Q

What is porcelain gallbladder?

A

Calcification of the gallbladder wall; cholecystectomy is indicated to prevent carcinoma.

Past estimates of gallbladder cancer in porcelain gallbladder were approximately 20%.

Recent reports suggest the estimate is lower than that, ranging between 7% and 15%.

How well did you know this?
1
Not at all
2
3
4
5
Perfectly
82
Q

What is Murphy sign?

A

Inspiratory arrest upon right upper quadrant (RUQ) palpation

How well did you know this?
1
Not at all
2
3
4
5
Perfectly
83
Q

What percentage of patients with gallstones have CBD stones?

A

15% to 20%

How well did you know this?
1
Not at all
2
3
4
5
Perfectly
84
Q

What is the most common cause of cholangitis?

A

Stone impacted in the CBD (85% of the time)

How well did you know this?
1
Not at all
2
3
4
5
Perfectly
85
Q

Which organisms are most commonly cultured from bile?

A

Escherichia coli, Klebsiella, Pseudomonas, Proteus, enterococci

How well did you know this?
1
Not at all
2
3
4
5
Perfectly
86
Q

What is the constellation of RUQ pain, jaundice, and fever?

A

Charcot triad

How well did you know this?
1
Not at all
2
3
4
5
Perfectly
87
Q

What is Reynold pentad?

A

Altered mental status and hypotension plus Charcot triad

How well did you know this?
1
Not at all
2
3
4
5
Perfectly
88
Q

Which autoimmune disease is associated with destruction of extrahepatic and intrahepatic bile ducts?

A

Primary sclerosing cholangitis

How well did you know this?
1
Not at all
2
3
4
5
Perfectly
89
Q

What is a type I choledochal cyst?

A

Saccular or fusiform dilatation of the common hepatic and CBD

How well did you know this?
1
Not at all
2
3
4
5
Perfectly
90
Q

What is a type II choledodial cyst?

A

This choledochal cyst is characterized as a diverticulum protruding from the CBD

How well did you know this?
1
Not at all
2
3
4
5
Perfectly
91
Q

What is a type III choledochal cyst?

A

Choledochocele; found in the intraduodenal portion of the CBD

How well did you know this?
1
Not at all
2
3
4
5
Perfectly
92
Q

What is a type IV choledochal cyst?

A

Choledochal cyst that involves the extrahepatic bile duct and intrahepatic ducts

How well did you know this?
1
Not at all
2
3
4
5
Perfectly
93
Q

What is a type V choledochal cyst?

A

Caroli disease (multiple intrahepatic cysts)

How well did you know this?
1
Not at all
2
3
4
5
Perfectly
94
Q

List the areas where dilated collateral veins are seen in patients with severe cirrhosis draining into the systemic circulation:

A

Hemorrhoidal, azygos, renal, adrenal veins, paraumbilical vein from abdominal wall

How well did you know this?
1
Not at all
2
3
4
5
Perfectly
95
Q

What is the most common cause of hemobilia?

A

Iatrogenic trauma to the liver and biliary tree

How well did you know this?
1
Not at all
2
3
4
5
Perfectly
96
Q

What is the first test to rule out hemobilia?

A

Esophagogastroduodenoscopy

How well did you know this?
1
Not at all
2
3
4
5
Perfectly
97
Q

What is the first-line therapy for hemobilia?

A

Angiography (embolization)

How well did you know this?
1
Not at all
2
3
4
5
Perfectly
98
Q

What is the treatment of pyogenic liver abscess?

A

Antibiotics and percutaneous drainage

How well did you know this?
1
Not at all
2
3
4
5
Perfectly
99
Q

What is the antibiotic course for pyogenic liver abscess?

A

Broad-spectrum intravenous antibiotics for 2 to 3 weeks followed by 4 to 6 weeks of oral antibiotics.

Duration of antibiotics is controversial, but most would give at least 2 weeks if no signs of bacteremia or sepsis.

How well did you know this?
1
Not at all
2
3
4
5
Perfectly
100
Q

What is the most common cause of hydatid disease?

A

Echinococcus granulosus

How well did you know this?
1
Not at all
2
3
4
5
Perfectly
101
Q

What cyst may rupture and result in anaphylactic shock?

A

Echinococcal cyst

How well did you know this?
1
Not at all
2
3
4
5
Perfectly
102
Q

What skin test is used in the diagnosis of hydatid disease?

A

Casoni test (intradermal injection of hydatid fluid)

How well did you know this?
1
Not at all
2
3
4
5
Perfectly
103
Q

What in the wall of a liver cyst is highly suggestive of hydatid disease?

A

Calcifications

How well did you know this?
1
Not at all
2
3
4
5
Perfectly
104
Q

What abnormality may be seen on complete blood count in about 25% of patients with echinococcal cyst?

A

Eosinophilia

How well did you know this?
1
Not at all
2
3
4
5
Perfectly
105
Q

What is the medical treatment of choice for Echinococcus?

A

Mebendazole/albendazole

How well did you know this?
1
Not at all
2
3
4
5
Perfectly
106
Q

What technique describes percutaneous management of hydatid cysts in patients who refuse or cannot undergo surgery?

A

PAIR (Percutaneous Aspiration, Injection, and Reaspiration)

How well did you know this?
1
Not at all
2
3
4
5
Perfectly
107
Q

What is the most common scolicidal agent used in hydatid cyst injection?

A

20% sodium chloride solution

How well did you know this?
1
Not at all
2
3
4
5
Perfectly
108
Q

What are surgical options for hydatid cysts?

A

Pericystectomy, partial hepatectomy

Cyst must be excised in whole without spilling its contents

How well did you know this?
1
Not at all
2
3
4
5
Perfectly
109
Q

What is the recurrence rate of postoperative hydatid cysts?

A

About 20%

How well did you know this?
1
Not at all
2
3
4
5
Perfectly
110
Q

What parasite causes amebiasis if transmitted through a few-oral route?

A

Entamoeba histolytica

How well did you know this?
1
Not at all
2
3
4
5
Perfectly
111
Q

What is the appropriate treatment of cyst rupture and bile duct obstrumon?

A

ERCP with papillotomy

How well did you know this?
1
Not at all
2
3
4
5
Perfectly
112
Q

What liver abscess is typically described as having an anchovy-paste appearance?

A

Amebic liver abscess

How well did you know this?
1
Not at all
2
3
4
5
Perfectly
113
Q

What is the treatment of amebic abscess?

A

750 mg of metronidazole 3 times a day for 10 days

How well did you know this?
1
Not at all
2
3
4
5
Perfectly
114
Q

What is the most common cause of intrahepatic presinusoidal hypertension worldwide?

A

Schistosomiasis

How well did you know this?
1
Not at all
2
3
4
5
Perfectly
115
Q

What is Budd-Chiari syndrome?

A

Hepatic venous outflow obstruction

How well did you know this?
1
Not at all
2
3
4
5
Perfectly
116
Q

What are some causes of Budd-Chiari syndrome?

A

Polycythemia vera (most common reason in Western part of the world), factor V Leiden mutation, thrombocytosis, protein C and S, antithrombin III, antiphospholipid antibody syndrome

How well did you know this?
1
Not at all
2
3
4
5
Perfectly
117
Q

What are possible sequelae ofhepatic adenomas?

A

Rupture or malignancy

How well did you know this?
1
Not at all
2
3
4
5
Perfectly
118
Q

What liver tumor is associated with oral contraceptive pill use?

A

Hepatic adenoma

How well did you know this?
1
Not at all
2
3
4
5
Perfectly
119
Q

How would you manage a patient with hepatic adenoma who wants to get pregnant?

A

Elective resection of hepatic adenoma

How well did you know this?
1
Not at all
2
3
4
5
Perfectly
120
Q

What liver mass is characterized by a central stellate scar seen on computed tomography (CT) imaging?

A

FNH

How well did you know this?
1
Not at all
2
3
4
5
Perfectly
121
Q

What is the indication for resection of FNH?

A

Symptomatic FNH or an enlarging lesion

How well did you know this?
1
Not at all
2
3
4
5
Perfectly
122
Q

What is Kasabach-Merritt syndrome?

A

Hepatic hemangioma, thrombocytopenia, and consumptive coagulopathy

How well did you know this?
1
Not at all
2
3
4
5
Perfectly
123
Q

What is the most common benign liver tumor?

A

Hemangioma

How well did you know this?
1
Not at all
2
3
4
5
Perfectly
124
Q

What are the 2 types of hepatic hemangioma?

A

Capillary hemangioma (clinically insignificant) and cavernous hemangioma

How well did you know this?
1
Not at all
2
3
4
5
Perfectly
125
Q

What diagnostic tests are indicated?

A

CT with IV contrast

How well did you know this?
1
Not at all
2
3
4
5
Perfectly
126
Q

Should biopsy be performed for a suspected hepatic hemangioma?

A

No (hemorrhage risk!)

How well did you know this?
1
Not at all
2
3
4
5
Perfectly
127
Q

Which hepatitis virus is transmitted by the fecal-oral route via contaminated food?

A

Hepatitis A

How well did you know this?
1
Not at all
2
3
4
5
Perfectly
128
Q

Which tests indicate acute hepatitis B virus (HBV) infection?

A

IgM anti-HBc, HBsAg

How well did you know this?
1
Not at all
2
3
4
5
Perfectly
129
Q

Which tests indicate chronic HBV infection?

A

HBsAg, IgG anti-HBc

How well did you know this?
1
Not at all
2
3
4
5
Perfectly
130
Q

What is caput medusae?

A

Dilation of paraumbilical veins arising from the left portal vein and extending to the umbilicus producing umbilical and abdominal wall varices

How well did you know this?
1
Not at all
2
3
4
5
Perfectly
131
Q

Where can collaterals form as a result of portal hypertension?

A

Esophagus, retroperitoneum, rectum, abdominal wall

How well did you know this?
1
Not at all
2
3
4
5
Perfectly
132
Q

What are the 3 categories of portosystemic shunts?

A

Nonselective, selective, and partial shunts

How well did you know this?
1
Not at all
2
3
4
5
Perfectly
133
Q

What are 2 basic types of nonselective shunts?

A

End-to-side portacaval shunt, side-to-side portosystemic shunt

How well did you know this?
1
Not at all
2
3
4
5
Perfectly
134
Q

What procedure has all but supplanted nonselective shunts?

A

Transjugular intrahepatic portosystemic shunt (TIPS)

How well did you know this?
1
Not at all
2
3
4
5
Perfectly
135
Q

What is the only commonly applied partial portosystemic shunt?

A

Interposition portacaval shunt

How well did you know this?
1
Not at all
2
3
4
5
Perfectly
136
Q

What is the surgical management of distal cholangiocarcinoma?

A

Pancreatoduodenectomy (Whipple)

How well did you know this?
1
Not at all
2
3
4
5
Perfectly
137
Q

What name is used to describe cholangiocarcinoma at the bifurcation of the common hepatic duct?

A

Klatskin tumor

How well did you know this?
1
Not at all
2
3
4
5
Perfectly
138
Q

What criteria for cholangiocarcinoma make it unresectable?

A

Bilateral hepatic artery involvement, encasement of the portal vein, bilateral hepatic duct involvement up to secondary radicals

How well did you know this?
1
Not at all
2
3
4
5
Perfectly
139
Q

What is the management of intrahepatic cholangiocarclnoma?

A

Hepatic resection

How well did you know this?
1
Not at all
2
3
4
5
Perfectly
140
Q

What is the management of perihilar cholangiocarcinoma?

A

Resection ofthe extrahepatic bile duct, cholecystectomy, and hepaticojejunostomy

How well did you know this?
1
Not at all
2
3
4
5
Perfectly
141
Q

Which potent hepatotoxin is produced by Aspergillus species?

A

Aflatoxin

How well did you know this?
1
Not at all
2
3
4
5
Perfectly
142
Q

What is the most common malignant hepatic tumor?

A

Metastases

How well did you know this?
1
Not at all
2
3
4
5
Perfectly
143
Q

What is the most common primary liver cancer worldwide?

A

Hepatocellular carcinoma (HCC)

How well did you know this?
1
Not at all
2
3
4
5
Perfectly
144
Q

What is the most common cause of HCC?

A

Hepatitis B and C

How well did you know this?
1
Not at all
2
3
4
5
Perfectly
145
Q

What distinct clinical variant of HCC is a well-circumscribed solitary lesion with a central scar?

A

Fibrolamellar carcinoma (better prognosis)

How well did you know this?
1
Not at all
2
3
4
5
Perfectly
146
Q

Which tumor marker may be helpful in the diagnosis of HCC?

A

a-Fetoprotein (AFP)

How well did you know this?
1
Not at all
2
3
4
5
Perfectly
147
Q

What are possible treatment options for HCC?

A

Partial hepatectomy, total hepatectomy with transplantation, ablation, embolization

How well did you know this?
1
Not at all
2
3
4
5
Perfectly
148
Q

This hepatic lesion does not produce AFP, but is associated with elevated neurotensin levels:

A

Fibrolamellar HCC

How well did you know this?
1
Not at all
2
3
4
5
Perfectly
149
Q

What is the most common primary hepatic tumor of childhood?

A

Hepatoblastoma

How well did you know this?
1
Not at all
2
3
4
5
Perfectly
150
Q

What hepatic sarcoma is associated with vinyl chloride, thorotrast, and arsenic?

A

Angiosarcoma

How well did you know this?
1
Not at all
2
3
4
5
Perfectly
151
Q

Which of the following lesions should be resected in the asymptomatic patient?

A. 5-cm FNH

B. 3-cm hepatic adenoma

C. 4-cm hydatid cyst

D. 6-cm hepatic hemangioma

E. 2-cm hepatic hamartoma

A

Answer: B.

Although regression of hepatic adenomas has been reported after discontinuation of oral contraceptives, the potential for bleeding and malignant transformation favors routine resection.

How well did you know this?
1
Not at all
2
3
4
5
Perfectly
152
Q

Which of the following is true regarding hemobilia?

A. Cholelithiasis is the most important cause of hemobilia.

B. Hepatoblastoma is the most commonly associated tumor.

C. It always presents with acute upper or lower gastrointestinal bleeding.

D. A tagged RBC scan is the diagnostic modality of choice.

E. Angioembolization is the initial definitive treatment of choice.

A

Answer: E.

Hemobilia is bleeding within the biliary tract. Causes include trauma, surgery, malignancy, and infections. Angiography is the most accurate and helpful diagnostic and therapeutic modality.

How well did you know this?
1
Not at all
2
3
4
5
Perfectly
153
Q

A 2-year-old boy is found to have fusiform dilation of the extrahepatic biliary duct. What is the best treatment option?

A. Transduodenal cyst excision

B. Excision with primary choledochorrhaphy

C. Complete cyst excision with Roux-en-Y reconstruction

D. Liver transplantation

E. Observation

A

Answer: C.

Fusiform extrahepatic dilation is a type I choledochal cyst.

Surgical resection is necessary to avoid recurrent episodes of infection from stasis of the bile within the cyst cavity, as well as the risk of developing a cholangiocarcinoma.

Type III cysts usually do not require resection.

Types I and II do have a cancer risk.

Therefore, the extrahepatic bile duct is resected and a Roux-en-Y choleenterostomy is performed.

How well did you know this?
1
Not at all
2
3
4
5
Perfectly
154
Q

What is the best treatment for a 6-cm encapsulated cyst of the right lobe of the liver in a febrile patient?

A. Right hepatectomy

B. Percutaneous drainage

C. Marsupialization

D. Albendazole

E. Metronidazole

A

Answer: B.

Pyogenic liver abscesses are best treated with initial drainage and broad-spectrum antibiotics.

Albendazole is reserved for hydatid/echinococcal cysts, while metronidazole is used for amebic cysts.

Resection and marsupialization are not first-line treatments of benign liver cysts.

How well did you know this?
1
Not at all
2
3
4
5
Perfectly
155
Q

Which of the following methods is the most accurate for detection of hepatic metastases?

A. Intraoperative palpation

B. CT

C. Laparoscopy

D. Transabdominal ultrasound

E. lntraoperative ultrasound

A

Answer: E.

Performed with handheld or laparoscopic transducers, intraoperative ultrasound has been established as the most accurate method to detect hepatic metastases.

It is a useful modality for identifying vascular structures and their relation to liver lesions, hence enabling an evaluation for resectability.

CT scanning is accurate for larger lesions, usually measuring more than 2 cm.

How well did you know this?
1
Not at all
2
3
4
5
Perfectly
156
Q

A 55-year-old man undergoes a laparoscopic cholecystectomy for acute cholecystitis. The final pathology report indicates gallbladder carcinoma that invades the muscularis. Which of the following would be the most appropriate management?

A. Radiation

B. Chemotherapy

C. Combined chemotherapy and radiation therapy

D. Observation

E. Reoperation with wedge resection of liver around the gallbladder fossa with regional lymph node dissection

A

Answer: E.

Gallbladder adenocarcinoma that is classified as carcinoma in situ or T1a (invades lamina propria) and that has negative margins if discovered incidentally
and removed can be managed by cholecystectomy alone.

Lesions that are T1b (invades muscularis propria layer), T2 (invades the perimuscular connective tissue that is superficial to the muscularis propria), or greater are treated with a radical cholecystectomy, which consists of subsegmental resection of liver segments 4b and 5, in addition to a hepatoduodenal ligament lymphadenectomy and cystic duct excision if the margin is positive.

Remember that the gallbladder has no submucosa.

Adjuvant chemotherapy and chemotherapy for unresectable disease can be considered.

How well did you know this?
1
Not at all
2
3
4
5
Perfectly
157
Q

A 25-year-old man presents with high fever and RUQ pain. His WBC count is 19,000. An ultrasound shows a 6-cm fluid collection in the right lobe of the liver. On CT scan, the fluid collection shows a peripheral rim of edema. The cause of the fluid collection is most likely determined by:

A. Blood and stool cultures

B. Percutaneous aspiration of the liver lesion

C. Serologic tests

D. LFTs

E. MRCP

A

Answer: C.

Amebic liver abscesses are caused by E. histolytica. Diagnosis is made by clinical presentation, ultrasound, and, most notably and distinctly, by CT or serologic testing.

The classic finding on CT is a single fluid collection in the right lobe with a rim ofperipheral edema, which distinguishes this entity from pyogenic and echinococcal cysts.

How well did you know this?
1
Not at all
2
3
4
5
Perfectly
158
Q

A cirrhotic patient presents with asymptomatic umbilical hernia with overlying erythematous skin changes. What is the most appropriate management?

A. Observation and delayed repair

B. Repair with primary fascial closure

C. Repair with synthetic permanent mesh

D. Repair with biologic mesh

E. Therapeutic paracentesis

A

Answer: C.

A recent Dutch study suggests that after excluding patients with an intact umbilical vein, umbilical hernia repair can be performed safely in patients with liver cirrhosis.

The authors point out that the ideal time to repair such hernias would be at the time of transplantation.

However, for many patients, the waiting time prior to transplantation can be prolonged, increasing the likelihood of an incarcerated hernia developing. Thus, it may be prudent to perform herniorrhaphy on some of these patients while they are awaiting liver transplantation.

Moreover, a permanent mesh can be used in complicated hernias in cirrhotic patients with minimal wound-related morbidity and a significantly lower rate of recurrence.

How well did you know this?
1
Not at all
2
3
4
5
Perfectly
159
Q

A 38-year-old woman presents with RUQ pain. Imaging with CT shows a 3-cm mass in segment 7 of the liver that is well demarcated with a central fibrotic area. This central scar does not enhance in the arterial phase of the CT scan. Workup shows no evidence of cirrhosis, hepatitis and a normal AFP level, and slightly elevated neurotensin. What is the diagnosis?

A. FNH

B. Hepaticadenoma

C. Fibrolamellar carcinoma

D. Hepatoma

E. Hemangioma

A

Answer: C.

The fibrolamellar variant of HCC (or hepatoma) is now sometimes considered a distinct pathologic entity.

It occurs in younger patients.

Unlike HCC, most patients are not cirrhotic, not hepatitis B positive, and have normal AFP levels and may have elevated neurotensin levels.

They tend to have a better prognosis than HCC.

The tumor is well demarcated and generally has a central fibrotic area.

As opposed to the central scar seen in FNH, this is not a vascular lesion and therefore will not enhance in the arterial phase of the CT, nor with gadolinium-enhanced MRI, whereas FNH will.

How well did you know this?
1
Not at all
2
3
4
5
Perfectly
160
Q

The Model for End-stage Liver Disease (MELD) score is based on what 3 parameters?

A. International normalized ratio {INR), creatinine, bilirubin

B. Albumin, creatinine, alkaline phosphatase

C. INR, bilirubin, AST

D. Amylase, GGT, albumin

E. AST, ALT, alkaline phosphatase

A

Answer: A.

The MELD is a scoring system for assessing the severity of chronic liver disease. It was initially developed to predict death within 3 months of surgery in patients who had undergone a TIPS procedure and was subsequently found to be useful in determining the prognosis and prioritizing for receipt of a liver transplant.

This score is now used by the United Network for Organ Sharing and Eurotransplant for prioritizing allocation of liver transplants instead of the older Child-Pugh score.

It is based on the INR, creatinine, and bilirubin, thereby avoiding the inclusion of subjective parameters that are incorporated in the previously used Child-Pugh score.

More recently, a modified MELD score has been used. The modified score incorporates sodium level into its calculation.

How well did you know this?
1
Not at all
2
3
4
5
Perfectly
161
Q

What is the first step to clear a CBD stone found on an intraoperative cholangiogram?

A. Transcystic duct exploration

B. Administer IV glucagon and flush duct with saline

C. Finish the laparoscopic cholecystectomy with plans for postoperative ERCP

D. Open CBD exploration

E. Choledochotomy and T-tube insertion

A

Answer: B.

Flushing the biliary tree with saline through the cystic duct and administering IV glucagon frequently helps in clearing the CBD of stones, especially if they are small.

Glucagon relaxes the sphincter of Oddi, which helps in clearing the stones.

If that fails, then the next step would be passing a Fogarty catheter or a basket under fluoroscopy for stone retrieval.

If that fails, then a choledoscope can be inserted along with a basket or wire catheter.

If that fails, then a choledochotomy is made and again a choledoscope can be used, or open exploration is commenced.

How well did you know this?
1
Not at all
2
3
4
5
Perfectly
162
Q

Which of the following demonstrates a peripheral-to-central enhancement on CT with IV contrast?

A. Focal nodular hyperplasia

B. Adenoma

C. Hemangioma

D. HCC

E. Hepatic metastases

A

Answer: C.

Hemangiomas have peripheral to central enhancement on CT scan.

The centripetal enhancement can be seen on the portal venous phase of the CT scan.

FNH has a central stellate scar.

Adenomas appear as a well-circumscribed mass and have homogenous enhancement on the arterial phase.

How well did you know this?
1
Not at all
2
3
4
5
Perfectly
163
Q

Which of the following is a correct statement?

A. The cystic artery is most commonly a branch of the left hepatic artery.

B. The blood supply for the CBD enters at the 12 and 6 o’clock positions.

C. The portal vein is the major venous drainage system of the liver.

D. The CBD lies posteriorly in the portal triad.

E. Choledochotomy is done vertically.

A

Answer: E.

The CBD lies lateral to the hepatic artery proper and anterior to the portal vein in the portal triad.

Its blood supply enters the duct at the 3 and 9 o’clock positions.

So a choledochotomy is done vertically to avoid injuring the blood supply of the CBD.

The cystic artery is most commonly a branch of the right hepatic artery.

The portal vein does not drain the liver; on the contrary, it supplies blood to the liver.

How well did you know this?
1
Not at all
2
3
4
5
Perfectly
164
Q

Which of the following statements regarding metastases to the liver is true?

A. The hepatic artery is the major blood supply of the metastases.

B. The most common primary cancer is lung.

C. Primary liver cancers are more common than metastases.

D. Resection of colorectal cancer metastasis to the liver does not improve survival.

E. Resection of metastasis to the liver is recommended only if there are fewer than 2 metastases and each is smaller than 3 cm

A

Answer: A.

Metastasis to the liver is fairly common with multiple tumors, including colorectal, pancreatic, cholangiocarcinoma, lungs, and breast.

It is more common than primary liver cancers.

The most common primary cancer is colorectal.

Resection of colorectal metastases to the liver improves 5-year survival to approximately 30%. The blood supply of the metastasis is from the hepatic artery.

This is clinically significant in certain treatment modalities such as chemoembolization.

Metastasectomy is recommended regardless of size and number of metastases as long as the remaining liver is functional (>20%).

How well did you know this?
1
Not at all
2
3
4
5
Perfectly
165
Q

A 42-year-old immigrant presents to the ED with right upper quadrant pain. Workup reveals a large cyst occupying his right liver with a calcified wall and multiple small other cysts in the same lobe. Which of the following is the most appropriate therapy?

A. Albendazole only

B. Marsupialization of the cyst

C. Right hepatectomy and adjuvant chemotherapy

D. IR drainage of the largest cyst

E. Aspiration of the contents, injection of hypertonic saline, followed by cystectomy

A

Answer: E.

This patient is presenting with hydatid disease. It is due to E. granulosus in most instances.

The therapy is mainly surgical, including resection of the cyst after aspirating its contents and injecting it with a scolicidal agent to kill the scolices.

Agents include hypertonic saline.

Towels should be placed around the liver to minimize the risk of anaphylaxis in case of spillage.

There is no role for chemotherapy or antihelminthics alone.

Formal resection is aggressive and not required.

How well did you know this?
1
Not at all
2
3
4
5
Perfectly
166
Q

Which of the following statements about the anatomy of the liver is true?

A. The right and left lobe are divided by the falciform ligament.

B. The portal triad consists of the hepatic artery, hepatic vein, and bile duct.

C. The caudate lobe has its own separate arterial supply.

D. The right lobe contains anterior and posterior segments.

E. The American system describes eight segments.

A

ANSWER: D

COMMENTS: The surgical anatomy of the liver is based on the distribution of the hepatic veins and portal structures, and it has been modified several times.

There are two main anatomic classification systems for the liver: the American system and the French system.

In both these systems, the liver is divided into right and left lobes by the Cantlie line, a longitudinal plane that extends from the gallbladder fossa to the inferior vena cava. This plane, also called the portal fissure, contains the middle hepatic vein and the bifurcation of the portal vein.

In the American system, the liver is further broken down into four segments, with each lobe containing two segments. The right lobe of the liver consists of posterior and anterior segments. The left lobe consists of a medial segment (quadrate lobe) and a lateral segment divided by the falciform ligament. The caudate lobe can be considered anatomically independent of the right and left lobes because it receives portal and arterial blood supply from both sides and has venous drainage directly into the inferior vena cava.

In the French system, developed by C. Couinaud, the two lobes of the liver are broken down into eight segments. These eight segments are formed by three vertical planes (scissurae) created by the right, middle, and left hepatic veins, which results in four sectors. These four sectors are further divided by a plane created by the branching portal system. Therefore the left lobe, according to the French system, is divided into medial and lateral segments by the left hepatic vein. The lateral sector of the left lobe consists of a superior segment (II) and an inferior segment (III). The medial sector of the left lobe is segment IV. The right lobe consists of anteromedial and posterolateral sectors divided by a vertical plane containing the right hepatic vein. The anteromedial sector is made up of segment V (inferior) and segment VIII (superior), and the posterolateral sector is made up of segment VI (inferior) and segment VII (superior).

How well did you know this?
1
Not at all
2
3
4
5
Perfectly
167
Q

Which of the following statements is true about the hepatic arterial supply?

A. Aberrant hepatic arterial anatomy is present in less than 5% of all patients.

B. The cystic artery is usually a branch off the proper hepatic artery.

C. A “replaced” right hepatic artery arises from the superior mesenteric artery.

D. The hepatic artery provides 75% of the blood flow to the liver.

E. The hepatic artery lies dorsal to the portal vein within the hepatic hilum.

A

ANSWER: C

COMMENTS: The hepatic arterial supply is normally derived from the celiac axis by way of the common hepatic artery, which becomes the proper hepatic artery after giving off the gastroduodenal branch and subsequently bifurcates into right and left hepatic branches.

The hepatic artery lies ventral to the portal vein. The middle hepatic artery is usually a branch off the left hepatic artery, and the cystic artery is generally a branch off the right hepatic artery.

There is, however, significant variability in hepatic arterial anatomy in up to 50% of patients. In approximately 15% of individuals, the right hepatic artery arises from the superior mesenteric artery (replaced right hepatic artery) and is found in the right dorsal border of the hepatoduodenal ligament. In roughly 10% of individuals, the left hepatic artery originates from the left gastric artery and is located in the gastrohepatic ligament. These commonly encountered variants can have important surgical implications during upper abdominal operations. The arterial blood supply accounts for only 25% of hepatic blood flow, with the remainder being supplied by the portal vein.

Ref.: 1–3

How well did you know this?
1
Not at all
2
3
4
5
Perfectly
168
Q

Which of the following statements about the anatomy of the hepatic veins is true?

A. The left hepatic vein drains the entire left lobe.

B. Veins from the caudate lobe enter both the left and middle hepatic veins.

C. The middle hepatic vein usually joins the left hepatic vein.

D. There are valves in the hepatic venous system.

E. Hepatic veins have prominent hyperechoic walls on ultrasound imaging.

A

ANSWER:

C

COMMENTS: The hepatic veins begin in the liver lobules as the central veins and coalesce to form the right, left, and middle hepatic veins, which drain into the inferior vena cava and are of considerable surgical importance because they define the three vertical scissurae of the liver.

The right vein, which is generally the largest, drains most of the right lobe. The left vein drains the lateral segment of the left lobe and a portion of the medial segment as well. The middle vein drains the inferoanterior portion of the right lobe and the inferomedial segment of the left lobe. This vein joins the left hepatic vein in 80% of individuals and enters the inferior vena cava directly in the remainder.

There are also smaller veins, particularly those draining the caudate lobe dorsally, that enter directly into the inferior vena cava. The human hepatic venous system has no valves. The portal veins and hepatic veins can readily be differentiated from each other on the basis of their distinctive sonographic features. The portal veins (not the hepatic veins) have prominent hyperechoic walls.

How well did you know this?
1
Not at all
2
3
4
5
Perfectly
169
Q
  1. Which of the following statements is true about the portal vein?

A. It is formed by the junction of the inferior mesenteric vein and splenic vein.

B. It is the most dorsal structure in the hepatoduodenal ligament.

C. It contains the valves of Mirizzi.

D. The right portal vein typically branches later than the left portal vein.

E. It carries deoxygenated blood and provides only 10% of the liver’s oxygenation.

A

ANSWER:

B

COMMENTS: The portal vein is usually formed dorsal to the neck of the pancreas by the junction of the superior mesenteric vein and splenic veins. It ascends posterior to the common bile duct and hepatic artery in the hepatoduodenal ligament. These three structures make up the portal triad. There are no valves in the portal venous system (Pablo Mirizzi described valves in the common hepatic duct that do not exist). The portal vein bifurcates just outside the liver. The right portal vein has anterior and posterior branches that typically diverge only a short distance from the bifurcation and then quickly dive into the liver parenchyma. The left portal vein has a longer transverse portion (pars transversus) and then angulates anteriorly in the umbilical fissure (pars umbilicus), where it gives off medial branches to segment IV and lateral branches to segments II and III. The portal vein provides approximately 75% of the hepatic blood flow, and although the blood is largely deoxygenated, it provides up to 50%–70% of the liver’s oxygenation secondary to the portal system’s large volume flow rate.

How well did you know this?
1
Not at all
2
3
4
5
Perfectly
170
Q

Which of the following characteristics is typically seen on ultrasound imaging of the hepatic portal vein branches?

A. Hyperechoic vessel walls

B. Hepatofugal blood flow

C. Diastolic reversal of blood flow

D. Location between hepatic segments

E. Vertical orientation

A

ANSWER: A

COMMENTS: The portal veins and hepatic veins can readily be differentiated from each other on the basis of their distinctive sonographic features. The portal vein and its branches have prominent hyperechoic walls. This appearance has been attributed to the accompanying intrahepatic branches of the hepatic artery and bile duct, which are not generally seen individually on external ultrasound imaging. In contrast, the hepatic veins appear to be essentially “wall less.” They are anechoic or hypoechoic tubular structures that are vertically oriented and increase in caliber as they course toward the inferior vena cava. The portal veins are more transversely oriented and of larger caliber centrally. The portal vein branches are located within the anatomic liver segments, and the hepatic veins are found between the segments. Doppler ultrasound permits characterization of flow patterns in the hepatic vessels. Under normal circumstances, portal vein flow is toward the liver (hepatopedal). Flow in the portal vein is usually of fairly low velocity, with minor undulations and continued forward flow during diastole. Flow in the hepatic veins is hepatofugal and varies according to the cardiorespiratory cycle. The portal veins are horizontally oriented, whereas the hepatic veins are vertically oriented.

How well did you know this?
1
Not at all
2
3
4
5
Perfectly
171
Q

Which of the following is true regarding the hepatic functional unit?

A. The center of the hepatic lobule is the hepatic venule.

B. Blood flows from the hepatic vein to the portal triad.

C. Zone I is the most susceptible to hypoxic injury.

D. Hepatocytes in zone I have the lowest oxygen tension.

E. Bile flows toward the centrilobular hepatic venule.

A

ANSWER: A

COMMENTS: The functional histologic unit of the liver is the acinus. At the center of the acinus is the portal triad, which consists of a terminal branch of the portal vein (portal venule) along with a hepatic arteriole and bile ductule. Blood from the terminal portal venule goes into the hepatic sinusoids, around which hepatocytes are located. Eventually, blood returns to the central vein leading to the terminal hepatic venules at the periphery of the acinar unit. The hepatocytes of the acinus are divided into three zones, with zone I being closest to the afferent portal venule and zone III being nearest the efferent central hepatic venule. Zone II is between these two points. Within the acinus, there is a gradient of solute concentration and oxygen tension that is greatest near the portal venules at the center of the acinus. The hepatocytes in zone I are therefore exposed to more oxygen and are less subject to hypoxia compared with the hepatocytes near the periphery of the acinus (zone III). This explains the histologic pattern of centrilobular necrosis that occurs following ischemia. The hepatic venule is at the center of the histologic hepatic lobule. Each hepatic lobule is thus surrounded by several peripheral acini. Bile is formed within the hepatocytes and empties into terminal canaliculi, which coalesce into bile ducts. The bile then flows toward the portal triad.

How well did you know this?
1
Not at all
2
3
4
5
Perfectly
172
Q

What is the pathologic feature of cirrhosis?

A. Apoptosis of hepatocytes, involving lobules and portal
tracts

B. Fat accumulation in hepatocytes

C. Brown pigment in hepatocytes

D. Periductal fibrosis with an onion-skin appearance

E. Dense matrix material deposition in the perisinusoidal space

A

ANSWER: E

COMMENTS: Cirrhosis may be due to many processes. It is defined as end-stage liver damage with hepatocyte death and dis- ruption of hepatic parenchyma by diffuse fibrosis (dense matrix material deposition) and abnormal nodular architecture. Pathologic characteristics include fibrous septa around regenerative nodules of hepatocytes.

The bands of fibrosis originate from stellate cells located beneath endothelial cells that line the sinusoids, in the space of Disse (perisinusoidal space).

Clinical manifestations include increased hepatic pressure, portal venous congestion and portal hypertension, ascites, coagulopathy, steroid hormone imbalances, encephalopathy, hepatorenal syndrome, and hepatopulmonary syndrome.

Despite historical schools of thought, it is now known that even in late-stage disease, some regression of cirrhosis is possible.

Apoptosis of hepatocytes involving lobules and portal tracts is seen in hepatitis.

Fat accumulation in hepatocytes is seen in the alcoholic fatty liver disease.

Brown pigment in hepatocytes, or hemosiderosis, is due to iron deposition in hepatocytes in hemochromatosis.

Periductal fibrosis with an onion-skin appearance is seen in primary biliary cirrhosis.

How well did you know this?
1
Not at all
2
3
4
5
Perfectly
173
Q

Which of the following statements related to liver embryology is true?

A. The liver is derived from the hindgut.

B. Two umbilical veins carry placental blood to the liver.

C. One umbilical artery carries placental blood to the placenta.

D. The ductus venosus shunts blood from the portal vein to the systemic circulation.

E. The falciform ligament is the obliterated ductus arteriosus.

A

ANSWER: D

COMMENTS: The liver, gallbladder, and biliary tree are derived from a ventral diverticulum of the junction of the foregut and midgut.

In fetal circulation, one umbilical vein carries oxygenated blood from the placenta to the ductus venosus, which shunts blood from the portal vein to the inferior vena cava. The result is that the maternal blood, which has already undergone hepatic metabolism in the mother, largely bypasses the fetal liver. Two umbilical arteries carry deoxygenated blood back to the placenta. In the transition to extrauterine circulation, the umbilical vein degenerates as flow ceases, and stasis and thrombosis cause ductus venosus closure.

In the adult, the falciform ligament carries the umbilical vein remnant from the umbilicus. The ligamentum teres extends from the falciform ligament and carries the obliterated umbilical vein to the undersurface of the liver. The ligamentum venosum derives from the ductus venosus.

How well did you know this?
1
Not at all
2
3
4
5
Perfectly
174
Q

During fasting, the liver provides energy substrates by all but which of the following mechanisms?

A. Glycogenolysis
B. Glycolysis
C. Gluconeogenesis from alanine
D. Gluconeogenesis from lactate
E. Formation of ketone bodies from fatty acids
A

ANSWER: B

COMMENTS: The liver plays a pivotal role in energy metabolism. In the fed state, glucose is converted to glycogen for storage. The liver itself obtains its energy primarily from ketoacids rather than glucose, although it can use glycolysis during periods of glucose excess (fed state).

During fasting, the liver provides glucose by the breakdown of the stored glycogen (glycogenolysis). Glucose is a critical energy source for red blood cells, the central nervous system, and the kidneys. Because glycogen stores are depleted after about 48 h, the liver generates glucose from other sources.

Alanine, other amino acids, lactate, and glycerol can serve as carbon sources for gluconeogenesis. Lipolysis occurs during prolonged fasting, and the fatty acids released from adipose stores are oxidized in hepatocytes to form ketone bodies.

Ketone bodies are an important alternative fuel source for brain and muscle.

How well did you know this?
1
Not at all
2
3
4
5
Perfectly
175
Q

The cytochrome P-450 system transforms compounds by all except which of the following mechanisms?

A. Oxidation
B. Hydrolysis
C. Conjugation
D. Reduction
E. Hydrogenation
A

ANSWER: C

COMMENTS: The liver is responsible for the biotransformation of many endogenous and exogenous substances. For the most part, this process detoxifies potentially injurious substances and facilitates their elimination.

In some instances, however, hepatic bio- transformation produces more toxic metabolites.

There are two general mechanisms by which the liver accomplishes biotransformation: oxidation, reduction, and hydrolysis (phase I reactions) and conjugation (phase II reactions).

The cytochrome P-450 enzyme system catalyzes phase I reactions. The second mechanism involves an array of enzymes that conjugate substances with other endogenous molecules.

These reactions are referred to as phase II reactions, and their purpose is to convert hydrophobic compounds to hydrophilic ones that are water soluble and can thus be eliminated in bile or urine.

The liver is also the principal site of conversion of ammonia to urea via the urea cycle, which is a separate process.

How well did you know this?
1
Not at all
2
3
4
5
Perfectly
176
Q

A 63-year-old woman with unresectable cholangiocarcinoma undergoing palliative therapy presents with fever, right upper quadrant pain, jaundice, and cough for 1 day. Temperature was 101.7°F, pulse 93 beats/min, blood pressure 112/65 mmHg, respiratory rate 20 breaths/min, and saturation 98% on room air. Physical examination shows a soft and nondis- tended abdomen with tenderness in the right upper quadrant and hepatomegaly. Lab results include a white blood cell count of 14,000/mm3 and alkaline phosphatase level of 215 IU/L. Chest x-ray shows an elevated right hemidiaphragm and right lower lobe atelectasis. Ultrasound is performed showing a round hypoechoic lesion within the liver, with well-defined borders and several internal echoes, mildly dilated intrahepatic bile ducts, and no cholelithiasis or pericholecystic fluid. In addition to further imaging, what is the most appropriate management?

A. Antibiotics for 10 to 14 days
B. Percutaneous drainage and intravenous (IV) antibiotics C. Endoscopic retrograde cholangiopancreatography
D. Laparoscopic cholecystectomy
E. Emergent exploratory laparotomy

A

ANSWER: B

COMMENTS: Liver abscesses classically present as fever, right upper quadrant pain, nausea, pleuritic chest pain, cough or dyspnea, and sometimes with sepsis. They may be pyogenic (bacterial), amebic, or fungal. Pyogenic abscess is usually polymicrobial, although common microbes include Escherichia coli or other gram- negative bacteria, Streptococcus species, and anaerobes such as Bacteroides.

Patient risk factors include underlying biliary cancer, recent ablative liver therapy, or liver transplantation. Today, the most frequent source of pyogenic abscess is a contiguous infection in the biliary tract, such as cholangitis. Other sources include infectious foci within the portal venous drainage system, direct extension from perihepatic sites, and hematogenous spread. The right lobe is the most commonly involved, which has been attributed to a streaming effect on the portal vein. Approximately 20% of pyogenic abscesses are cryptogenic.

The diagnosis is based on the clinical findings and hepatic imaging and may be confirmed by fine-needle aspiration. While ultrasound is a good modality to visualize the liver abscess, gallbladder, and intrahepatic bile ducts, computed tomography (CT) is more sensitive and further allows for evaluation of another underlying cause such as a biliary tree cancer.

Treatment of pyogenic abscess requires eradication of both the abscess and the source. Treatment of the abscess usually requires drainage by operative or percutaneous approaches. Antibiotic therapy alone may suffice for the treatment of multiple small abscesses.

How well did you know this?
1
Not at all
2
3
4
5
Perfectly
177
Q

A 30-year-old man visiting from Mexico comes to the emergency department with a history of 2 weeks of right upper quadrant pain and tenderness, fevers, chills, and diarrhea. He is febrile to 102.9°F. His heart rate and blood pressure are 120 beats/min and 100/75 mmHg, respectively. Laboratory results include a white blood cell count of 16,000/mm3, aspartate aminotransferase (AST) level of 50 IU/L, and alanine aminotransferase (ALT) level of 93 IU/L. Ultrasound of the abdomen shows a 4 × 7-cm2 round, hypoechoic, nonhomogeneous lesion abutting the liver capsule without rim echoes. Subsequent CT also demonstrates a non–rim-enhancing hypoechoic lesion with a smaller adjacent lesion measuring 2 × 2 cm2. Which of the following is the most appropriate course of action?

A. Observation
B. Open surgical drainage
C. Broad-spectrum antibiotics and percutaneous drainage
D. Serologic testing for Entamoeba histolytica and oral metronidazole
E. Therapeutic fine-needle aspiration

A

ANSWER: D

COMMENTS: Amebic abscesses are caused by the protozoan E. histolytica, which is spread through the fecal–oral route. Once ingested, the cysts pass into the intestines, where the trophozoite is released and transmitted to the colon. These trophozoites can then invade the colonic mucosa and subsequently reach the liver via the portal vein. In the liver, these trophozoites produce a liquefaction necrosis responsible for the classic “anchovy paste” appearance. Protozoa are not usually isolated from the abscess because they are located in the peripheral rim of tissue.

Diagnosis requires hepatic imaging (usually ultrasound or CT) and serologic testing for the presence of E. histolytica antibodies as well as a thorough history and physical examination. The patient in this question is a young man from an endemic region who has signs and symptoms similar to those of a pyogenic liver abscess; however, his classic history and the lack of rim enhancement on imaging suggest the diagnosis of amebic abscess rather than a pyogenic abscess.

Hepatic amebiasis is treated primarily by the administration of amebicidal drugs, with metronidazole being the drug of choice. Percutaneous aspiration may be indicated if the patient does not respond to medical management or the diagnosis is in question. Percutaneous or operative drainage is also indicated in the presence of secondary bacterial infection, which occurs in about 10% of amebic abscesses.

How well did you know this?
1
Not at all
2
3
4
5
Perfectly
178
Q

A 50-year-old woman complains of a 4-month history of right-sided abdominal pain and nausea. Her vital signs are stable, and she is afebrile. Her physical examination is unremarkable except for hepatomegaly. Ultrasound of the abdomen shows an 8-cm well-circumscribed cyst with a rosette appearance. What is the preferred treatment of this patient?

A. Pericystectomy
B. Percutaneous catheter drainage 
C. Transperitoneal surgical drainage 
D. Metronidazole
E. Albendazole
A

ANSWER: A

COMMENTS: The helminth Echinococcus granulosus is respon- sible for most hydatid diseases of the liver. It is usually a unilocular process involving the right lobe, although it may be manifested as multiple cysts. Complications include intrabiliary, intraperitoneal, or intrapleural rupture; secondary infection; anaphylaxis; and mass replacement of the liver.

These lesions often have a calcified wall and can be diagnosed serologically by indirect hemagglutination tests, complement fixation tests, serum immunoelectrophoresis, and, formerly, the Casoni skin test.

CT and ultrasound may demonstrate characteristic daughter cysts (hydatid sand) or grand- daughter cysts (rosette appearance) within the cyst.

Treatment is primarily surgical. Percutaneous aspiration or drainage is generally contraindicated because of the risk for intraperitoneal dissemination; however, since the advent of chemotherapeutic agents such as albendazole, some clinicians have proposed percutaneous drainage.

The principles of surgical therapy are to avoid spillage and remove the entire germinal layer. The cyst consists of an inner germinal layer (endocyst) and an outer fibrous membrane layer (pericyst).

Resection is usually accomplished by pericystectomy. Anatomic hepatic resection is not generally required but may be used.

Surgery, in addition to preoperative and postoperative benzimidazole compounds, has been shown to be very effective.

Metronidazole is used for the treatment of amebic liver abscesses. Because 20% of echinococcal cysts exhibit biliary communication, assessment by preoperative ERCP or intraoperative cholangiography is important in any patient with jaundice, cholangitis, elevated liver enzyme levels, or bile noted during resection.

Scolicidal agents should be used with caution because of the risk of sclerosing the bile ducts in the event that the agent finds its way into the ductal system.

How well did you know this?
1
Not at all
2
3
4
5
Perfectly
179
Q

A 28-year-old asymptomatic white woman is incidentally found to have a 3.5-cm hypervascular lesion with a central scar in the right lobe of her liver. On delayed images, there is increased uptake of contrast material in the scar in compari- son with the surrounding liver parenchyma. She is otherwise healthy and takes no medications. Liver enzyme and α-fetoprotein levels are within normal limits. Which of the following is the most appropriate management of this patient?

A. Open liver resection
B. Open surgical biopsy
C. Observation
D. Chemoembolization
E. Hepatic artery embolization
A

ANSWER: C

COMMENTS: This patient has focal nodular hyperplasia (FNH), which is often found incidentally on imaging or during laparotomy.

FNH is a benign liver tumor that predominantly occurs in women in the third to fifth decades of life. It is similar to hepatic adenoma (HA) but with important differentiating clinical and histologic features and therapeutic implications.

Both occur most commonly in women of childbearing age; however, HA is associated with the use of oral con- traceptives and anabolic steroids and is also seen in certain glycogen storage diseases. HA is usually symptomatic (80% of cases) and is associated with rupture and bleeding in a substantial proportion of patients, whereas FNH is usually asymptomatic and found incidentally.

Furthermore, HA has the potential for malignant transformation, whereas the risk for malignancy in FNH is unlikely but uncertain.

Histologically, HA consists of hepatocytes without bile ducts or Kupffer cells. FNH contains Kupffer cells along with a central stellate scar surrounded by fibrous tissue. Scanning for Kupffer cell activity with technetium-99m (99mTc)-labeled sulfur colloid is thus useful in differentiating the lesions. Because of the asymptomatic nature of this patient, small size of the lesion, and negligible risk for malignant transformation, observation is appropriate.

Surgical resection is reserved for symptomatic patients or when the diagnosis is uncertain.

How well did you know this?
1
Not at all
2
3
4
5
Perfectly
180
Q

Right upper quadrant abdominal pain develops in a 25-year- old woman taking oral contraceptives. CT demonstrates a hypodense, 6-cm mass in the right lobe of the liver. A 99mTc-labeled scan reveals a defect in the area of the mass. Angiography reveals a hypervascular tumor with a peripheral blood supply. Which of the following is the appropriate management?

A. Discontinuation of oral contraceptives and observation
with serial CT
B. Percutaneous needle biopsy 
C. Hepatic resection
D. Arterial embolization
E. Radiation therapy
A

ANSWER: C

COMMENTS: The imaging characteristics described are typical of HA. Because HA does not contain Kupffer cells, it does not take up radioisotope. This point may be useful for differentiating HA from FNH but not necessarily from other mass lesions of the liver.

Percutaneous biopsy of suspected HA is not advisable because of the risk for hemorrhage. HAs associated with oral contraceptives tend to be larger and have a higher risk for bleeding.

Regression does not reliably occur with cessation of oral contraceptives. However, for lesions smaller than 4 cm, a trial of cessation of contraceptives or steroids with observation may be attempted.

Resection is indicated for most suspected HAs, particularly for symptomatic lesions, for patients not taking oral contraceptives, and if the diagnosis is uncertain.

Embolization may be useful for treating hemorrhage in a patient whose HA is inoperable. Radiation has no role in the management of HA.

How well did you know this?
1
Not at all
2
3
4
5
Perfectly
181
Q

A 75-year-old woman with recurrent breast cancer undergoes CT of the chest, abdomen, and pelvis for metastatic workup. She is found to have a 7-cm liver mass. Follow up triple- phase CT shows a hypervascular lesion with peripheral to central enhancement. It is thought to be a hemangioma. She denies abdominal pain or any episodes of jaundice. What would be a reasonable indication for operative intervention?

A. Pain, shortness of breath, early satiety
B. Hemorrhage
C. High-output cardiac failure
D. Disseminated intravascular coagulation (DIC)
E. All of the above

A

ANSWER: E

COMMENTS: Hepatic hemangiomas are the most common benign hepatic tumor. They have no risk of malignant degeneration and are often found incidentally. They typically appear as hyper- vascular lesions with initial peripheral enhancement followed by late central enhancement, which follows the direction of blood flow into the lesion. Biopsy is not necessary and furthermore carries the risk of hemorrhage.

Asymptomatic uncomplicated hemangiomas are simply observed. Indications for surgical resection include symptoms from mass effect, consumptive coagulopathy manifesting as DIC (seen in Kasabach-Merritt syndrome), high-output cardiac failure from arteriovenous shunting, or rupture and hemorrhage. Enucleation can generally be performed, without formal liver resection being necessary.

How well did you know this?
1
Not at all
2
3
4
5
Perfectly
182
Q

A 50-year-old woman with hypertension presents to her primary care physician (PCP) with right upper quadrant pain and jaundice. There is a palpable soft mass along her liver edge. Workup is negative for viral hepatitis, and liver enzymes are mildly elevated. Right upper quadrant ultra- sound shows a 4-cm cystic structure that is extrahepatic and separate from the gallbladder. ERCP is performed and demonstrates a biliary diverticulum obstructing the common bile duct. What is the most appropriate next step in management?

A. Observation
B. Percutaneous drainage 
C. Cholecystectomy
D. Resection
E. Hepaticojejunostomy
A

ANSWER: D

COMMENTS: When a cyst is an incidental finding with no symptoms and the diagnosis is secure, no further intervention is indi- cated. In larger cysts, there is a risk of biliary obstruction leading to pancreatitis, cholangitis, and obstructive jaundice, and symptom- atic patients should undergo resection.

Percutaneous drainage or injection of alcohol or other sclerosing agents does not suffice and is not recommended. If a cyst is found to communicate with the bile ducts, either excision or Roux-en-Y cystojejunostomy may be performed.

Choledochal cysts are classified into five types. Type I cysts are saccular dilatations of the entire common bile duct, and treat- ment is Roux-en-Y hepaticojejunostomy. Type II cysts, like in this case, are true diverticula of the common bile duct, and treatment is simple excision. Type III cysts, also called choledochodeles, are local dilations of the distal common bile duct extending into the duodenal wall, and treatment is marsupialization or excision.

Type IV cysts are multiple and involve both intra- and extra- hepatic ducts. Type V cysts are intrahepatic cysts and the rarest type. They are sometimes associated with congenital hepatic fibro- sis and medullary sponge kidney.

Treatment of type IV and V cysts depends on the extent of liver tissue involved. Excision, often requiring Roux-en-Y jejunostomy, is preferred over drainage. However, resection may not be an option at all, such as in the case of multifocal cysts involving both lobes with background hepatic fibrosis.

How well did you know this?
1
Not at all
2
3
4
5
Perfectly
183
Q

Which of the following statements is true regarding intrahepatic cholangiocarcinoma?

A. Survival following resection is generally lower than that for distal bile duct cancer.
B. Resection is contraindicated unless histologically negative margins can be obtained.
C. The best survival is achieved with liver transplantation.
D. Adjuvant chemotherapy improves survival following resection.
E. None of the above.

A

ANSWER: A

COMMENTS: Cholangiocarcinoma arises from the bile duct epithelium and can occur anywhere along the biliary tract. It constitutes 5%–20% of primary liver cancers.

Tumors arising from the extrahepatic bile ducts differ from those located intra- hepatically in terms of their clinical findings, therapy, and prog- nosis. Tumors of the extrahepatic bile ducts are typically manifested as biliary obstruction.

Intrahepatic tumors appear similar to hepatocellular cancer, a liver mass with absent or vague symptoms such as pain, weight loss, nausea, and anorexia. The treatment of choice is surgical excision, which is associated with a 15%–20% 5-year survival rate. The prognosis is best for tumors of the distal bile ducts that can be resected by a pancre- aticoduodenectomy.

Tumors involving the bifurcation of the bile duct (Klatskin tumor) are less often resectable.

Tumor size and the presence of satellite nodules are correlated with outcome.

Histologically negative margins are always desirable, but prolonged survival can be attained even with microscopically involved margins.

If the tumor cannot be resected, improved survival has been noted with bypass or stenting procedures. Liver transplantation for cholangiocarcinoma has been associated with frequent recurrence and has not generally been encouraging.

Adjuvant chemotherapy has not typically been useful for bile duct cancer.

How well did you know this?
1
Not at all
2
3
4
5
Perfectly
184
Q

A 75-year-old man with hepatitis C cirrhosis presents for an annual checkup. He denies any new complaints, including jaundice, abdominal pain, ascites, gastrointestinal bleeding, or encephalopathy. Temperature is 99.1°F, pulse 85 beats/ min, blood pressure 109/78 mmHg, respiratory rate 18 breaths/min, and saturation 98% on room air. What studies should be ordered?

A. None
B. Ultrasound
C. α-Fetoprotein (AFP) level
D. Esophagogastroduodenoscopy (EGD) 
E. CT scan
A

ANSWER: B

COMMENTS: Hepatocellular carcinoma (HCC) is among the most common cancers worldwide. Anyone with cirrhosis is at risk, and some etiologies of cirrhosis are independent risk factors separate from cirrhosis. For example, in hepatitis B infection, HCC can develop without underlying cirrhosis. Other populations at risk include patients with hepatitis C, alcoholic liver disease, hemochromatosis, alpha-1-antitrypsin deficiency, primary scle- rosing cholangitis, aflatoxins, HA, anabolic steroid use, and pesticides.

HCC screening is therefore recommended in all individuals with cirrhosis. The recommended screening method is ultrasonography every 6 months.

Ultrasonography is 60%–90% sensitive, depending on the size of the lesions. Adding AFP testing is no longer recommended due to its cost and false-positive rates.

How well did you know this?
1
Not at all
2
3
4
5
Perfectly
185
Q

A 69-year-old woman with Child’s C cirrhosis has an elevated AFP and a new liver mass on ultrasound. Triple- phase contrast CT confirms a 3.5 ×4-cm2 mass with arterial enhancement and venous washout. She is other- wise healthy with no cardiopulmonary disease. What is the next most appropriate step in workup and treatment?

A. Interventional radiology (IR) biopsy and pathology 
B. Neoadjuvant chemotherapy
C. Liver resection
D. Liver transplant
E. Assess portal vein pressures
A

ANSWER: D

COMMENTS: In a patient with positive HCC screening, diagno- sis is generally made by imaging. Triple-phase magnetic resonance imaging (MRI) is more sensitive and specific than triple-phase CT, but the latter is more often used.

The characteristic description of HCC is a lesion with arterial enhancement and venous washout. Biopsy is usually not necessary, except for unusual cases in which the clinical diagnosis is still in doubt.

Management of HCC is resection, and the most important factor in assessing candidacy for liver resection is a hepatic reserve.

In general, a patient with Child’s A cirrhosis can tolerate up to 50% liver resection, and Child’s B up to 25% liver resection. Liver resection is contraindicated in Child’s C cirrhosis, and trans- plant has better survival than resection in these patients. Portal vein hypertension must also be taken into account, as liver resec- tion may not be tolerated if portal vein pressures are above 10 mmHg. The Barcelona Clinic Liver Cancer (BCLC) staging system is often used to guide HCC treatment based on patient and tumor factors.

Liver transplantation is indicated in Child’s C cirrhosis as long as the patient is thought to be able to tolerate this.

The Milan criteria have been proposed to define HCC tumors in Child’s C cirrhosis that would benefit from transplantation.

These are tumors in which there is no extrahepatic disease, no macrovascular invasion, and either one tumor up to 5.0 cm in size, or up to three tumors and each up to 3.0 cm in size.

Patients with a new HCC diagnosis are awarded additional model for end-stage liver disease (MELD) exception points to minimize waiting list dropout due to disease progression.

How well did you know this?
1
Not at all
2
3
4
5
Perfectly
186
Q

Appropriate use of locoregional therapies in patients with
HCC includes all of the following except:

A. An attempt at down-staging tumors initially regarded as
unresectable
B. To maintain a patient within transplant listing criteria
C. Treatment of multiple small tumors that are multifocal
D. Treatment of tumors that are in locations of anatomic constraint
E. An alternative to resection in patients with good hepatic reserve

A

ANSWER: E

COMMENTS: Locoregional therapies, such as radiofrequency ablation, transarterial chemoembolization (TACE), and radiation, have been used with the aim of down-staging tumors initially regarded as unresectable. The resectability of a tumor is determined by a number of factors including hepatic reserve, patient functional status, and tumor size. Resection is still preferred over locoregional therapy and should be performed in patients without contraindication.

Additionally, locoregional therapies have been used in patients with cirrhosis who do not qualify for liver transplantation based on the Milan criteria, in efforts to bring or maintain them within transplant listing criteria. Other uses include treatment of multiple bilobar tumors unresectable due to distribution and tumors in loca- tions that are difficult to access surgically. Finally, another role for these modalities is in the treatment of liver metastases from colorectal cancer.

How well did you know this?
1
Not at all
2
3
4
5
Perfectly
187
Q

A 55-year-old woman with a history of colorectal cancer 10 years ago, treated with right hemicolectomy and chemotherapy and with no evidence of disease since then, presents with vague abdominal pain. Temperature is 98.7°F, pulse 67 beats/ min, blood pressure 132/78 mmHg, and saturation 99% on room air. Labs are remarkable for AST 67 IU/L, ALT 83 IU/L, and carcinoembryonic antigen (CEA)-27 ng/mL. Triple-phase contrast CT shows a new peripheral liver mass that slightly enhances on arterial phase. She has hypertension that is well controlled with medications. What are the most likely diagnosis and most appropriate management of her liver lesion?

A. FNH; observation
B. Primary liver cancer; resection
C. Primary liver cancer; chemotherapy
D. Metastatic colorectal cancer; resection
E. Metastatic colorectal cancer; palliation

A

ANSWER: D

COMMENTS: Metastatic liver tumors are much more common than primary liver tumors. The liver is the most common site of colorectal cancer metastasis, followed by the lung. Other common sources of liver metastases include pancreas, lung, and breast carcinomas. A new hepatic lesion with a known history of one of these carcinomas is generally sufficient for diagnosis, without a need for biopsy. CEA is usually elevated as well if colorectal cancer is the primary.

Characteristics that further support metastasis over primary liver cancer include a history of primary cancer, peripheral residing lesions, multiple lesions, and mass hypovascularity (hence only slightly enhancing compared with adjacent liver parenchyma).

Resection of hepatic metastases from colorectal cancer pro- vides a clear survival advantage over any other treatment and should be performed whenever possible. Over the past two decades, improvements in intraoperative techniques have afforded improved outcomes in liver surgery. Experienced centers demon- strate 5-year survival rates of 25%–40% with mortality rates of less than 5%.

Candidacy for resection is similar to that in HCC and, in other words, depends highly on hepatic reserve. In general, an isolated liver and/or lung metastasis in a patient with colorectal carcinoma should be resected unless contraindicated.

This patient should also undergo surveillance colonoscopy to evaluate for local recurrence.

How well did you know this?
1
Not at all
2
3
4
5
Perfectly
188
Q

Which of the following is the most accurate method for
identifying hepatic metastases?

A. Transabdominal ultrasound 
B. CT
C. Laparoscopy
D. Intraoperative palpation
E. Intraoperative ultrasound imaging
A

ANSWER: E

COMMENTS: Transabdominal ultrasound is as accurate as CT for detecting liver tumors that are 2 cm in size or larger. For smaller lesions, CT is more accurate, although it can miss the smallest lesions (<1 cm). Laparoscopy is useful for identifying small metastases on the liver or peritoneal surfaces that escape discovery by noninvasive preoperative imaging modalities.

Laparoscopy has been incorporated into the staging workup of a variety of intraabdominal malignancies, including those of the liver. However, one of its limitations is its ability to assess the interior structure of solid organs. It is now well recognized that intraoperative ultrasound is the most accurate method for detecting and assessing hepatic tumors. Not only does intraoperative ultrasound discover more lesions than any other modality (including palpation), but it also clearly demonstrates the anatomic relationship of tumors to important vascular structures, which is a critical determinant of resect- ability and the extent of resection necessary.

Intraoperative ultrasound can be performed with handheld or laparoscopic trans- ducers. Experience with intraoperative ultrasound for liver tumors has shown that the sonographic findings affect the surgical manage- ment of one-third to one-half of patients. Intraoperative ultrasound imaging has become an indispensable component of hepatic surgery.

How well did you know this?
1
Not at all
2
3
4
5
Perfectly
189
Q

A 56-year-old woman with hepatitis C cirrhosis has had a worsening mental status that has now progressed to hepatic coma. Which of the following can be used for initial treatment of a patient in a hepatic coma?

A. Reduction of dietary protein to 50 g/day or less 
B. Control of active bleeding
C. Lactulose
D. Neomycin
E. All of the above
A

ANSWER: E

COMMENTS: Treatment of hepatic encephalopathy and coma is aimed at limiting the nitrogen that the liver must metabolize by eliminating nitrogenous material from the gastrointestinal tract and by inhibiting its absorption. At the same time, precipitating causes are sought and treated.

Nutritional support is important and can be initiated with standard amino acids and restriction of dietary protein.

Cessation of any gastrointestinal bleeding from varices is an important step in reducing the conversion of intraluminal blood to ammonia.

Lactulose acts as a cathartic and also inhibits the absorption of ammonia by acidifying the colon.

Nonabsorbable antibiotics, such as neomycin and kanamycin, reduce colonic flora and the production of ammonia.

Systemic antibiotics may be useful for treating specific infections that precipitate encephalopathy but are not indicated empirically. Because the colon is the major site of ammonia absorption, colon resection or exclusion has been suggested to improve encephalopathy but is not a widely used therapeutic measure.

How well did you know this?
1
Not at all
2
3
4
5
Perfectly
190
Q

A 43-year-old man with alcoholic cirrhosis has had increas- ing abdominal distention over the last month. His vital signs are stable, and he is afebrile. Physical examination reveals a distended abdomen with a fluid wave. The initial management of the patient’s ascites should include all of the following except:

A. Transjugular intrahepatic portosystemic shunt (TIPS) 
B. Sodium restriction
C. Diuretic administration
D. Fluid restriction
E. Diagnostic paracentesis
A

ANSWER: A

COMMENTS: Ascites is the most common major complication of hepatic cirrhosis. It is associated with a 2-year survival rate of 50%, and its onset in a cirrhotic patient should prompt an evaluation for liver transplantation.

Treatment of ascites depends on its cause, and therefore diagnostic paracentesis is required after a history and physical examination.

Abdominal ultrasound can confirm the presence of ascites if it is not certain by examination.

The serum-ascites albumin gradient is useful diagnostically. A high gradient (1.1 g/dL) indicates portal hypertension and suggests that the patient will be responsive to medical management consisting of sodium restriction (2000 mg/day) and oral diuretics.

Usually, both spironolactone and furosemide are administered to produce fluid loss and natriuresis. Spironolactone alone may cause hyper- kalemia, and furosemide alone is less effective.

Medical therapy controls ascites in about 90% of patients. When the ascites is refractory, serial therapeutic paracenteses (with or without the administration of albumin or other plasma volume expanders) are indicated. Liver transplantation is the ultimate treatment.

A peritoneovenous shunt is an option for patients with refractory ascites who are not transplantation candidates or who cannot undergo repeated paracenteses.

These shunts are fraught with potential complications, however, and do not prolong survival in comparison with medical management.

TIPSs or operative side-to-side–type portosystemic shunts may control the ascites in select patients.

How well did you know this?
1
Not at all
2
3
4
5
Perfectly
191
Q

What is the preferred site for needle entry in paracentesis?

A. 3 cm medial and 3 cm superior to the anterior superior iliac supine (ASIS) on either side

B. 3 cm medial and 3 cm superior to the ASIS in the right lower quadrant (RLQ)

C. 3 cm medial and 3 cm superior to the ASIS in the left lower quadrant (LLQ)

D. 3 cm medial to and at the level of the ASIS on either side E. Midline at linea alba, inferior to arcuate line

A

ANSWER: C

COMMENTS: The mechanism of ascites in cirrhosis is thought to be a result of the local release of vasodilators, such as nitric oxide. This causes splanchnic arterial vasodilation, leading to a reduction in effective arterial blood volume. The impending result is systemic vasoconstriction and renal sodium-retention, leading to total body fluid retention.

The serum-ascites albumin gradient (SAAG) is used to help determine the etiology of ascites. SAAG-serum albumin – ascitic albumin. SAAG ≥ 1.1 is consistent with portal hypertension, such as from cirrhosis, Budd-Chiari syndrome, portal vein thrombosis, congestive heart failure, liver lesion, and alcoholic hepatitis. SAAG < 1.1 points to other nonhepatic etiologies, such as peri- toneal carcinomatosis, nephrotic syndrome, and pancreatitis. Diagnostic paracentesis should be performed on all patients with new-onset ascites, as the etiology of the ascites will guide treatment.

The management of ascites due to portal hypertension includes dietary sodium restriction, fluid restriction, and diuretics. For ascites refractory to medical therapy, treatment options include large- volume paracentesis, TIPS, and peritoneovenous shunt. Liver trans- plantation is the definitive treatment, and ascites in a patient with cirrhosis should prompt an evaluation for liver transplantation.

In paracentesis, the generally accepted site for needle entry is 3 cm medial and 3 cm superior to the ASIS in the LLQ. Although controversial, a common practice is to follow paracentesis with albumin 1-g infusion for every 100 cc removed, in an attempt to maintain plasma oncotic pressure.

How well did you know this?
1
Not at all
2
3
4
5
Perfectly
192
Q

A 43-year-old woman with primary biliary cirrhosis compli- cated by massive ascites, hepatic encephalopathy, and multiple episodes of upper gastrointestinal bleed requiring endoscopic management presents with hematemesis. Temperature is 98.7°F, pulse 109 beats/min, blood pressure 103/77 mmHg, respiratory rate 20 breaths/min, and saturation 97% on 2 L of nasal cannula oxygen. After acute resuscitation with intravenous fluids and blood products, EGD is per- formed showing multiple bleeding esophageal varices, which undergo endoscopic band ligation. When the patient is stabilized, TIPS is considered. Further workup including chest radiography shows a large right pleural effusion. What is the most important relative contraindication for portovenous shunting?

A. Hepatic encephalopathy
B. Concern for continued active hemorrhage 
C. Massive ascites
D. Pleural effusion
E. No contraindications to TIPS
A

ANSWER: A

COMMENTS: Portal hypertension is responsible for the majority of the morbidity and mortality associated with cirrhosis, such as variceal bleeding, refractory ascites, and hepatic hydrothorax. First- line therapy for patients with primary variceal bleeding is endo- scopic therapy with variceal band ligation or sclerotherapy. However, there is a high risk for rebleeding not amenable to endo- scopic techniques (refractory bleeding) or continuation of bleeding (recurrent bleeding).

This patient has failed medical and endoscopic management of variceal bleeding. TIPS is very effective at reducing or normal- izing portal pressure, the underlying cause of variceal bleeding. Other indications for TIPS include refractory ascites, hepatic hydrothorax, and Budd-Chiari syndrome refractory to anticoagulation.

The mechanism is via diverting blood into systemic circulation and decompressing the portal system. Because hepatic metabolism is bypassed, hyperammonemia may occur manifesting clinically as worsening hepatic encephalopathy.

Hepatic encephalopathy is therefore a relative contraindication to TIPS.

How well did you know this?
1
Not at all
2
3
4
5
Perfectly
193
Q

A 57-year-old man with alcoholic cirrhosis presents with abdominal pain and a large umbilical hernia. Temperature is 97.8°F, pulse 78 beats/min, blood pressure 105/63 mmHg, respiratory rate 16 breaths/min, and saturation 98% on room air. Exam shows a large, dull to percussion abdomen with flank fullness, a fluid shift, and a reducible umbilical hernia. What is the most appropriate management?

A. Observation
B. Diuresis and paracentesis 
C. Elective surgical repair 
D. Emergent surgical repair 
E. Liver transplantation
A

ANSWER: C

COMMENTS: Umbilical hernias are common in patients with ascites, occurring in up to 20% of these patients. They develop as a result of increased intraabdominal pressure, muscle wasting, fascial thinning, and nutritional deficits. Ascites management should be optimized prior to repair in order to decrease intraab- dominal pressure and minimize recurrence. Ascites management should be optimized prior to repair to minimize recurrence.

Asymptomatic hernias should be observed. Indications for repair include symptoms, leakage of ascitic fluid, incarceration, and strangulation. Spontaneous rupture and massive leakage of ascitic fluid is a surgical emergency due to the risk of peritonitis and death. Unfortunately, the recurrence rate following repair in patients with ascites is as high as 73%.

194
Q

Which of the following statements is true regarding spontaneous bacterial peritonitis (SBP)?

A. The diagnosis can be made clinically without paracentesis.
B. Infection is most commonly polymicrobial.
C. Antibiotic therapy is reserved for patients with positive findings on ascitic fluid culture.
D. Gram-negative enteric bacteria are often present.
E. None of the above.

A

ANSWER: D

COMMENTS: SBP is a potentially lethal complication of ascites that affects about 10% of patients with cirrhotic ascites.

Fever and abdominal pain are common manifestations, but the signs and symptoms may be subtle.

Diagnosis requires paracentesis with a demonstration of an elevated ascitic fluid polymorphonuclear neutrophil (PMN) count (>250 cells/mm3) or, eventually, positive find- ings on culture.

Antibiotic therapy should be instituted promptly based on an elevated ascitic fluid PMN count or on symptoms even if the PMN count is lower.

Infection is usually from one organism, most commonly E. coli, Klebsiella, or pneumococcus. A third- generation cephalosporin is typically the preferred antibiotic. Dif- ferentiation from bacterial peritonitis secondary to a surgical condition is critical.

Patients with SBP typically respond to appropriate antibiotics within 48 h, and ascitic PMN counts decrease.

Failure to improve, the presence of polymicrobial infection, or ascitic fluid with a total protein level greater than 1 g/dL, a lactate dehydrogenase (LDH) level greater than the serum level, or a glucose level less than 50 mg/dL suggests secondary peritonitis.

Risk factors for SBP include previous SBP, variceal hemorrhage, and low-protein ascites (<1.0 g/dL). Short- or long-term prophylactic antibiotics may be appropriate for high-risk patients.

195
Q

Eight weeks after open heart surgery with transfusions, a 56-year-old man notes dark urine, fatigue, and anorexia. Physical examination discloses only mild, tender hepatomeg- aly. Laboratory investigations reveal a bilirubin level of 2 mg/dL; an AST level of 540 IU/L; an ALT level of 620 IU/L; an alkaline phosphatase level of 1120 IU/L; and negative assay results for hepatitis B surface antigen (HBsAg), hepatitis B core antibody (anti-HBc), immunoglobulin M anti-hepatitis A virus (HAV) antibody (IgM anti-HAV), and anti-hepatitis C virus (HCV) antibody (anti-HCV). Which of the following is the most likely explanation for the patient’s clinical condition?

A. Acute viral hepatitis A 
B. Acute viral hepatitis B 
C. Acute viral hepatitis C 
D. Acute viral hepatitis D 
E. Acute viral hepatitis E
A

ANSWER: C
COMMENTS: Posttransfusion non-A, non-B hepatitis is mostly the result of HCV infection. The incubation period is usually 5 to 10 weeks, and the mean peak aminotransferase levels are 500 to 1000 IU/L. Anti-HCV antibody is commonly not detectable until 18 weeks after onset of the illness.

Approximately 70% of patients with acute hepatitis C progress to chronic hepatitis and potentially cirrhosis. The negative serologic study results exclude acute infection with HAV and HBV.

Hepatitis D (delta) virus (HDV) is capable of infecting only patients who also have HBsAg because HDV is an incomplete RNA virus.

Hepatitis E (epidemic) virus is rare, except in association with water-borne epidemics in India, the Middle East, and South America.

196
Q

Which of the following clinical conditions is indicated by the presence of serum antibodies against hepatitis B surface antigen (anti-HBs) and absence of serum anti-HBc?

A. Active, acute infection with HBV
B. Normal response to vaccination with the hepatitis B vaccine
C. Chronic active hepatitis secondary to HBV
D. Recovery with subsequent immunity following acute hepatitis B
E. Asymptomatic chronic carrier of HBV

A

ANSWER: B
COMMENTS: The pattern of negative HBsAg, positive anti-HBs, and positive anti-HBc assays is seen during the recovery phase following acute hepatitis B and clearance of HBsAg from the liver.

This antibody pattern may persist for years and is not associated with liver disease or infectivity. Vaccination with the hepatitis B vaccine (genetically manufactured HBsAg particles without HBcAg or HBV DNA) is associated with the development of anti-HBs antibody alone.

Active, ongoing infection with HBV, whether acute hepatitis, chronic active hepatitis, or an asymptomatic chronic carrier state, is manifested by the presence of HBsAg and anti-HBc in serum.

197
Q

A 32-year-old man with no past medical history is brought to the emergency room with altered mental status. Temperature is 100.4°F, pulse 89 beats/min, blood pressure 101/70 mmHg, respiratory rate 19 breaths/min, and saturation 97% on room air. The examination is notable for being awake but not oriented, inconsistently following commands, jaundice, and scleral icterus. Labs are remarkable for sodium 127 mEq/L, AST 2100 IU/L, ALT 2399 IU/L, and international normalized ratio (INR) 1.6. The next most important step in management is:

A. Broad-spectrum antibiotics and percutaneous drainage
B. IV fluid resuscitation and copper chelation
C. Calculate MELD for transplant candidacy
D. Emergent TIPS
E. Transfer to a liver transplant center for emergent
evaluation

A

ANSWER: E
COMMENTS: Acute liver failure (ALF) is a rare but serious emergency, with mortality approaching 60%–80%.

It is defined as a new- onset liver disease with no preexisting cirrhosis, occurring over a span of less than 26 weeks, with an INR ≥ 1.5 and hepatic encephalopathy.

Complications include multiorgan system failure, intracranial hypertension, and rapid death. Therefore timely transfer to a liver transplant center intensive care unit is important. Outcomes following liver transplantation in these patients are poor compared with patients with elective transplants after being on the transplant waiting list.

The MELD scoring system is not applicable in ALF. Instead, the King’s College Criteria is the algorithm aimed at identifying patients that would benefit from liver transplantation versus those in whom it would likely be futile.

The most common cause is acetaminophen overdose, but other causes include other drugs, hepatitis B infection, autoimmune hepatitis, Wilson disease, acute Budd-Chiari syndrome, and HELLP (hemolysis, elevated liver enzymes, low platelet count) syndrome.

Wilson disease is an autosomal recessive disorder of adenosine triphosphate (ATP)-mediated hepatocyte copper transport, causing copper buildup in tissues, with resultant cirrhosis and neurologic manifestations.

Half of the patients have Kayser-Fleischer rings in the cornea.

Even if this patient’s ALF is due to Wilson disease, in the fulminant presentation of this disease, chelator treatment is ineffective, and liver transplantation is still indicated.

198
Q

During liver resection, where is the most common site of
life-threatening hemorrhage?

A. Main hepatic arteries
B. Portal vein branches
C. Intrahepatic vena cava 
D. Hepatic vein branches
E. Hepatic artery variants
A

ANSWER: D
COMMENTS: The crucial steps in a liver resection involve inflow occlusion at the porta hepatis, followed by parenchymal transection, and finally outflow control of the hepatic veins. A thorough exploration of the abdomen must take place at the beginning because findings may affect management and lead one to abort surgery, such as in the case of diffuse peritoneal implants. Additionally, vascular variants such as a replaced right hepatic artery would affect subsequent steps.

Intraoperative ultrasound (IOUS) is an important technique for surveying the liver and delineating intrahepatic vasculature and tumor location. IOUS should be repeated throughout the operation.
Hemorrhage is one of the major hazards during liver resection. Troublesome bleeding is most likely to occur during the division of the hepatic parenchyma, and life-threatening hemorrhage is most commonly from the hepatic veins and their branches.

A variety of intraoperative techniques have been used in an effort to minimize the risk of hemorrhage. Note that inflow occlusion at the porta hepatis (Pringle maneuver) would not occlude hepatic vein hemorrhage. Total hepatic vascular isolation requires occlusion of the inferior vena cava above and below the liver, in addition to the Pringle maneuver.

A disadvantage of any vascular occlusion, however, is the potential for ischemic injury to the liver. This may be particularly hazardous in patients with underlying hepatocellular disease to begin with. Additional techniques that have been employed to help limit blood loss often involve interventions by anesthesia to achieve low central venous pressure, head-down positioning, and vasodilator effects.

199
Q

What is the difference between a standard right hepatectomy and extended right hepatectomy?

A. Extended includes left lobe segments III and IV
B. Extended includes the middle hepatic vein and its parenchymal counterparts
C. Extended includes segment IV and includes the middle hepatic vein
D. Extended includes everything until the line transecting the gallbladder and inferior vena cava (IVC)
E. Extended includes dissection in the plane of the falciform ligament

A

ANSWER: C
COMMENTS: Extended right hepatectomy includes not only the anatomic right lobe (segments V through VIII), but also segment IV. Additionally, the plane of transection is carried over to the left of the middle hepatic vein.

Therefore inflow control is not adequately obtained by ligating the right portal vein and right hepatic artery at the porta hepatis. Smaller inflow vessels need to be ligated within the umbilical fissure.

Left lobectomy includes segments II, III, and IV. Left lateral segmentectomy includes only segments II and III. The umbilical fissure is the segmental plane between the medial and lateral segments of the left lobe of the liver.

A portion of the left branch of the portal vein, known as the pars umbilicus, runs in the inferior portion of the falciform ligament. Dissection is therefore never carried out directly in the segmental fissure. During left lateral segmentectomy, the plane of the parenchymal dissection is to the left of the fissure, whereas with right trisegmentectomy, the parenchyma is divided to the right of the fissure.

Both right and left lobectomies involve dissection well to the right of this plane.

200
Q

Which surgeon performed the world’s first laparoscopic cholecystectomy?

A. Karl Langenbuch 
B. Phillipe Mouret
C. J. Barry McKernan 
D. William B. Saye
E. Eric Mühe
A

ANSWER: E

COMMENTS: Karl Langenbuch performed the very first operation to remove the gallbladder on July 15, 1882. The first “laparoscopic” cholecystectomy was performed by Eric Mühe in Germany in 1985. Although technically different from modern laparoscopic cholecystectomy, it was a landmark contribution. Mühe was severely criticized and, in fact, vilified by the surgical community at the time.

Only years later was the significance of his accomplishment recognized.

Phillip Mouret, a French surgeon from Lyons, performed the procedure in 1987. This was after Mühe had already performed 94 such “laparoscopic” cholecystectomies. J. Barry McKernan and William B. Saye are credited with performing the first laparoscopic cholecystectomy in the United States on June 22, 1988 in Marietta, Georgia.

201
Q

During palpation of the hepatoduodenal ligament, a pulsation is felt dorsal and slightly to the right of the common bile duct (CBD). Which of the following does this pulsation most likely represent?

A. A normal common hepatic artery 
B. A normal right hepatic artery
C. A replaced right hepatic artery 
D. A gastroduodenal artery
E. A right renal artery
A

ANSWER: C
COMMENTS: The most common variation in hepatic arterial anatomy is origination of the right hepatic artery from the superior mesenteric artery.

This is a replaced hepatic artery and not simply an accessory vessel that can be sacrificed with impunity.

When an operation is performed in the right upper part of the abdomen, the pulsations encountered in the porta hepatis and gastrohepatic ligaments should be assessed. If the hepatic artery is absent or small, the surgeon must be alert to the possibility of a replaced hepatic vessel.

When the right hepatic artery originates from the superior mesenteric artery, it courses dorsal to the head of the pancreas and the portal vein and is usually identified dorsolateral to the CBD.

This vessel and its origin can readily be identified with intraoperative ultrasonography.

Only rarely does a replaced right hepatic artery course through the pancreas.

A replaced left hepatic artery originates from the left gastric artery and is located in the gastro- hepatic ligament, where it is frequently encountered during operations on the stomach and gastroesophageal junction.

202
Q

Which of the following anatomic features may contribute to stricture formation after injury to the CBD?

A. The blood supply to the supraduodenal bile duct has a longitudinal pattern.

B. The blood supply to the supraduodenal bile duct has a lateral pattern.

C. The blood supply to the supraduodenal bile duct has a segmental end-artery arrangement.

D. The blood supply to the CBD is derived primarily from the common hepatic artery.

E. The blood supply to the CBD has a fragile anastomotic network.

A

ANSWER: A
COMMENTS: Ischemia is an important contributing factor to the development of postoperative bile duct stricture. The blood supply to the area of the bile duct bifurcation and the distal retro- pancreatic duct is primarily lateral in arrangement, whereas the blood supply to the supraduodenal portion of the bile duct has a primarily axial or longitudinal pattern.

The so-called 3- and 9-o’clock arteries and other small vessels arise from the right hepatic artery and the retroduodenal artery, which is a branch of the gastroduodenal artery, and form the skeleton of a perichole- dochal plexus of vessels.

An additional source of blood supply to the CBD can be the retroportal artery. This vessel arises from the celiac axis or the superior mesenteric artery and generally joins the retroduodenal artery; however, in approximately one-third of individuals it ascends the back of the CBD to the right hepatic artery.

The portion of the bile duct supplied by the longitudinal vessels receives most of its arterial blood supply from below, thus rendering the proximal portion of the duct subject to ischemia after injury or transection.

203
Q

If a patient has complete bile duct obstruction, which of the following does not occur?

A. Triglyceride absorption 
B. Vitamin K absorption 
C. Cholesterol synthesis 
D. Bilirubin conjugation
E. All of the above
A

ANSWER: B

COMMENTS: Bile has a number of critical functions related to the digestion and absorption of fats and the elimination of various endogenous and exogenous substances.

Bile interacts with pancreatic lipase and colipase in the intraluminal hydrolysis of dietary triglycerides. It subsequently solubilizes the monoglycerides and fatty acids produced by triglyceride metabolism by forming mixed micelles.

The micelles facilitate mucosal uptake of triglycerides by permitting transport across the water barrier adjacent to the enterocyte membrane. Although bile therefore plays an important role in triglyceride absorption, a substantial amount of triglycerides can be absorbed, even in the absence of bile, because of the long length of the intestine.

The same is not true for the fat-soluble vitamins A, D, E, and K, which are minimally water soluble and are not absorbed in any substantial amount in the absence of micelles. Patients with long- standing cholestasis generally require supplementation of these fat- soluble vitamins to prevent the clinical effects of deficiency. Bile is the sole pathway for elimination of bilirubin and cholesterol from the body.

Bilirubin is secreted into hepatic bile by an active transport mechanism following hepatic uptake and conjugation. Cholesterol is eliminated both by synthesis of bile acids from cholesterol and by solubilization of cholesterol in bile during secretion.

204
Q

Cholic acid is converted by bacteria to which of the following secondary bile acids?

A. Deoxycholic acid
B. Chenodeoxycholic acid 
C. Lithocholic acid
D. Ursodeoxycholic acid 
E. None of the above
A

ANSWER: A
COMMENTS: The primary human bile acids cholic acid and chenodeoxycholic acid are synthesized from cholesterol in the liver.

The secondary bile acids deoxycholic acid and lithocholic acid are formed in the intestine as the result of bacterial enzyme activity.

7-Ketolithocholic acid is also a secondary bile acid. It is converted to the tertiary bile acid ursodeoxycholic acid in the liver.

205
Q

Conjugated bile acids are primarily absorbed in the intestine by which of the following mechanisms?

A. Active transport in the colon
B. Passive transport in the colon
C. Active transport in the ileum
D. Passive transport in the ileum 
E. Bacterial translocation
A

ANSWER: C
COMMENTS: Enterohepatic cycling of bile acids begins at the hepatocyte level. Bile acids are conjugated in the liver with glycine or taurine, secreted into the biliary system, concentrated and stored in the gallbladder, and then delivered to the duodenum after gall- bladder contraction.

Most bile acids are efficiently resorbed in the intestine. The site and mechanism of intestinal absorption differ according to the form of the bile acid and its corresponding lipid solubility.

Conjugated bile acids are predominantly ionized in the intestinal pH range and are relatively lipid insoluble. Conjugated forms are therefore absorbed by an active transport mechanism in the terminal ileum.

This mechanism accounts for approximately 70%–80% of the enterohepatic circulation.

Bacterial deconjugation of bile acids occurs in the colon and small intestine, as does conversion of primary bile acids to secondary forms.

Deconjugation raises the pKa of bile acids and enables resorption by passive nonionic diffusion, which occurs predominantly in the colon but to some extent in the small intestine as well.

Both primary and secondary bile acids are resorbed and taken back to the liver. Unconjugated forms are then reconjugated and resecreted.

Hepatic bile therefore contains both primary and secondary bile acids, with the primary bile acids normally constituting 60%–90% of the total bile pool. Hepatic synthesis of new bile acids approximates fecal losses of 300 to 600 mg/day.

The bile acid pool cycles four to eight times per day, and hepatic secretion is dependent on enteral return. Disruption of this cycle therefore diminishes bile acid secretion.

Clinical conditions that may be associated with bile acid malabsorption include ileal disease or resection, small bowel dysmotility or obstruction, and blind loop syndrome.

Clinical consequences of this disordered physiology may include fat malabsorption, deficiencies of fat-soluble vitamins (A, D, E, and K), choleretic diarrhea caused by impaired colonic water absorption by bile acids, and formation of gallstones.

206
Q

Which of the following usually produces gallbladder
contraction?

A. Adrenergic stimulation
B. Vasoactive intestinal peptide (VIP) 
C. Somatostatin
D. Cholecystokinin (CCK)
E. Secretin
A

ANSWER: D
COMMENTS: Gallbladder function is subject to many neurohormonal influences. Generally, stimulation of parasympathetic vagal nerves causes gallbladder contraction, and stimulation of sympathetic nerves from the celiac ganglion causes gallbladder relaxation.

Regulation of gallbladder function is actually a complex process that involves the interaction of various neural, hormonal, and peptidergic stimuli on various receptors located on the gall- bladder muscle, blood vessels, and nerves.

Cholinergic stimuli (including vagal) and CCK cause contraction. CCK receptors can be found on both gallbladder smooth muscle cells and intrinsic cholinergic nerves.

Adrenergic stimulation (sympathetic) usually causes relaxation, but selective stimulation of certain adrenergic receptors can cause contraction.

VIP and somatostatin inhibit gallbladder contraction, which can account for clinical biliary manifestations in patients with tumors that secrete those substances or in patients being administered somatostatin agonists.

Many other peptides, hormones, and neurotransmitters may also affect gallbladder function, although their clinical significance is not completely known.

207
Q

Which of the following is true regarding gallbladder volume during the interdigestive period?

A. It correlates with CCK plasma levels.
B. It depends on peristalsis of the CBD.
C. Volume is constant.
D. The gallbladder gradually fills, with cyclic periods of emptying.
E. The gallbladder gradually fills, uninterrupted.

A

ANSWER: D
COMMENTS: Bile flow in the biliary tract varies according to the fasting or fed state of the individual.

CCK, which is released by the duodenum in response to the ingestion of food substances, facilitates delivery of bile to the intestine by stimulating contraction of the gallbladder and relaxation of the sphincter of Oddi.

Normal contraction of the gallbladder in response to meals results in approximately 80% emptying in 2 h.

CCK has no role in the interdigestive or fasting period.
The CBD is for the most part a passive conduit in humans and does not play an active role in biliary motility.

Filling of the gallbladder after it has emptied (i.e., the fasting state) depends on neural and hormonal factors that relax the gall- bladder and increase resistance of the sphincter of Oddi.

During this fasting state, the gallbladder gradually fills, but this filling is interrupted by cyclic periods of emptying, during which time approximately one-third of the gallbladder volume is dispensed.

This cyclic pattern during fasting correlates with the interdigestive myoelectric migratory complex of the intestine and is related to increased levels of plasma motilin.

Motilin is a 21–amino acid peptide, and plasma motilin levels vary cyclically during the fasting period.

208
Q

Which of the following is decreased after cholecystectomy?

A. Size of the bile acid pool
B. Rate of enterohepatic recycling 
C. Rate of bile acid secretion
D. Cholesterol solubility in bile
E. Rate of bilirubin conjugation 

ANSWER: A

COMMENTS: The total size of the bile acid pool is diminished after cholecystectomy as a result of loss of the gallbladder reservoir.

However, cholecystectomy produces a more continuous flow of bile into the intestine, which increases the frequency of enterohepatic cycling and stimulates bile acid secretion.

For these reasons, even though the size of the bile acid pool is diminished, cholecystectomy improves cholesterol solubility in bile.

The solubility of cholesterol in bile depends on the relative molar concentration of cholesterol in relation to the concentration of bile acids and the phospholipid lecithin.

A

ANSWER: A

COMMENTS: The total size of the bile acid pool is diminished after cholecystectomy as a result of loss of the gallbladder reservoir.

However, cholecystectomy produces a more continuous flow of bile into the intestine, which increases the frequency of enterohepatic cycling and stimulates bile acid secretion.

For these reasons, even though the size of the bile acid pool is diminished, cholecystectomy improves cholesterol solubility in bile.

The solubility of cholesterol in bile depends on the relative molar concentration of cholesterol in relation to the concentration of bile acids and the phospholipid lecithin.

209
Q

Which of the following is not a part of the process of cholesterol gallstone formation?

A. Supersaturation of bile with cholesterol 
B. Bilirubin deconjugation
C. Crystal nucleation
D. Aggregation of cholesterol monomers 
E. Stone growth
A

ANSWER: B
COMMENTS: Cholesterol gallstone formation is a complex physicochemical process. The requisite steps in the genesis of cholesterol stones can be conceptually simplified as cholesterol saturation, nucleation, and stone growth.

The cholesterol content of bile must exceed the capacity of bile to solubilize cholesterol in vesicles and micelles.

Cholesterol supersaturation alone, however, is not sufficient to cause stones because this process can occur in normal individuals.

Nucleation must also take place; that is, cholesterol monohydrate crystals must form and aggregate. Finally, the crystals must enlarge by fusion or continued solid deposition to produce a stone large enough to be clinically relevant.

Bacterial infection is thought to be an important pathogenic factor in the development of some pigment stones but not generally cholesterol stones.

Bacterial infection is associated with deconjugation of bilirubin and subsequent formation of insoluble calcium bilirubinate complexes.

Bacterial infection can also result in the production of glycocalyx, an adhesive glycoprotein that plays a role in pigment stone formation.

210
Q

In which of the following patients with symptomatic
cholelithiasis would pigment gallstones be expected?

A. A 45-year-old (y/o) male with a body mass index (BMI) of 49
B. A 27-y/o female status post ileal resection
C. A-38 y/o female with rapid weight loss after a sleeve gastrectomy
D. A 65-y/o female on estrogen therapy
E. A 30-y/o male who is a competitive eater (high-calorie
diet)

A

ANSWER: B
COMMENTS: Changes in bile composition that either increase the relative concentration of cholesterol or decrease the relative concentration of bile acids favor cholesterol gallstone formation.

Situations that lead to increased hepatocyte cholesterol secretion include obesity, rapid weight loss, diets high in calories and polyunsaturated fats, and estrogen therapy (Choices A, C, D, and E).

Theoretically, a relative decrease in the size of the bile acid pool would predispose a person to cholesterol gallstone formation in situations where there is excessive bile acid loss (e.g., ileal disease or resection).

However, stones associated with ileal disease or resection are of the pigment type.

Additionally, total parenteral nutrition (TPN) is associated with pigment gallstones in a high proportion of patients, depending on the duration of therapy.

211
Q

Which of the following is the main chemical component of pigment gallstones?

A. Cholesterol
B. Calcium bilirubinate 
C. Calcium carbonate 
D. Calcium phosphate
E. Calcium oxalate
A

ANSWER: B

COMMENTS: Pigment gallstones are composed primarily of calcium precipitated with bilirubin, carbonate, phosphate, or palmitate anions.

Two relatively distinct types of pigment gallstones are recognized: black pigment gallstones and brown pigment gallstones.

There are differences between black and brown pigment gallstones in terms of gross appearance, chemical composition, pathogenesis, and clinical implications.

Black pigment gallstones are small and spiculated. They contain calcium bilirubinate primarily in polymerized form, as well as calcium carbonate or phosphate.

Brown pigment gallstones are soft and yellow-brown and are also composed primarily of calcium bilirubinate, but they contain more calcium palmitate (fatty acid derived from lecithin) and cholesterol than do black stones. The oxalate salts of calcium play no role in gallstone disease.

212
Q

Which of the following features is more characteristic of
black pigment gallstones than brown pigment gallstones?

A. Association with hepatic cirrhosis
B. Association with bacterial infection
C. Location in the CBD
D. Treatment requiring bile duct drainage 
E. Higher risk for cholangitis
A

ANSWER: A
COMMENTS: There are some important clinical differences between patients with black pigment gallstones and those with brown pigment gallstones.

It is postulated that these stones form by different pathogenic mechanisms.

Stasis and infection are critical factors in the formation of brown pigment gallstones. Bile culture results are positive in most patients with brown pigment gallstones, and scanning electron microscopy demonstrates bacterial colonies or casts within the stones.

Brown pigment gallstones are found more frequently in the CBD than in the gallbladder. They occur in older patients with stasis and in postcholecystectomy patients.

Black pigment gallstones are thought to have a metabolic cause. They often occur in patients with cirrhosis or hemolysis. The precise role of stasis and infection in black stone formation remains unclear, however. Approximately 20% of patients with black pigment gallstones have positive bile culture results, and some investigators have demonstrated bacteria in black stones.

A subset of patients with gallstones have combined features of both black and brown pigment gallstones.

The important therapeutic implication in differentiating black from brown pigment gallstones is that patients with brown pigment gallstones may require a definitive biliary drainage procedure to prevent recurrence, whereas patients with black pigment gallstones may be treated successfully by cholecystectomy alone.

213
Q

A 22-y/o female presents to your office with postprandial epigastric pain. Which of the following sonographic findings is not a feature of gallstone disease?

A. Hyperechoic intraluminal structure
B. Mobility of the intraluminal structure
C. Shadowing posterior to the structure
D. Acoustic enhancement posterior to the structure
E. Sonographic Murphy’s sign in acute cholecystitis

A

ANSWER: D

COMMENTS: External ultrasound (US) imaging has a sensitivity of about 95% for the diagnosis of gallstones.

The three sonographic criteria for gallstones are (1) the presence of a hyperechoic intraluminal focus, (2) shadowing posterior to that focus, and (3) movement of the focus with changes in position of the patient.

Problems in interpretation arise when all of these criteria are not fulfilled. For example, small stones may not shadow well, and impacted stones do not move.

Ultrasound imaging may also fail to diagnose stones if the gallbladder cannot be visualized well because it is contracted or close to excessive bowel gas.

For an optimal elective ultrasound scan, the gallbladder should be examined after the patient has fasted for about 6 h.

Posterior acoustic enhancement is a sonographic feature of hypodense structures such as cysts. The signals behind the structure are “whiter” because the sound wave energy is less attenuated as it passes through.

Additionally, a cholesterol polyp on US is seen with a nonmobile, hyperechoic focus with associated “comet tail” artifact.

A sonographic Murphy’s sign refers to tenderness when the ultrasound transducer is placed over the gallbladder. This is a typical finding in a patient with gallstones and acute cholecystitis.

214
Q

Ultrasound imaging reveals gallstones in an asymptomatic 50-y/o woman. Which of the following is the recommended treatment?

A. Observation
B. Laparoscopic cholecystectomy
C. Open cholecystectomy
D. Ursodeoxycholic acid
E. Extracorporeal shock wave lithotripsy (ESWL)
A

ANSWER: A
COMMENTS: The appropriate management of asymptomatic cholelithiasis is sometimes controversial.

First, the physician must determine whether the patient is in fact asymptomatic because gastrointestinal (GI) complaints other than pain may be attributable to biliary tract disease.

It was formerly thought that symptoms would eventually develop in most patients with silent gallstones and that the risk for subsequent complications was high. Subsequent studies suggested that symptoms develop in about 1%–2% of patients each year and that serious complications are relatively infrequent.

The morbidity, mortality, and cost of intervention in these patients may exceed those of expectant therapy.

The availability of laparoscopic cholecystectomy has not changed the basic indications for surgery, although it has probably altered the symptomatic threshold for surgical referral.

Nonoperative pharmacologic dissolution and ESWL are neither definitive nor cost effective.

Therefore the current incidental finding of asymptomatic cholelithiasis is not an indication for therapy in most situations.

Circumstances that may be exceptions and that merit consideration on an individual basis include:

(1) a transplant patient with anticipated immunosuppression because of the risk for sepsis,
(2) anticipated long-term parenteral nutrition because of associated stasis and sludge formation,
(3) anticipated pregnancy because of the possibility of becoming symptomatic as gallbladder emptying is impaired and because of the potential risk imposed on both the mother and fetus if complicated cholelithiasis occurs,
(4) concurrent abdominal surgery for an unrelated problem because of the relative ease and safety of incidental cholecystectomy in most situations and in con- sideration of the potential for postoperative cholecystitis otherwise, and
(5) bariatric operations because of the high incidence of gall- stones associated with obesity and during rapid weight loss.

In patients requiring massive intestinal resection, concomitant cholecystectomy has been recommended even when the gallbladder is normal because disease will probably develop during parenteral nutrition.

215
Q

In a patient with which of the following conditions is early elective cholecystectomy for symptomatic gallstones not indicated?

A. Elderly status
B. Diabetes mellitus
C. Child class C cirrhosis 
D. TPN-induced gallstones 
E. Chronic renal failure
A

ANSWER: C
COMMENTS: Patients with certain medical conditions are often considered to be at higher risk for morbidity and mortality from gallstone disease.

Complications of cholelithiasis, such as sepsis, perforation, and choledocholithiasis, more frequently develop in elderly patients.

They also have a higher mortality rate during emergency operations.

Elective cholecystectomy can usually be performed safely in the elderly and is recommended for symptomatic patients.

Although the supportive evidence has not always been conclusive, diabetic patients may also be at increased risk, particularly if emergency intervention is required, and should therefore be considered for early elective cholecystectomy.

Gallstones develop in a high proportion of patients maintained on long-term TPN, and reports suggest that complications, emergency operations, and mortality are more frequent in this population as well.

Early cholecystectomy is therefore indicated.

Cholecystectomy is also indicated for patients with chronic renal failure, particularly if they are candidates for renal transplantation.

Patients with hepatic cirrhosis, however, have high morbidity and mortality rates related to cholecystectomy, especially those with hepatocellular dysfunction and portal hypertension.

In selected patients, with Child-Pugh A and B cirrhosis, laparoscopic cholecystectomy can be performed safely with acceptable morbidity.

However, patients with Child-Pugh class C have an unacceptably high risk of morbidity and mortality and therefore cholecystectomy is not advised.

216
Q

A patient with abdominal pain has a normal ultrasound of the gallbladder, followed by a CCK-stimulated cholescintigraphy (CS) scan that demonstrates an ejection fraction (EF) of 14% with reproduction of symptoms. Which of the following is true regarding biliary dyskinesia?

A. Cholecystectomy is not indicated because persistent or recurrent symptoms are likely.

B. Cholecystectomy is successful (resolution or improve- ment of symptoms) 50% of the time.

C. Reproduction of symptoms during CCK-stimulated CS scan is the most accurate predictor of success after laparoscopic cholecystectomy.

D. In patients diagnosed with biliary dyskinesia, improved outcomes postlaparoscopic cholecystectomy are noted in those with an EF closer to zero.

E. The technique used for CCK-stimulated CS in the diagnosis of biliary dyskinesia is important.

A

ANSWER: E
COMMENTS: Biliary dyskinesia is a subsegment of Functional Disorders of the Gallbladder as defined by the Rome III criteria.

Diagnosis is suspected when both typical and atypical biliary symptoms are found in the absence of stones or sludge on ultrasonography.

CCK-stimulated CS has been useful for identifying patients who may have symptoms as a result of motility disorders of the gallbladder, specifically biliary dyskinesia.

The technique used is important as multiple studies have examined the optimal approach and have found this to be an infusion of 0.02 μg/kg of sincalide in infused over 30 min in patients who have been fasting for 3 to 4 h.

In general, laparoscopic cholecystectomy is recommended for patients with typical symptoms and an EF of less than 35%–40% on CCK-CS, with success rates between 85% and 90%. Although initially thought to be a predictor of success, CCK provocation tests (reproduction of pain with CCK-CS) are not believed to be accurate as CCK stimulates other organs that also can contribute to the pain.

No studies to date have found a concrete correlation between decreasing EF and improved outcome.

217
Q

Laparoscopic cholecystectomy is most strongly contraindicated in which of the following situations?

A. Pregnancy
B. Previous upper abdominal surgery
C. Known CBD stones
D. Recent myocardial infarction (MI) with severe coronary artery disease noted on cardiac catheterization, requiring full anticoagulation
E. Gallbladder cancer
A

ANSWER: D
COMMENTS: When laparoscopic cholecystectomy was first introduced worldwide during the late 1980s, there were a number of circumstances in which it was more or less strongly contraindicated.

Today, most contraindications are relative, and in fact the laparoscopic approach is preferred when possible in certain situations that were initially considered contraindications (e.g., acute cholecystitis, choledocholithiasis, and obesity).

Basically, the surgeon must be adequately trained and the patient must be reasonably fit for an operation and give informed consent that includes the possibility of laparotomy.

It must be recognized that there are patients for whom the potential physiologic consequences of CO2 pneumoperitoneum are more important, but the presence of underlying disease itself does not prohibit a laparoscopic approach. In fact, laparoscopic cholecys- tectomy may be more beneficial to the postoperative course of a compromised patient.

Pregnancy is not a contraindication with appropriate precautions, although the physiologic effects on the fetus are not completely known.

Originally deemed a contraindication to laparoscopic cholecystectomy because of risk for dissemination, gallbladder cancer is being reevaluated. Recent literature shows favorable long-term oncologic results in appropriately selected patients (those with early-stage gallbladder cancer with no evidence of liver invasion).

Any contraindication to open surgery remains a contraindication to laparoscopic surgery, as illustrated by the individual with recent MI and severe cardiac disease.

218
Q

Most major bile duct injuries during laparoscopic cholecystectomy occur in patients under which of the following circumstances?

A. Acute cholecystitis
B. Gallstone pancreatitis
C. Choledocholithiasis
D. Elective cholecystectomy
E. Conversion of a laparoscopic procedure to an open procedure
A

ANSWER: D
COMMENTS: There are several risk factors for bile duct injury during laparoscopic cholecystectomy. Pathologic risk factors include severe acute or chronic inflammation.

Several studies have found a statistical correlation between the rate of duct injury and the presence of acute cholecystitis.

Bleeding has long been implicated as a factor predisposing to duct injury during open or laparoscopic cholecystectomy.

Injuries are sometimes attributed to the “anomalous” anatomy of the bile ducts. More often than not, however, such “anomalies” are simply common anatomic variations that the surgeon must recognize to prevent injury (see Question 27).

The surgeon’s experience, or the “learning curve,” is clearly a risk factor because higher rates of duct injury have been well documented in less experienced surgeons. It is interesting to note that there is no convincing evidence that duct injury is more frequent during cases involving laparoscopic management of CBD stones, possibly because these procedures are performed by more experienced surgeons.

Unfortunately, most major bile duct injuries during laparoscopic cholecystectomy have occurred in elective and otherwise uncomplicated cases.

Despite the presence or absence of risk factors, the primary problem resulting in duct injury is misidentification of the anatomy. The most frequent mechanism of injury is mistaking a major bile duct for the cystic duct and clipping and cutting it.

This pitfall is best avoided by correct operative strategy, which means appropriate retraction and adequate dissection to obtain the “critical view of safety.” The critical view is achieved by dissecting the base of the gallbladder off the liver for an adequate distance to visualize the cystic plate and to verify that the only structures entering the gallbladder are the true cystic duct and the cystic artery.

Intraoperative bile duct imaging with cholangiography or laparoscopic ultrasonography can also aid in discerning the anatomy.

If the cystic duct cannot be conclusively identified, the surgeon must resort to alternative approaches such as laparoscopic subtotal cholecystectomy, conversion to an open operation, or termination of the procedure.

219
Q

Which of the following statements regarding the critical view of safety is true?

A. It helps to recognize a bile duct injury, but does not
prevent it.

B. It can be achieved in every laparoscopic cholecystectomy.

C. It does not apply if the patient has aberrant anatomy.

D. It was first described in 1995 in response to increasing bile duct injury rates with the advent of the laparoscopic cholecystectomy.

E. Interest in the critical view of safety has dwindled in recent years due to improved technology.

A

ANSWER: D

220
Q

Which of the following statements about “infundibular
technique” is true?

A. The CBD can be mistaken for the cystic duct when
obtaining this view.

B. It requires dissection of the cystic plate over the bottom third of the gallbladder.

C. It has been shown to prevent bile duct injuries.

D. It facilitates identification of the anatomy in the hepatocystic triangle.

E. It requires clearance of the hepatocystic triangle.

A

ANSWER: A
COMMENTS: Drs. Strasberg, Hertl, and Soper first described the critical view of safety in 1995 in response to an increasing bile duct injury rate with the advent of the laparoscopic cholecystectomy. It has been shown to prevent bile duct injuries and is internationally accepted as a standard practice.

In 2014, Dr. Brunt made it the central focus of the SAGES safe cholecystectomy program, which was formed to further propagate a “universal culture of safety in cholecystectomy.” The three criteria to fulfill the critical view of safety are (1) clearance of tissue from the hepatocystic triangle, (2) complete dissection of the bottom one-third of the cystic plate, and (3) two and only two structures are seen entering the gallbladder.

If these criteria cannot be achieved, it should alert the surgeon that the anatomy is not clearly defined, it is not safe to proceed, and an alternate strategy should be employed.

Adjunct measures that can assist with identifying the biliary anatomy include intraoperative ultrasound or cholangiogram. Otherwise, another operative strategy should be to perform a subtotal cholecystectomy (see Question 39).

The infundibular technique involves encircling the cystic duct and tracing it up to the characteristic funnel-shaped junction with the gallbladder. It does not require further clearance of the hepatocystic triangle, identification of the cystic artery, or cystic plate dissection.

The CBD can be mistaken for the cystic duct using this technique, resulting in a bile duct injury. Therefore it is an unreliable method of ductal identification and should be abandoned.

221
Q

A surgeon encounters difficulty during an elective laparo- scopic cholecystectomy in a healthy 25-y/o woman and converts to an open procedure. The 4-mm common hepatic duct has been transected 1 cm below the bifurcation. Which of the following procedures is the most appropriate?

A. Duct-to-duct repair over a T-tube
B. Duct-to-duct repair without a stent
C. Roux-en-Y hepaticojejunostomy
D. Hepaticoduodenostomy
E. Ligation of the duct and placement of a drain
A

ANSWER: C
COMMENTS: When a transection or resection injury of the extra- hepatic biliary tree is discovered at the time of cholecystectomy, the surgeon must make some careful decisions.

Repair at the time is preferable, provided that the surgeon is adequately experienced in performing such a repair so that a successful outcome is likely.

Unfortunately, the weight of evidence indicates that most primary repairs by the initial operating surgeon have failed, thus necessitating repeated operations and other interventions.

The initial repair of a major duct injury has the best chance for long-term success. A less experienced surgeon should not attempt anastomosis of a small bile duct but seek the help of an experienced colleague if available.

Otherwise, drains should be placed and transfer to an experienced hepatobiliary surgeon arranged.

If repair at the time is appropriate, the standard reconstruction for this type of injury is a Roux-en-Y hepaticojejunostomy.

Duct- to-duct repairs usually fail in this situation.

Hepaticoduodenostomy is not recommended for an injury at this level.

222
Q

How would the bile duct injury described in Question 25 be classified?

A. Bismuth type 1 
B. Bismuth type 2 
C. Bismuth type 3 
D. Bismuth type 4
E. Bismuth type 5
A

ANSWER: B
COMMENTS: The Bismuth classification of bile duct injuries and strictures describes the level of injury in relation to the bifurcation of the main right and left hepatic ducts.

Higher injuries are more difficult. They require a greater degree of technical skill and expertise to reconstruct, and reconstructions may have a lower long-term success rate.

Many of the injuries resulting from laparoscopic cholecystectomy have been higher than those seen with open cholecystectomy. Moreover, many injuries, initially lower, end up being higher when repaired because of the need to debride unhealthy ductal tissue as a result of ischemia or inflammation and infection caused by bile leakage.

With a type 1 injury, 2 cm or more of the common hepatic duct is preserved below the bifurcation.

With a type 2 injury, less than 2 cm remains.

A type 3 injury reaches the bifurcation with preservation of continuity between the right and left ducts.

A type 4 injury involves destruction of the hepatic duct confluence with separation of the right and left hepatic ducts.

A type 5 injury involves a separate inserting right sectoral duct with or without injury to the common hepatic duct.

223
Q

What is the most common anatomic reason for a type B (Strasberg classification) bile duct injury during a laparoscopic cholecystectomy?

A. A short cystic duct

B. An aberrant right hepatic duct

C. A replaced right hepatic artery

D. A replaced left hepatic artery

E. An aberrant left hepatic duct

A

ANSWER: B

COMMENTS: A type B iatrogenic injury is an occlusion of part of the biliary tree, which is usually caused by an aberrant right hepatic duct.

Normal variants include the cystic duct emptying into a right hepatic duct, which then drains into the CBD, or a low- entering right hepatic duct on the CBD.

In both scenarios a surgeon can falsely identify the aberrant right duct as the cystic duct.

The Strasberg classification is generally the easiest to follow and is more applicable in the days of laparoscopy (see Fig. 25.5 below)

STRASBERG/BISMUTH

TYPE A: Bile leak from cystic duct or liver bed (minor duct) without injury (ie, minor duct still in continuity with CBD)

TYPE B: Partial occlusion of the biliary tree, most frequently from an aberrant right hepatic duct

TYPE C: Bile leak from duct (aberrant right hepatic duct) that is not communicating with the common bile duct

TYPE D: Lateral injury of the biliary system, without loss of continuity

TYPE E: Circumferential injury of the biliary tree with loss of continuity

E1: Low CHD stricture/injury, length of CHD stump ≥2cm

E2: Proximal CHD stricture/injury, CHD stump <2cm

E3: Hilar stricture/injury, no residual CHD but hepatic ductal confluence is preserved

E4: Hilar stricture/injury, with involvement of confluence and loss of communication between right and left hepatic duct

E5: Involvement of aberrant right hepatic duct concomitantly with CHD

224
Q

On the second postoperative day following an elective laparoscopic cholecystectomy, a 40-y/o woman complains of nausea and abdominal pain. Examination shows a tempera- ture of 100°F (37.8°C), a pulse of 100 beats/min, mild abdominal distention, and moderate right upper quadrant (RUQ) tenderness. Which of the following would be the most appropriate initial step?

A. Administration of intravenous (IV) antibiotics
B. Magnetic resonance cholangiopancreatography (MRCP) C. Hepatobiliary iminodiacetic acid (HIDA) scan
D. Endoscopic retrograde cholangiopancreatography (ERCP)
E. Percutaneous transhepatic cholangiography (PTC)

A

ANSWER: C
COMMENTS: A bile leak should be highly suspect in the above patient, as it occurs in 1%–2% of patients after elective cholecystectomy.

Other problems such as retained bile duct stones or intestinal injury can occur as well, although they are less frequent.

A HIDA scan is often the most reasonable initial investigation as it can demonstrate whether the leak is ongoing, unlike ultrasound or computed tomography (CT).

These two imaging studies can demonstrate fluid collections or intrahepatic bile duct dilation and would be reasonable after a HIDA is obtained. If a fluid collection is seen, percutaneous aspiration can determine whether the fluid is bile. If a bile leak is confirmed, cholangiography is necessary to establish the site of leakage and help determine further therapy.

Endoscopic cholangiography is generally the first choice and may be all that is necessary for bile leaks that originate from lateral injuries, the cystic duct stump, or the gallbladder fossa.

PTC is necessary for complete anatomic definition in patients with transection or resection injuries or injuries to sectoral hepatic ducts that may not be in continuity with the rest of the extrahepatic bile ducts.

MRCP is not an initial diagnostic examination but can be useful for delineation of bile duct anatomy in complex situations.

225
Q

Which of the following is true regarding the use of intraop- erative cholangiography (IOC) and bile duct injury during laparoscopic cholecystectomy?

A. Selective use of IOC effectively prevents bile duct injury.

B. Routine use of IOC effectively prevents bile duct injury.

C. Selective use of IOC is associated with a higher rate of bile duct injury.

D. The severity of bile duct injury is independent of the use of IOC.

E. Use of IOC increases the intraoperative diagnosis of injury.

A

ANSWER: E
COMMENTS: As long as there are imaging studies to assess the bile ducts intraoperatively, the debate between proponents of routine versus selective use of such studies will continue. Proponents of IOC argue that its routine or liberal use can be advantageous in terms of bile duct injury and that there is an association between routine IOC and lower rates of duct injury.

Cholangiograms can be incomplete or misinterpreted, however, and injuries can occur after IOC has been done. Properly performed IOC does not prevent duct injury.

There is a compelling argument that IOC may limit the severity of duct injury. For example, IOC may allow a surgeon to recognize that the cholangiogram catheter has been placed in the common duct and not in the cystic duct before transection of the common duct.

Some evidence suggests that the number of high duct injuries and anastomotic repairs required to remedy duct injuries has been lower when IOC was performed.

The use of IOC increases the intraoperative recognition of any injury that has occurred.

About 70%–90% of injuries have been identified intraoperatively when IOC has been performed compared with only 15%–25% of injuries when IOC has not been performed.

Failure to interpret the results of IOC correctly can account for missed injuries.

The two primary reasons for misinterpreting the results of IOC are failure to completely visualize the proximal ducts (including both the right anterior and posterior ducts) and extravasation of dye of uncertain origin.

226
Q

A 99mTc-iminodiacetic acid scan in a fasting patient demonstrates the following: normal liver activity, no gallbladder visualization at 60 min, intestinal activity present at 60 min, and gallbladder visualization at 120 min. These findings are most consistent with which of the following situations?

A. Normal study results
B. Acute calculous cholecystitis 
C. Acute acalculous cholecystitis 
D. Chronic cholecystitis
E. Partial bile duct obstruction
A

ANSWER: D
COMMENTS: Since the mid-1970s, technetium-labeled derivatives of iminodiacetic acid (i.e., HIDA, para-isopropylacetanilido-iminodiacetic acid (PIPIDA), and diisopropyl iminodiacetic acid (DISIDA) have been important in the evaluation of biliary tract disease. After IV injection, these radioisotopes are taken up by the liver and excreted into the biliary tract.

The characteristics of a normal study include visualization of the gallbladder within 60 min in fasting patients and the appearance of radioisotope in the duodenum by about the same time.

In nonfasting patients, visualization of the gallbladder may be delayed.

The hepatic phase of the study may demonstrate mass lesions or diminished uptake in patients with hepatic dysfunction.

Such results are similar to those of a liver scan.

With both calculous and acalculous acute cholecystitis, the gallbladder is not visualized because of cystic duct obstruction.

No visualization or delayed visualization is common with chronic cholecystitis.

The distinction between acute and chronic cholecystitis therefore depends on the clinical findings, not simply on abnormal scan results.

Bile duct obstruction may cause delayed or absent clearance of isotope from the liver or delayed hepatic uptake.

Radioisotope scans can be useful in the clinical assessment of disorders other than cholecystitis, including biliary motility, biliary- enteric anastomosis, bile fistulas or leaks, and enterogastric reflux.

227
Q

A 65-y/o female presents with RUQ pain, nausea, and vomiting. Her temperature is 38°C, heart rate (HR) is 115 beats/min, and blood pressure is (BP) 88/62 mmHg; she is saturating 92% on 2-L nasal cannula. Her abdominal examination is significant for a positive Murphy’s sign. Laboratory values are significant for a white blood cell (WBC) count of 19,000/mm3 and a normal total bilirubin. Ultrasound shows a gallbladder wall of 5 mm, pericholecystic fluid, gallstones, and a CBD diameter of 4 mm. She is admitted to the surgical intensive care unit (SICU), IV fluids and vasopressors are started, and antibiotics are administered. Which is the most appropriate surgical management?

A. Percutaneous cholecystestomy tube placement followed by cholecystectomy before hospital discharge

B. Emergent cholecystectomy

C. Percutaneous cholecystostomy tube placement followed by cholecystectomy in 3 months

D. Percutaneous cholecystostomy tube placement and cholecystectomy only if her symptoms recur

E. Percutaneous cholecystostomy tube placement only

A

ANSWER: C
COMMENTS: See Question 33.

228
Q

A 27-y/o female presents with RUQ pain, which started 12 h ago. Her vitals are normal, and her examination is significant for RUQ tenderness without peritonitis. Laboratory values are significant for a WBC count of 12,000/mm3 and normal total bilirubin. Ultrasound shows a gallbladder wall of 4 mm, pericholecystic fluid, gallstones, and a CBD diameter of 3 mm. Which is the most appropriate surgical treatment?

A. IV fluids, antibiotics, and interval elective cholecystec- tomy in 1 month after her symptoms resolve

B. ERCP with sphincterotomy for biliary decompression

C. Cholecystectomy within 72 h of onset of symptoms

D. Biliary drainage followed by interval cholecystectomy within 72 h of onset of symptoms

E. Percutaneous cholecystostomy tube placement

A

ANSWER: C
COMMENTS: See Question 33.

229
Q

A 36-y/o male presents with RUQ pain, nausea, and vomiting, which started 5 days ago. His vitals are significant for a HR of 106 beats/min and BP 126/75 mmHg. His examination is significant for RUQ tenderness and positive Murphy’s sign, but no peritonitis. Laboratory values are significant for a WBC count of 19,000/mm3 and normal total bilirubin. Ultrasound shows a gallbladder wall of 5 mm, pericholecystic fluid, gallstones, and a CBD diameter of 3 mm. Which is the most appropriate surgical treatment?

A. Percutaneous cholecystostomy tube placement

B. Immediate cholecystectomy

C. Cholecystectomy within 72 h of admission

D. IV fluids, antibiotics, and interval elective cholecystec- tomy in 3 months after his symptoms resolve

E. ERCP with sphincterotomy for biliary decompression

A

ANSWER: B
COMMENTS: The Tokyo Guidelines, originally published in 2007 with revision in 2013, are the expert consensus on the diagnosis and management of both cholangitis and cholecystitis.

The three diagnostic criteria for acute cholecystitis are
(A) local signs of inflammation (e.g., Murphy’s sign, RUQ mass/pain/tenderness),
(B) systemic signs of inflammation [e.g., fever, elevated C-reactive protein (CRP), or elevated WBC count], and
(C) imaging characteristic of acute cholecystitis (e.g., wall thickening, pericholecystic fluid).

For a suspected diagnosis of acute cholecystitis, one item in A and one item in B must be present.

For a definite diagnosis, one item must be present from (A), (B), and (C).

Grade III (severe) acute cholecystitis is associated with organ dysfunction with any of the following signs: hypotension, altered mental status, PaO2/ FiO2 < 300, oliguria or serum creatinine > 2 mg/dL, INR > 1.5, or platelet count < 100,000/mm3.

Grade II (moderate) acute cholecystitis is associated with any one of the following conditions: elevated WBC count (>18,000/mm3), palpable tender mass in the RUQ, duration of complaints > 72 h, or marked local inflammation (gangrenous cholecystitis, pericholecystic abscess, hepatic abscess, biliary peritonitis, or emphysematous cholecystitis).

Grade I (mild) acute cholecystitis does not meet the criteria of grade II or III. Initial treatment for all grades includes IV fluid resuscitation and initiation of antimicrobials.

For grade I (mild) acute cholecystitis, cholecystectomy within 72 h of onset of symptoms or if no response to medical treatment within 24 h AND still within the 72 h since the onset of symptoms is recommended.

For grade II (moderate) acute cholecystitis, immediate cholecystectomy or biliary drainage if the patient is at high surgical risk is recommended.

For grade III (severe) acute cholecystitis, antibiotics and supportive measures are recommended.

Performing an emergent biliary drainage as soon as the patient is stable is then recommended.

For patients who undergo biliary drainage and have gallbladder stones, an elective cholecystectomy should be performed 3 months after the patient’s condition improves.

The patient in Question 31 presents with grade III (severe) cholecystitis as evidence by organ dysfunction (hypo- tension).

The recommended initial treatment is percutaneous cholecystostomy tube placement.

The patient in Question 32 presents with grade I (mild) cholecystitis, and cholecystectomy is recommended within 72 h since the onset of symptoms.

The patient in Question 33 presents with grade II (moderate) cholecystitis; the recommended treatment is immediate cholecystectomy or biliary drainage if the patient is at a high surgical risk.

230
Q

A 46-y/o male presents with 2 days of RUQ pain. His labs are significant for a WBC count of 15,000/mm3. His ultrasound notes numerous gallstones with wall thickening and pericholecystic fluid. On examination, his temperature is 99.5°F, and he has a positive Murphy’s sign. What is the best management for this individual?

A. Early laparoscopic cholecystectomy
B. Delayed cholecystectomy after “cool-down” period 
C. IV antibiotics
D. Percutaneous cholecystostomy tube
E. Early open cholecystectomy
A

ANSWER: A
COMMENTS: The former debate over early versus late cholecystectomy for acute cholecystitis has been put to rest.

Prospective studies have demonstrated that early cholecystectomy within the first few days is not associated with higher morbidity or mortality and that delayed surgery requires longer hospitalization, is more expensive, and risks recurrent biliary problems before definitive therapy.

Most patients are treated effectively by stabilization and prompt surgery.

From a technical standpoint, cholecystectomy is often easier during the first day or two of the patient’s illness when the inflammation tends to be more edematous rather than necrotic and hyperemic, as it becomes when the process progresses.

Most advocate cholecystectomy within the first 72 h, while others push this to within the first 96 h.

Laparoscopic cholecystectomy is the preferred treatment in most circumstances, with decreased length of stay and morbidity compared with open cholecystectomy. However, conversion to an open procedure is required more often than when the procedure is performed electively for nonacute symptoms.

231
Q

With regard to acalculous cholecystitis, which of the following statements is true?

A. It most commonly affects elderly patients in an outpatient setting.

B. The primary pathophysiologic feature involves gallbladder stasis.

C. HIDA scan results are usually normal.

D. Ultrasound imaging of the gallbladder is usually normal.

E. Treatment requires cholecystectomy.

A

ANSWER: B
COMMENTS: Approximately 5%–10% of acute cholecystitis cases occur in patients without gallstones. The primary predisposing factor is gallbladder stasis with subsequent distention and ischemia.

Acalculous cholecystitis typically develops in hospitalized patients, often after trauma, unrelated surgery, or other critical illnesses. Factors present in these patients that may contribute to biliary stasis include hypovolemia, intestinal ileus, absence of oral nutrition, multiple blood transfusions, narcotic use, and positive pressure ventilation.

Because of the clinical situation in which acute acalculous cholecystitis occurs, the diagnosis may not be readily apparent. The patient may have fever or unexplained sepsis, and abdominal signs may not be initially appreciated.

The results of imaging studies are generally abnormal. Because of stasis and functional obstruction of the cystic duct, HIDA scanning fails to allow visualization of the gallbladder, and ultrasonography may demonstrate sludge, thickening of the gallbladder wall, or pericholecystic fluid.

None of these findings are specific for the presence of acute acalculous cholecystitis, however, and the diagnosis must rely on clinical suspicion.

Standard surgical treatment consists of cholecystectomy (or cholecystostomy for patients who are too infirm to withstand general anesthesia).

Percutaneous cholecystostomy can be a valuable technique for establishing gallbladder decompression in these critically ill patients.

Later cholecystectomy may not be required if stones are not present and subsequent cholangiography demonstrates a patent cystic duct.

Cholecystectomy is the only effective treatment if the gallbladder is necrotic or gangrenous.

232
Q

The pertinent area of a plain x-ray film of the abdomen of a 78-y/o diabetic man with RUQ pain is shown in Fig. 25.6 (Curvilinear radiolucencies in the RUQ). Which of the following is the appropriate next step?

A. Ultrasound imaging of the gallbladder

B. CT

C. HIDA scan

D. Cholecystectomy

E. ERCP

A

ANSWER: D

COMMENTS: Emphysematous cholecystitis occurs most typically in elderly diabetic men.

Curvilinear radiolucencies in the RUQ have the configuration of the gallbladder, are in the location of the gallbladder, and are diagnostic of gas in the gallbladder wall. In their totality, they are pathognomonic of emphysematous cholecystitis.

Gas may also be seen in the gallbladder lumen. This condition is associated with a high incidence of gallbladder necrosis, perforation, and sepsis.

Unnecessary diagnostic examinations would only delay prompt surgical therapy and possibly affect the outcome adversely. Urgent surgery is needed.

An ultrasound study of an emphysematous gallbladder would show highly reflective shadows as a result of the gas. Differentiation from bowel gas may be difficult, although the diagnosis is usually evident.

About one-third of patients do not have stones.

CT would show the abnormal gas in the gallbladder wall, lumen, or both.

HIDA scans would fail to allow visualization of the gallbladder.

ERCP is unnecessary.

233
Q

A 26-y/o female who is 10 weeks pregnant presents to your office with symptomatic cholelithiasis. She has had two recent trips to the emergency room for significant pain. Her ultrasound from her most recent visit demonstrates a wall of 3 mm, no pericholecystic fluid, an impacted stone in the neck of the gallbladder, and a CBD of 4 mm. On examina- tion, she has mild RUQ pain but no Murphy’s sign. The most appropriate management is:

A. Urgent laparoscopic cholecystectomy
B. Percutaneous cholecystostomy tube
C. Laparoscopic cholecystectomy in 4 weeks
D. Open cholecystectomy in 4 weeks
E. Nonoperative management until term and postpartum laparoscopic cholecystectomy

A

ANSWER: C
COMMENTS: While elective operations during pregnancy have traditionally been avoided for fear of miscarriage/fetal loss and birth defects secondary to anesthetic effect, the data supporting this are mostly theoretical.

The largest study following the out- comes of women undergoing nonobstetric surgery was based on the Swedish Birth Registry. They found no increased risk for congenital malformations or stillbirth when compared with the control population. However, the rates of low birth weight were significantly higher in the surgical group. Additionally, the authors noted an increased incidence of preterm birth and an increased rate of neonatal death within 7 days in the surgery group. However, the authors did note that it was difficult to separate the many possible confounding factors (operation type, surgical indication, and type of anesthesia used) that could contribute to the aforemen- tioned adverse outcomes.

In the case of semielective surgery, as illustrated in the above question, the risks and benefits must be carefully weighed. Symptomatic gallstone disease is second to appendicitis as the most common nonobstetric surgical problem that affects pregnant women. The overwhelming majority of women who become symptomatic during the first trimester of pregnancy will have continuing or recurrent symptoms before delivery.

Without definitive treatment, rehospitalizations are frequent and there is an ongoing risk to both the mother and the fetus.

Most propose that the second trimester is the ideal time to operate. This avoids the risk of spontaneous miscarriage and the theoretic concern of teratogenicity in the first trimester, while also avoiding the risk of preterm birth in the third trimester.

That being said, laparoscopic cholecystectomy has been performed successfully during all stages of pregnancy.

In late term, however, the size of the gravid uterus interferes with trocar placement. For this reason, many surgeons prefer an open approach if surgery is necessary during the third trimester.

234
Q

A 35-y/o female with a BMI of 42 presents for evaluation for bariatric surgery. After discussion, you decide to proceed with a laparoscopic Roux-en-Y gastric bypass procedure. You notice in her chart that a previous RUQ ultrasound shows that she has gallstones, but she denies any symptoms. Which is the most appropriate management of this patient?

A. Perform Roux-en-Y bypass with concomitant cholecys- tectomy.

B. Perform Roux-en-Y bypass followed by interval chole- cystectomy in 1 month.

C. Recommend a gastric band to prevent complications from her cholelithiasis.

D. Proceed with Roux-en-Y bypass and only perform a cholecystectomy in the future if she becomes symptomatic.

E. Recommend a sleeve gastrectomy to prevent complications from her cholelithiasis.

A

ANSWER: D
COMMENTS: Rapid weight loss (>25% of original weight) is the most significant risk factor for gallstone formation after bariatric surgery.

The caloric restriction reduces bile acid secretion, which, combined with excess cholesterol, results in supersaturated bile and stone formation.

In addition, for Roux en y gastric bypass (RYGBP), the decreased CCK secretion and (potential) inadvertent injury to the hepatic branch of the vagus nerve during gastric pouch formation may lead to delayed gallbladder emptying and stone formation.

Prophylactic cholecystectomy during a bariatric operation was once a routine procedure, but now is no longer recommended. It increases intraoperative time and hospital length of stay.

Recent studies have shown the postoperative symptomatic cholelithiasis rate for both Roux-en-Y gastric bypass and sleeve gastrectomy is similar and low at 6%–8%. It is even lower for laparoscopic adjustable gastric band likely due to the decreased postoperative weight loss.

Therefore the most appropriate management is selective cholecystectomy in patients who become symptomatic.

235
Q

A 68-y/o male presented to the hospital with chronic cholecystitis. He was taken to the operating room for a laparoscopic cholecystectomy. Intraoperatively, the critical view of safety could not be obtained due to dense adhesions in the hepatocystic triangle. Which is the most reasonable next operative step?

A. Obtain the infundibular view and then proceed with cholecystectomy.

B. Abort the operation.

C. Proceed with subtotal fenestrating cholecystectomy.

D. Proceed with subtotal reconstituting cholecystectomy.

E. Both C and D.

A

ANSWER: E
COMMENTS: Safe management of a patient with a difficult laparoscopic cholecystectomy requires technical skill, considerable judgment, and familiarity with a spectrum of operative options.

Such options include open cholecystectomy, “fundus first” cholecystectomy, laparoscopic or open cholecystostomy, and laparoscopic or open subtotal cholecystectomy.

Alternatives to total cholecystectomy can help avoid major bile duct or vascular injury under difficult circumstances.

Cholecystostomy tube placement can be lifesaving; potential disadvantages include tube complications, the possible need for a reoperation later, and possible inability to place a tube if the gallbladder is necrotic or gangrenous.

Subtotal cholecystectomy can help avoid a bile duct injury and bleeding and reduce the need for cholecystostomy and reoperation.

The ability to safely perform subtotal excision of the gallbladder laparoscopically can decrease the rate of conversion to an open operation and potential morbidity in critically ill patients.

Two types of subtotal cholecystectomy have been described. A subtotal fenestrating cholecystectomy does not leave a remnant gallbladder, whereas a subtotal reconstituting cholecystectomy does.

There are no long-term data as to which is superior. The goal for both is to resolve the patient’s symptoms and avoid a second operation/ procedure.

Aborting the operation is a very reasonable and safe option if the surgeon does not feel comfortable proceeding, but would not be ideal, as it would require an additional intervention.

236
Q

A 71-y/o female presents to your office for a persistent high-output biliary fistula 3 weeks after you performed a subtotal fenestrating cholecystectomy with subhepatic Jackson Pratt (JP) drain placement for acute cholecystitis. She is asymptomatic. What is the next step in management?

A. ERCP
B. Observation as a persistent fistula is expected after a subtotal fenestrating cholecystectomy
C. Reoperation to close the fistula
D. Make her nil per os and start TPN
E. Octreotide infusion
A

ANSWER: A
COMMENTS: A bile leak is not uncommon following subtotal cholecystectomy, especially of the fenestrated type, but most is self-limited and will close without any intervention.

A persistent bile leak after a subtotal fenestrating cholecystectomy should raise concern for a distal biliary obstruction.

In this case, the most likely etiology would be a retained stone. Therefore an ERCP would be both diagnostic and therapeutic. There is no role for TPN or octreotide or reoperation in this patient.

237
Q

When compared with standard three-or four-port laparoscopic cholecystectomy, single-incision laparoscopic surgery (SILS) is associated with which of the following?

A. Decreased rate of trocar site hernias 
B. Increased bile duct injuries
C. Decreased pain
D. Decreased hospital stay
E. Increased operative time
A

ANSWER: E
COMMENTS: There has been substantial interest in the development of single-incision laparoscopic approaches for many operations, including cholecystectomy, colectomy, fundoplication, and appendectomy.

Proponents of SILS cite the potential advantages of decreased pain and improved cosmesis in comparison with laparoscopic operations using multiple trocar sites.

So far, for most surgeons, SILS has been more difficult and time-consuming than standard laparoscopic approaches.

Two recent meta-analyses have examined postoperative outcomes between single-incision and multiport cholecystectomy. In the first meta-analysis, seven randomized controlled studies were included. The primary outcomes measured were postoperative complications and postoperative pain score, while secondary outcomes measured were operating time and length of hospital stay. The only difference found between the two was operative time, being longer in the SILS group. In the second meta-analysis, nine randomized controlled trials were included. They found significantly improved cosmesis postoperatively, with no difference in pain score or hospital stay.

Postoperative complications, while higher in the SILS group, were not statistically significant.

238
Q

With regard to choledocholithiasis, which of the following statements is true?

A. Common-duct stones are present in one-third of patients undergoing cholecystectomy.

B. The incidence of common-duct stones is highest in elderly patients.

C. Most common-duct stones are composed of calcium bilirubinate.

D. Common-duct stones are found more frequently when cholecystectomy is performed for chronic cholecystitis than for acute cholecystitis.

E. Laparoscopic cholecystectomy is contraindicated if choledocholithiasis is suspected.

A

ANSWER: B
COMMENTS: About 8%–18% of patients with symptomatic gall- stones have choledocholithiasis, which has a spectrum of clinical manifestations. Approximately 6% of patients undergoing cholecystectomy have CBD stones that were completely unsuspected. Proper recognition of common-duct stones is important because of the associated risk for biliary tract obstruction and cholangitis.

The incidence of choledocholithiasis increases with each decade over the age of 60 years.

Most common-duct calculi originate in the gallbladder and are therefore of the cholesterol variety.

Friable “earthy” stones (brown-colored gallstones) contain calcium complexed with bilirubinate and other anions and arise de novo in the common duct in association with biliary stasis and infection.

Choledocholithiasis occurs as often with acute cholecystitis as with chronic cholecystitis. Therefore appropriate evaluation of the patient for potential choledocholithiasis is mandatory.

Laparoscopic cholecystectomy is the preferred approach for patients with choledocholithiasis. This can be accomplished in conjunction with preoperative ERCP for patients with a high likelihood of common duct stones.

For those at intermediate or low risk for common-duct stones, intraoperative duct imaging (cholangiography, laparoscopic ultrasound) is performed. If stones are found in the common duct, most can then be cleared with laparoscopic techniques.

239
Q

Which of the following is the best indication for preoperative ERCP in a patient with gallstones?

A. Obstructive jaundice
B. Gallstone pancreatitis
C. History of jaundice
D. Alkaline phosphatase levels elevated to twice the normal
E. Ultrasound demonstrating numerous small calculi within
the gallbladder

A

ANSWER: A

COMMENTS: The rationale for preoperative ERCP is to identify and remove CBD stones so that patients may subsequently undergo laparoscopic cholecystectomy and, it is hoped, avoid the potential need for an open operation or for operative treatment of the CBD.

However, because endoscopic evaluation of the bile duct entails its own risks, it should be selected for patients at the highest risk for choledocholithiasis. Unfortunately, there are no absolute predictors of CBD stones. The yield of ERCP in identifying CBD stones is highest in patients with obstructive jaundice or clinical cholangitis or when a duct stone is actually seen on ultrasound. In all other circumstances, most patients have negative endoscopic cholangiograms, and the examination was not necessary for most of these patients.

As the number of parameters suggestive of CBD stones increases, however, so does the likelihood of finding stones. There is no substitute for good clinical judgment in the use of preoperative ERCP. It is an unquestionably valuable tool for diagnosing and removing CBD stones, but its overuse is dangerous and must be discouraged.

Magnetic resonance cholangiopancreatography can be a useful noninvasive screening tool for choledocholithiasis that allows ERCP to be reserved for those with positive studies.

Another option for the diagnosis of bile duct stones is an endoscopic ultrasound, which has a specificity and sensitivity of 95% and 98%, respectively. It is less invasive than an ERCP and uses no contrast or radiation.

240
Q

An intraoperative cholangiogram obtained during laparoscopic cholecystectomy shows several 2- to 3-mm filling defects in the distal common duct. What should be done next?

A. Complete the laparoscopic cholecystectomy and perform ERCP postoperatively.

B. Perform open surgical CBD exploration.

C. Administer glucagon, and flush the CBD through the
cystic duct.

D. Laparoscopically dilate the cystic duct, and perform transcystic choledochoscopy.

E. Perform laparoscopic choledochotomy.

A

ANSWER: C
COMMENTS: Choledocholithiasis discovered intraoperatively can often be managed laparoscopically, depending on a number of considerations, such as the size, number, and location of the stones and the size and anatomy of the bile ducts.

When approaching common bile stones laparoscopically, one should start with simple techniques and progress to more complex maneuvers as necessary.

Small stones can often be cleared by flushing the common duct through a transcystic catheter after glucagon has been administered to relax the choledochoduodenal sphincter.

Other transcystic manipulations can be used if the cystic duct is dilated or dilatable (with hydrostatic balloons) and provided that there is a relatively direct course between the cystic duct and the CBD. Such techniques include retrieval with balloon catheters or stone baskets under fluoroscopic or choledochoscopic visualization.

Experienced laparoscopic surgeons can perform choledochotomy when the CBD is sufficiently large and simpler efforts have failed.

In general, the surgeon should not leave common-duct stones untreated but may elect to terminate the procedure when (1) the stones are very small or questionable, (2) the CBD is narrow, (3) laparoscopic clearance is not feasible, and (4) the morbidity of an open CBD exploration is judged to be too high for a particular patient.

Intraoperative endoscopic retrieval of CBD stones has been successful but may be logistically impractical.

Relying on postoperative endoscopy for intentionally neglected stones carries the risk that endoscopic removal may fail.

A traditional open CBD exploration is a safe, reliable fallback for most patients when laparoscopic methods are unsuccessful and the duct is not too small.

241
Q

A 35-y/o female presents with choledocholithiasis. A laparoscopic cholecystectomy with biliary clearance is recommended. Which of the following is FALSE regarding laparoscopic CBD exploration (LCBDE) versus preoperative ERCP followed by laparoscopic cholecystectomy?

A. Morbidity and mortality are equivalent.

B. Hospital stay is shorter for the LCBDE group.

C. Cost is greater for the ERCP followed by the LC group.

D. There are fewer retained stones after the laparoscopic common bile duct exploration (LCBDE) group.

E. Successful CBD clearance is equivalent between the two.

A

ANSWER: D

COMMENTS: The ideal method of treating choledocholithiasis is controversial.

The two-stage approach involves a preoperative ERCP for the CBD clearance followed by laparoscopic cholecystectomy.

The alternative one-stage procedure is laparoscopic cholecystectomy with LCBDE.

The morbidity and mortality rates are equivalent between the two groups. The CBD clearance rates are also equivalent, both around 90%.

The one-stage approach is more cost effective, and the overall hospital stay for the patient is shorter. However, the operative time is longer for the one-stage approach.

There is no significant difference in rates of retained stones between the two groups (both 8%–15%).

242
Q

When compared with IOC, laparoscopic ultrasound for evaluation of the CBD during cholecystectomy is most associated with each of the following except:

A. Better sensitivity for detecting common-duct stones

B. Less time requirement

C. Less need for dissection of cystic structures

D. More accurate identification of the proximal bile ducts

E. Better identification of vascular variations

A

ANSWER: D

COMMENTS: IOC and intraoperative ultrasonography are the most commonly used methods for evaluating the bile ducts during cholecystectomy.

Some of the advantages of sonography are that it is relatively quick, it can be performed without the need for dissection of the cystic duct, and it can easily be repeated.

Intraoperative ultrasound is more sensitive than IOC for the detection of small stones or sludge in the CBD, although these findings may not necessarily be clinically relevant.

Sonography can also demonstrate the vascular anatomy of the hepatoduodenal region.

Ultrasound is less reliable than cholangiography for delineation of the anatomy of the proximal bile ducts, such as the presence of separately inserting segmental hepatic ducts.

Both imaging methods can be useful in the avoidance of bile duct injury.

243
Q

Which of the following is the best treatment for a patient with choledocholithiasis 3 years after cholecystectomy?

A. Administration of ursodeoxycholic acid

B. Percutaneous transhepatic stone extraction

C. Endoscopic sphincterotomy and stone extraction

D. LCBDE and T-tube placement

E. Open CBD exploration and choledochoduodenostomy

A

ANSWER: C

COMMENTS: Most CBD stones found in patients after cholecystectomy can be treated successfully by nonoperative methods.

Stone extraction through a T-tube or endoscopically after endoscopic sphincterotomy if the patient does not have a T-tube in place results in successful duct clearance with a low complication rate in more than 90% of patients.

By definition, bile duct stones occurring more than 2 years after cholecystectomy are considered primary common-duct stones. These are pigment gallstones related to biliary stasis and infection rather than the typical cholesterol stones found in the gallbladder.

In addition to stone removal, some type of ductal drainage procedure is therefore also indicated in most of these patients to prevent stone recurrence.

When performed by experienced clinicians, endoscopic sphincterotomy is successful in more than 90% of patients, and when combined with endoscopic extraction with the use of balloon catheters or baskets, results in stone clearance in 85%–90% of patients.

Duct stones have been removed successfully via the percutaneous transhepatic route when endoscopic approaches are not successful.

A number of situations may make endoscopic clearance of bile duct stones difficult or unsuccessful, including large impacted stones, the presence of a distal bile duct stricture, previous gastrectomy with gastroenterostomy or Roux-en-Y anastomosis, complications of endoscopic sphincterotomy before stone extraction, or the presence of a duodenal diverticulum.

For patients with an altered GI tract, as is the case in those with a Roux-en-Y reconstruction, there are specialized endoscopic or combined endoscopic and laparoscopic approaches available to access the biliary tree.

If access to the bile duct can be achieved endoscopically, adjuvant modalities, such as intracorporeal fragmentation techniques (i.e., mechanical, electrohydraulic, or laser lithotripsy) or ESWL, may allow successful removal of even difficult stones.

Reoperation on the biliary tract for clearance of duct stones is reserved for physiologically fit patients in whom other extraction techniques are unsuccessful.

Ursodeoxycholic acid does not dissolve pigment stones.

244
Q

Which of the following is the most appropriate initial test for the evaluation of obstructive jaundice?

A. HIDA scan

B. Ultrasound imaging

C. CT

D. PTC

E. ERCP

A

ANSWER: B

COMMENTS: All of the aforementioned imaging modalities may be useful for evaluating a patient with obstructive jaundice.

Overall, ultrasound is the most cost-effective initial examination. It permits identification or visualization of ductal dilation, suggests the level of obstruction, and provides information about the liver, the pancreas, and the presence or absence of calculous disease.

CT or magnetic resonance imaging may best delineate the anatomy of mass lesions in the hepatobiliary and pancreatic region and assist in the preoperative assessment of resectability.

Magnetic resonance cholangiography can provide precise delineation of ductal anatomy and is increasingly important in the evaluation of malignant disease.

PTC can demonstrate the proximal extent of obstruction and is useful for assessing the suitability of the proximal hepatic ducts for anastomosis.

ERCP is particularly useful in cases of distal biliary tract obstruction and allows evaluation of the ampullary region.

Both PTC and ERCP allow cytologic or histologic sampling, and both can be used to place catheters for decompression of the obstructed biliary tract. Although 99mTc-iminodiacetic acid scans can demonstrate ductal obstruction, they do not provide sufficient anatomic definition to determine cause or assist in making therapeutic decisions.

245
Q

Two weeks following hepaticojejunostomy for the treatment of a benign bile duct stricture, a patient has a serum bilirubin level of 6 mg/dL. The patient was jaundiced for 4 weeks before the operation and had a preoperative serum bilirubin level of 12 mg/dL. Which of the following is the most likely explanation for this current serum bilirubin level?

A. Anastomotic stricture

B. Persistent delta-bilirubinemia

C. Postoperative hepatitis

D. Normal expected decline after relief of any obstructive jaundice

E. Renal failure

A

ANSWER: B

COMMENTS: After relief of biliary obstruction, there is a prompt increase in bile flow, and normal bile acid secretion resumes within several days.

Serum bilirubin levels decline approximately 50% by 36 to 48 h after surgery and 8% per day thereafter. This rate varies depending on the duration of the jaundice.

Delta-bilirubin is a form of bilirubin that is covalently bonded to albumin and is measured as part of the direct bilirubin fraction. As such, it is not filtered by the kidneys and has the same serum half-life as albumin, approximately 18 days, which accounts for the slow decline in serum bilirubin levels observed in patients following relief of long- standing jaundice.

Although 90% of patients who had jaundice for 1 week or less have a normal serum bilirubin level 3 to 4 weeks postoperatively, only one-third of patients who had jaundice for 4 weeks or longer obtain normal levels by the same time.

Anastomotic stenosis does not usually develop early during the postoperative period.

Postoperative hepatocellular dysfunction as a result of hepatitis or other causes can occur early in the postoperative period, but it is a less likely cause of hyperbilirubinemia in a patient whose serum bilirubin levels are gradually declining and who would be anticipated to have persistent delta-bilirubinemia.

246
Q

Which of the following is the most likely explanation for a serum bilirubin level of 40 mg/dL in a patient with obstructive jaundice?

A. The patient has complete biliary obstruction.

B. The duration of obstruction has exceeded 2 weeks.

C. The patient has associated renal dysfunction.

D. The patient has malignant biliary obstruction.

E. The patient also has Gilbert disease.

A

ANSWER: C

COMMENTS: In the presence of complete biliary obstruction, serum bilirubin levels generally plateau at 25 to 30 mg/dL.

At this point, the daily bilirubin load equals that excreted by the kidneys.

Situations in which even higher bilirubin levels can be found include renal insufficiency, hemolysis, hepatocellular disease, and, rarely, a bile duct–hepatic vein fistula.

Hyperbilirubinemia tends to be more pronounced in patients with obstruction caused by malignant disease than with obstruction resulting from benign causes.

However, malignant obstruction in the absence of the previously enumerated factors does not produce this degree of hyperbilirubinemia.

247
Q

Which of the following conditions is usually associated with the highest incidence of positive bile culture results?

A. Acute cholecystitis

B. Chronic cholecystitis

C. Choledocholithiasis

D. Postoperative bile duct stricture

E. Bile duct malignancy

A

ANSWER: D

COMMENTS: Recognition of clinical situations in which bacteria are likely to be present in bile is important because the presence of bacteria in bile is correlated with the risk for postoperative infectious complications.

Prophylactic antibiotics have decreased infectious morbidity in patients older than 50 years and in those with jaundice, acute cholecystitis, or choledocholithiasis and cholangitis.

Bile cultures are positive in approximately 5%–40% of patients with chronic cholecystitis, 29%–54% of patients with acute cholecystitis, 60%–80% of patients with choledocholithiasis, 59%–93% of patients with cholangitis, and nearly all patients with bile duct stricture. Bacterial infection of bile occurs in 25%–50% of patients with malignant obstruction.

Bile culture results are expected to be positive in any patient with an indwelling biliary tube.

248
Q

Which of the following organisms is most commonly isolated from bile?

A. Escherichia coli

B. Clostridium spp.

C. Bacteroides fragilis

D. Pseudomonas spp.

E. Enterococcus spp.

A

ANSWER: A

COMMENTS: All of the aforementioned organisms are found in the biliary tract, but gram-negative aerobic organisms, particularly E. coli and Klebsiella, are found most frequently.

Other gram-negative aerobic bacteria that can be cultured are Pseudomonas and Enterobacter spp.

Gram-positive organisms, especially Enterococcus spp. and Streptococcus faecalis, are also frequently observed.

Anaerobes are now recognized in 4%–20% of cases, most commonly B. fragilis, followed by Clostridium spp. Polymicrobial infection occurs in approximately 60% of cases.

Prophylactic or therapeutic antibiotic therapy must be effective against the anticipated organisms and tailored according to the local antibiogram.

Specific antibiotic administration and duration also depend on whether it is community-acquired or health care–associated and the severity grade of the biliary tract infection.

For community-acquired grade III and health care–associated biliary infections, consider adding coverage for extended-spectrum beta-lactamase (ESBL)-producing organisms, Pseudomonas and Enterococcus spp.

Data available on the duration of therapy are scarce, and so the following is based on expert opinion.

For grade I cholecystitis, antibiotic therapy can be stopped within 24 h after cholecystectomy or continued for 4 to 7 days if there are emphysematous changes, necrosis, or perforation of the gallbladder.

For cholangitis (any grade), grade II/III cholecystitis, and health care–associated biliary infections, antibiotic therapy should continue for 4 to 7 days after source control is achieved. Certain clinical scenarios, such as bacteremia, may require a prolonged antibiotic course.

249
Q

Which of the following is the most common mechanism leading to bacteria in bile:

A. Ascending infection from the duodenum

B. Hematogenous portal venous spread

C. Hematogenous arterial spread

D. Lymphatic spread

E. Systemic immunosuppression

A

ANSWER: B

COMMENTS: Bile is usually sterile. There are various routes by which bacteria can reach the biliary tract, and although not proved, dissemination from the portal venous system via the liver is favored as the most common mechanism.

Ascending infection from the duodenum does not occur to a significant extent. In addition, evidence suggests that the direction of lymphatic flow is from the liver downward rather than in the reverse direction.

Hematogenous dissemination via hepatic arterial flow is a mechanism of hepatic abscess formation and may lead to bactibilia but is thought to be less common than portal venous spread.

250
Q

Which of the following conditions is sufficient to cause cholangitis with bacteremia?

A. Bacteria in bile

B. Complete duct obstruction

C. Bacteria in bile with bile duct obstruction

D. None of the above

E. Any of the above

A

ANSWER: C

COMMENTS: The pathophysiology of cholangitis requires both bacterial infection of bile and bile duct obstruction with elevated intraductal pressure.

Neither the presence of bacteria in bile nor biliary obstruction alone is sufficient to produce bacteremia.

When bacteria are present in bile and common-duct pressures exceed 20 cm H2O, cholangiovenous and cholangiolymphatic reflux occurs and results in systemic bacteremia.

Partial or complete bile duct obstruction may produce cholangitis if bacteria are present. In fact, cholangitis occurs more commonly with partial obstruction because it is more frequently associated with stone disease, whereas complete obstruction is more often found with malignancy.

Calculous disease is the most common cause of cholangitis, which is understandable because it is associated with both bile duct obstruction and bacterial infection.

251
Q

If an antibiotic is effective against the bacteria present in bile, which of the following is the most important consideration for effective treatment of biliary tract infection?

A. Serum concentration of the antibiotic

B. Bile concentration of the antibiotic in an unobstructed biliary tract

C. Bile concentration of the antibiotic in an obstructed biliary tract

D. Potential renal toxicity of the antibiotic

E. Potential hepatic toxicity of the antibiotic

A

ANSWER: A
COMMENTS: The most important pharmacologic considerations pertaining to selection of antimicrobial agents for the treatment of biliary sepsis are the spectrum of antibacterial activity of the agent and achievement of adequate serum levels of the drug.

Therapy cannot be adequate if the agents selected are not effective against the anticipated organisms (i.e., gram-negative Enterobacteriaceae, enterococci, and anaerobes) or if dosing does not produce sufficient serum levels.

The significance of biliary levels of antibiotics is often discussed, but they are of little clinical importance.

High bile levels of an antibiotic are meaningless if the agent is not effective against the bacteria present. Moreover, agents that achieve high concentrations in the normal biliary tract may not reach such levels in the presence of biliary obstruction.

Aminoglycosides, for example, have traditionally been effective agents against the gram-negative organisms that cause biliary sepsis, but they are not concentrated in bile.

The potential nephrotoxicity of an antibiotic is an important consideration because the risk for renal compromise already exists in a patient with sepsis and biliary obstruction.

This has encouraged the use of nonaminoglycoside drugs for gram-negative coverage, but this consideration is not as important as the activity spectrum and adequate serum levels of the drugs.

252
Q

All of the following are poor prognostic factors in a patient with cholangitis except:

A. Renal failure
B. Liver abscess
C. Cirrhosis
D. Cholangiocarcinoma 
E. Ileus
A

ANSWER: E

COMMENTS: Charcot’s triad, which consists of fever, jaundice, and upper abdominal pain, is the clinical hallmark of acute cholangitis.

When accompanied by shock and changes in mental status, it is referred to as Reynold’s pentad.

The current mortality rate in patients with acute cholangitis is approximately 5%.

Poor prognostic factors include renal failure, liver abscess, cirrhosis, and proximal malignant obstruction.

Cholangitis varies widely in severity, and treatment must be individualized according to the patient’s condition.

Initial therapy consists of fluid resuscitation and antibiotics that are effective against gram-negative organisms, enterococci, and anaerobes.

Approximately 5%–10% of patients initially have severe toxic cholangitis and manifestations of the Reynold’s pentad.

Patients who fail to improve or who deteriorate despite antibiotic and fluid support require urgent biliary decompression. This can generally be accomplished nonoperatively by percutaneous transhepatic or endoscopic approaches depending on the suspected location of the obstruction based on ultrasonographic findings and on the availability of local expertise in these procedures.

The ability to decompress the biliary tract nonoperatively in these cases has been advantageous because it not only allows stabilization of a high percentage of patients but also permits diagnostic cholangiography to be performed when the patient is stabilized.

When initial operative decompression of the biliary tract was the only approach for these critically ill patients, the mortality rate was high, and there was a frequent need for subsequent reoperation on the biliary tract because of the inability to identify or deal with the underlying pathologic condition at the time of the initial operation.

If effective nonoperative drainage of the biliary tract is not possible, surgery should not be delayed in these critically ill patients. T-tube decompression of the CBD is performed.

Choledochoduodenostomy is not performed in critically ill patients, but it can be considered if the CBD is dilated to 15 mm or greater, the patient is physiologically stable, and other conditions permit safe performance of an anastomosis.

253
Q

A 46-y/o female presents with fevers to 38°C and jaundice for 5 days after an uneventful laparoscopic cholecystectomy for symptomatic cholelithiasis. Her HR is 105, but otherwise her vitals are normal. An ultrasound reveals no fluid collections but shows a CBD diameter of 9 mm. Laboratory values include a WBC count of 10,000/mm3 and a total bilirubin of 3 mg/dL. What is the most appropriate management?

A. IV fluid resuscitation and antibiotics only
B. Emergent endoscopic biliary drainage
C. Emergent bile duct exploration
D. Percutaneous cholecystostomy tube placement
E. IV fluids resuscitation and antibiotics, and biliary drainage if no response to treatment in 24 h

A

ANSWER: E

COMMENTS: See Question 58.

254
Q

A 38-y/o male presents to the emergency department with fevers and RUQ pain. His temperature is 39°C, he is tachycardic to 110, and the remainder of his vitals are normal. On abdominal examination, he has a positive Murphy’s sign. Laboratory values are significant for a WBC count of 15,000/mm3 and a total bilirubin of 5.5 mg/dL. Ultrasound shows a thickened gallbladder wall, stones, pericholecystic fluid, and a CBD diameter of 8 mm. What is the most appropriate initial management?

A. IV fluids resuscitation and antibiotics, and biliary drainage if no response to treatment in 24 h

B. IV fluids resuscitation, antibiotics, and urgent biliary drainage

C. Cholecystectomy

D. Percutaneous cholecystostomy tube placement

E. IV fluid resuscitation and antibiotics only

A

ANSWER: B
COMMENTS: As discussed in Question 33, the Tokyo Guidelines also provide diagnostic criteria and a management algorithm for the treatment of acute cholangitis.

The three diagnostic criteria for acute cholangitis are (A) signs of systemic inflammation (e.g., fever, abnormal WBC, increased CRP), (B) cholestasis, and (C) imaging suggestive of biliary dilatation and/or etiology of obstruction (stricture, stone, etc.).

A patient has a suspected diagnosis of cholangitis if he or she has one item in A and one item in either B or C. A patient has a definite diagnosis of cholangitis if he or she has one item from A, B, AND C.

The guidelines stratify patients into severity grades, which allow recommendations for management.

Grade III (severe) acute cholangitis is associated with organ dysfunction with any of the following signs: hypotension, altered mental status, PaO2/FiO2 < 300, oliguria or serum creatinine > 2 mg/dL, INR > 1.5, or platelet count < 100,000/mm3.

Grade II (moderate) acute cholangitis is associated with any two of the following conditions: abnormal WBC count (>12,000/mm3, <4000/ mm3), fever ≥ 39°C, age ≥ 75 years old, total bilirubin ≥ 5 mg/dL, or hypoalbuminemia (lower limit of normal × 0.7).

Grade I (mild) acute cholangitis does not meet the criteria of grade II or III.

Initial treatment for all grades includes IV fluid resuscitation and initiation of antimicrobials.

For grade I (mild), when no response to the initial treatment is observed within 24 h, biliary tract drainage is recom- mended.

For grade II (moderate), biliary tract drainage is immediately performed, in conjunction with IV fluids and antibiotics.

For grade III (severe), after IV fluid resuscitation with antibiotics, organ system supportive care takes priority (positive pressure ventilation, vasopressors, etc.) and biliary tract drainage is recommended when safe to do so.

Treatment for the etiology of acute cholangitis (e.g., cholecystectomy for gallstones) is considered once acute illness has resolved.

The patient in Question 57 presents with grade I (mild) cholangitis after a laparoscopic cholecystectomy, likely from a retained stone. The recommended treatment is IV fluids and antibiotics with biliary drainage reserved for patients who do not improve within 24 h.

The patient in Question 58 presents with grade II (moderate) cholangitis. The recommended treatment is IV fluids, antibiotics, and urgent biliary drainage.

255
Q

Ultrasound of the gallbladder demonstrates a 5mm hyperechoic focus along the gallbladder wall that does not move or produce shadowing and that has a “comet tail” echo pattern behind it. What is the most likely diagnosis?

A. Adenomatous polyp

B. Cholesterol polyp

C. Gallstone

D. Adenomyomatosis

E. Xanthogranulomatous inflammation

A

ANSWER: B

COMMENTS: The term hyperplastic cholecystosis describes a group of benign proliferative conditions of the gallbladder, including cholesterolosis and adenomyomatosis, or adenomatous hyperplasia.

These conditions can be symptomatic and are often diagnosed on the basis of their sonographic features.

Cholesterolosis consists of deposits of cholesterol in foamy histiocytes in the gallbladder wall. A localized collection of such cholesterol-laden cells covered by a normal layer of epithelium and connected to the mucosa by a small pedicle is known as a cholesterol polyp.

Ultrasound imaging shows hyperechoic foci with a “comet tail” artifact. Unlike gallstones, the foci do not move or produce acoustic shadowing.

Adenomatous hyperplasia is a proliferative lesion characterized by increased thickness of the mucosa and muscle along with mucosal diverticula known as Rokitansky-Aschoff sinuses.

Segmental, diffuse, and localized forms of adenomyomatous hyperplasia have been described. Of these, a localized form involving the fundus of the gallbladder is most frequently encountered.

Ultrasound demonstrates a mass lesion or “pseudotumor.” Adenomatous polyps are true neoplasms derived from the glandular epithelium of the gallbladder.

Xanthogranulomatous inflammation is a condition in which foamy histiocytes are found in conjunction with inflammatory cells and a fibroblastic vascular reaction, often with mucosal ulceration.

256
Q

With regard to adenomyomatosis of thegallbladder, which of the following statements is true?

A. It is a premalignant lesion.

B. It results from chronic inflammation.

C. It may cause RUQ pain in the absence of gallstones.

D. It is rarely associated with cholelithiasis and cholecystitis.

E. It is not an indication for cholecystectomy in asymptom- atic patients.

A

ANSWER: C

COMMENTS: Adenomyomatosis is a hyperplastic abnormality of the gallbladder that is not related to inflammation or neoplasia.

Approximately one-half or more of patients with adenomyomatosis also have cholelithiasis and cholecystitis, but the relationship is not causal.

Adenomyomatosis is not a premalignant lesion.

The hyperplastic conditions of adenomyomatosis and cholesterolosis may be associated with functional abnormalities of the gallbladder, as evidenced by disturbances in motility or hyperconcentration during oral cholecystography.

These abnormalities may be the cause of biliary tract symptoms in patients with hyperplastic cholecystosis in the absence of cholelithiasis.

Cholecystectomy can relieve the symptoms in these patients.

257
Q

Which of the following is the second most common type of biliary-enteric fistula?

A. Cholecystocolic

B. Cholecystoduodenal

C. Cholecystoduodenocolic

D. Choledochoduodenal

E. Choledochogastric

A

ANSWER: A

COMMENTS: Almost all internal biliary fistulas are acquired communications between the extrahepatic biliary tree and the intestinal tract.

In rare instances, acquired or congenital bronchobiliary or acquired pleurobiliary fistulas occur.

Biliary-enteric fistulas most commonly involve the gallbladder and the duodenum (70%–80% of cases) and are the result of chronic inflammation caused by gallstone disease.

The second most common fistula occurs between the gallbladder and colon; infrequently, the stomach or multiple sites (cholecystoduodenocolic) are involved.

Occasionally, the biliary site of the fistula is the CBD.

Choledochoduodenal fistulas are most frequently caused by penetrating peptic ulcers, but they might occur in patients with choledocholithiasis and previous cholecystectomy.

Other, less common causes of biliary-enteric fistulas are malignancy and penetrating trauma.

258
Q

A 73-y/o female presents with acute onset of abdominal distention, pain, nausea and vomiting. She has no previous surgeries. She has no abdominal or inguinal hernias on examination. Abdominal x-ray shows dilated small bowel with air-fluid levels and pneumobilia (Fig. 25.7). The obstruction is most likely at which site?

A. Pylorus

B. Duodenum

C. Jejunum

D. Ileocecal valve

E. Colon

A

ANSWER: D

COMMENTS: Gallstone ileus is a rare cause of intestinal obstruction. It is a misnomer because it is a mechanical obstruction and not an ileus.

The most common etiology is erosion of a large gallstone into the duodenum via a cholecystoduodenal fistula, where the stone passes in the GI tract until it obstructs at the ileocecal valve.

Other, less common, locations of fistulas include cholecystogastric and cholecystocolic, which can cause gastric outlet obstruction and colonic obstruction, respectively.

After resuscitation, the priority is relieving the obstruction by removing the stone.

259
Q

With regard to the management of a patient with gallstone ileus, which of the following statements is true?

A. Initial tube decompression and nonoperative management allow spontaneous stone passage in one-third of patients.

B. Operative treatment attempts to displace the stone into the colon without enterotomy.

C. Operative treatment involves enterotomy proximal to the site of obstruction.

D. Cholecystectomy and fistula repair at the time of stone removal are contraindicated.

E. Standard treatment is initial laparotomy for stone removal and reoperation for cholecystectomy when the patient is stable.

A

ANSWER: C

COMMENTS: Gallstone ileus is mechanical obstruction of the GI tract caused by a gallstone that has entered the intestine via an acquired biliary-enteric fistula.

Although gallstone ileus accounts for only 1%–3% of all small bowel obstructions, it is associated with a higher mortality rate than other nonmalignant causes of bowel obstruction because it tends to occur in the elderly population and typical cases are characterized by diagnostic delay as a result of waxing and waning of symptoms (“tumbling obstruction”).

Pathognomonic radiologic features include a gas pattern of small bowel obstruction with pneumobilia and an opaque stone outside the expected location of the gallbladder.

Not all of these radiologic features are usually present, however.

The most common site of obstruction is the terminal ileum. Infrequently, sigmoid obstruction occurs in an area narrowed by intrinsic colonic disease.

Initial therapy is appropriate resuscitation followed by surgery. Spontaneous passage is a rare phenomenon, and nonoperative management is associated with a prohibitive mortality rate.

Stone removal is best accomplished with an enterotomy placed proximal to the site of obstruction. Care must be taken to search for additional intestinal stones, which are present in 10% of patients.

Attempts to crush the stone extraluminally or to milk it distally are contraindicated because they may cause bowel injury.

In rare instances, small bowel resection is necessary if there is ischemic compromise or bleeding at the site of impaction.

The main controversy regarding surgical treatment of gallstone ileus is whether a definitive biliary tract operation with cholecystectomy, fistula repair, and possible common-duct exploration should be performed at the time of stone removal. This decision must be based on sound surgical judgment and consideration of the underlying physiologic status of the patient and the anatomic status of the RUQ.

Up to one-third of patients who do not undergo definitive biliary surgery experience recurrent biliary symptoms, including cholecystitis, cholangitis, and recurrent gallstone ileus.

Furthermore, the rate of spontaneous fistula closure is open to question.

For these reasons, a definitive one-stage procedure should be considered in physiologically fit patients if RUQ dissection does not prove unduly hazardous from a technical standpoint, particularly if residual stones can be demonstrated in the RUQ.

In properly selected patients, a definitive one-stage procedure is not associated with higher operative morbidity or mortality rates. However, because most of these patients are elderly and have a high incidence of comorbid disease, surgical therapy has been limited to stone removal in most instances.

Interval cholecystectomy should be considered for patients with postoperative biliary symptoms and for those with residual RUQ stones, provided that they are physiologically fit.

In reality, because of the compromised underlying status of many of these patients, interval elective procedures are not commonly performed.

260
Q

Which clinical scenario is not an indication for early (within 48 to 72 h) ERCP in a patient with proven gallstone pancreatitis?

A. Cholecystitis

B. Severe pancreatitis as measured by the Ranson score

C. Concomitant cholangitis

D. Choledocholithiasis

E. Both A and B

A

ANSWER: E

COMMENTS: Gallstone disease is the most common cause of acute pancreatitis in the United States.

Early studies have shown that patients with acute pancreatitis have a much higher rate of gallstones in their stool (85%–95%) compared with patients who only have symptomatic cholelithiasis (10%).

In addition, these series showed a high prevalence of impacted ampullary stones in patients who underwent surgery within 48 h after admission for acute pancreatitis.

Today, the debate of which patients benefit from a preoperative ERCP and the timing of this continues to be controversial.

Agreed-upon indications for an ERCP include cholangitis, evidence of choledocholithiasis (visualized CBD stones or dilated CBD), persistent biliary obstruction (bilirubin level > 4 mg/ dL), or clinical deterioration with the biliary system as the suspected source.

Intermediate risk factors for CBD stones include age > 55 years, cholecystitis, a dilated CBD > 6 mm without visualized stones, bilirubin level 1.8 to 4 mg/dL, or other abnormal liver function tests.

First-line endoscopic ultrasound or MRCP should be considered for these patients. Patients with none of these sequelae of gallstone pancreatitis require no further intervention.

Ultimately, cholecystectomy is advised during the hospital admission as the rate of recurrent pancreatitis or biliary complications is unacceptably high after hospital discharge.

261
Q

A 22-y/o female presents to the emergency room with RUQ pain and jaundice. Ultrasound shows a normal-appearing gallbladder without stones, but a diffusely dilated CBD. Follow-up ERCP confirms a diffusely dilated fusiform extrahepatic bile duct without filling defects, and normal-appearing intrahepatic biliary anatomy. What is the most appropriate management of this patient?

A. Observation, since there is no evidence of choledocholithiasis

B. Cholecystectomy

C. Hepaticoduodenostomy

D. Complete surgical excision of the dilated bile duct segment and reconstruction with a hepaticojejunostomy

E. Transduodenal excision of the dilated intramural segment within the duodenal wall

A

ANSWER: D

COMMENTS: Choledochal cysts are a rare cause of RUQ pain and jaundice. They are more common in women of Asian descent and are usually diagnosed in infants, but may present in adults as well.

They are premalignant lesions, carrying a 10%–30% risk of carcinoma, so observation is not appropriate.

The most accepted theory of their pathogenesis is related to an anomalous pancreaticobiliary junction, which allows pancreatic fluid to reflux up into the biliary tract. This results in chronic inflammation and cystic degeneration of the bile duct.

This patient has a type I cyst, which involves fusiform dilatation of the extrahepatic biliary tree and is treated by complete excision and hepaticojejunostomy reconstruction.

Type II cysts appear as a diverticulum off the CBD and are treated by excision, as well as biliary-enteric drainage if an anomalous pancreaticobiliary junction is present.

Type III cysts, also known as choledochoceles, appear as intramural CBD cystic dilatation within the duodenal wall. Their pathogenesis is not clear and so treatment is directed at relieving the biliary or duodenal obstruction (if present). This can be approached via a trans-duodenal excision or endoscopic sphincteroplasty.

Type IV disease involves both the intra- and extrahepatic biliary trees. Treatment depends on the anatomic pattern of disease.

Type V disease, also known as Caroli disease, involves only the intrahepatic bile ducts. Surgical treatment ranges from liver resection to transplantation depending on the extent of ductal involvement.

262
Q

With regard to balloon dilation of benign biliary strictures, which of the following statements is true?

A. Dilation can be performed via the transhepatic or endoscopic route.

B. Repeated dilations are not often required.

C. Perforation of the bile duct is the most frequent complication.

D. Better success is obtained with anastomotic strictures than with primary duct strictures.

E. The long-term success rate is better than that achieved with surgical repair.

A

ANSWER: A

COMMENTS: Nonoperative dilation of benign biliary strictures via endoscopic or percutaneous transhepatic access is an alternative to surgery that may be appropriate for some patients.

Repeated dilations are often required, but overall success rates of 70%–80% at 2 to 3 years of follow-up have been reported. Success has generally been somewhat higher in patients with primary ductal strictures than in those with strictures of biliary-enteric anastomoses.

Bleeding and sepsis have been the most frequent complications and can be life-threatening.

Data on long-term results are limited. Comparison between balloon dilation and surgery has demonstrated better long-term results (approximate mean follow-up at 5 years) with surgery, but no difference in overall morbidity, hospitalization, or cost between the two therapies. It cannot be ensured that the treatment groups are comparable, however.

Nonoperative dilation of biliary strictures may be appropriate as initial treatment of a strictured biliary anastomosis or for patients in whom surgical repair is deemed excessively difficult or dangerous.

The decision about how a biliary stricture is initially treated and when nonoperative maneuvers are abandoned in favor of surgery should be made in consultation with a skilled endoscopist, an interventional radiologist, and an experienced hepatobiliary surgeon.

263
Q

A 40-y/o man is evaluated for fluctuating jaundice, pruritus, and fatigue. Liver enzyme levels demonstrate cholestasis. Ultrasound imaging does not show gallstones or bile duct dilation. What diagnostic test should be obtained next?

A. Measurement of serum antimitochondrial antibodies

B. CT

C. HIDA scan

D. ERCP

E. Liver biopsy

A

ANSWER: D
COMMENTS: The findings described are fairly typical of sclerosing cholangitis, which can also be discovered in asymptomatic patients based on a cholestatic liver enzyme pattern.

Sclerosing cholangitis is a disease of undetermined cause characterized by inflammatory fibrosis and stenosis of the bile ducts.

The process can be considered primary when no specific etiologic factor is identified or secondary when associated with specific causes, such as bile duct stones, operative trauma, hepatic arterial infusion of chemotherapeutic agents, or intraductal instillation of various irritants for the treatment of echinococcal disease.

Primary sclerosing cholangitis may be an isolated finding or may occur in conjunction with a variety of other disease processes, most commonly ulcerative colitis and pancreatitis.

Although the cause of primary sclerosing cholangitis is unknown, most attention has focused on an autoimmune or infectious cause. Evidence of an autoimmune cause is largely inferential and based on the association of sclerosing cholangitis with a variety of autoimmune diseases.

Abnormal immunologic parameters can be found in the serum of some patients with sclerosing cholangitis, but there are no specific serologic markers for the disease.

Antimitochondrial antibodies are generally associated with primary biliary cirrhosis. The diagnosis is usually made following ERCP showing multiple strictures and dilations, which give a “beaded” appearance to the ducts.

Magnetic resonance imaging of the bile ducts may also show abnormalities.

Typically, sclerosing cholangitis is a diffuse process that affects both the intrahepatic and extrahepatic bile ducts. In some cases, more limited involvement of the distal bile duct, the intrahepatic ducts, or the area of the bifurcation can be seen.

Liver biopsy may show fibro-obliterative cholangitis or cirrhosis as the disease progresses.

264
Q

Definitive treatment of a patient with sclerosing cholangitis and biliary cirrhosis involves which of the following?

A. Ursodeoxycholic acid

B. Corticosteroids

C. Endoscopic balloon dilation and stenting

D. Extrahepatic bile duct resection and transhepatic stenting

E. Hepatic transplantation

A

ANSWER: E

COMMENTS: Once sclerosing cholangitis has progressed to cirrhosis, the only definitive treatment is hepatic transplantation.

The results of transplantation are generally similar to those obtained when it is performed for other indications.

Before the development of cirrhosis, a number of medical and surgical therapies may be useful. Pharmacologic approaches have included the use of immunosuppressants, bile acid–binding agents, and antifibrotic and antimicrobial drugs.

Unfortunately, there is little evidence that any medical therapy has been effective in slowing progression. Some hopeful results have been reported with ursodeoxycholic acid, which may improve liver enzyme test results and liver histologic study results.

Dominant strictures can be treated operatively or by nonoperative dilation via endoscopic or percutaneous transhepatic approaches.

The long-term efficacy of nonoperative approaches has often been limited, however.

Select patients with predominantly extrahepatic or bifurcation strictures have been treated successfully with bile duct resection followed by Roux-en-Y reconstruction and long-term anastomotic stenting.

265
Q

A 63-y/o female undergoes a laparoscopic cholecystectomy for symptomatic cholelithiasis. The pathology report states that there is a foci of carcinoma which invades, but does not penetrate through, the lamina propria. What is the most appropriate management?

A. Laparoscopic resection of the gallbladder fossa

B. Open extended cholecystectomy including segment 4b and 5 liver resection as well as removal of porta hepatis lymph nodes

C. Cholecystectomy is sufficient

D. CBD excision to achieve negative margins with Roux-en- Y hepaticojejunostomy reconstruction

E. Radiation and chemotherapy

A

ANSWER: C

COMMENTS: Gallbladder cancer is an aggressive malignancy that is often asymptomatic until it reaches an advanced stage.

This is an early-stage T1a tumor that was found incidentally. The treatment is cholecystectomy alone because the tumor is limited to the lamina propria and the chance of lymph node disease is less than 3%.

A more advanced T1b tumor, which presents with invasion of the muscularis proper with evidence of lymphovascular invasion, requires an open extended cholecystectomy.

Resection includes a 2-cm rim of liver parenchyma from the gallbladder fossa (segments 4b and 5) as well as removal of pericholedochal, periportal, hepatoduodenal, right celiac, and posterior pancreaticoduodenal lymph nodes.

Prognosis is substantially improved if an R0 resec- tion can be obtained. Therefore a CBD resection with a Roux-en-Y hepaticojejunostomy reconstruction may be necessary to achieve a negative cyst duct margin.

Radiation therapy and chemotherapy have generally been ineffective for the treatment of gallbladder cancer.

266
Q

Ultrasound imaging demonstrates a 15-mm polypoid lesion in the gallbladder of an asymptomatic 60-y/o patient. Which of the following best describes the recommended treatment?

A. Observation with repeated ultrasound studies in 6 months

B. Cholecystectomy

C. Cholecystectomy if the patient is female

D. Cholecystectomy only if symptoms develop

E. Cholecystectomy only if the patient also has gallstones

A

ANSWER: B
COMMENTS: Polypoid lesions of the gallbladder may be benign, premalignant, or malignant.

Inflammatory polyps and cholesterol polyps are benign, nonneoplastic lesions.

Benign adenomas are neoplasms that have a malignant potential similar to that of adenomas arising in other areas of the GI tract.

Polypoid lesions are typically diagnosed by ultrasound imaging and occasionally by other imaging modalities, such as CT.

The indications for cholecystectomy for the treatment of a polypoid lesion are (1) symptoms and (2) possible malignancy.

The risk for malignancy is related to the size of the lesion; it is higher for lesions that are 10 mm or larger and is quite substantial for lesions measuring 15 mm.

Therefore cholecystectomy is performed if the patient has biliary tract symptoms—regardless of polyp size or the presence or absence of gallstones—or if the lesion is larger than 10 mm.

Polypoid lesions in patients 60 years or older are also more frequently malignant.

The use of laparoscopic cholecystectomy for polypoid lesions is controversial.

Proponents argue that the laparoscopic approach is appropriate because most polyps are benign and even limited cancers may be cured by cholecystectomy alone.

However, gallbladder leakage is not infrequent during laparoscopic cholecystectomy, and consequent dissemination of otherwise “curable” early cancers has been reported.

It is generally advised that “open cholecystectomy” be performed for patients considered at risk for gallbladder cancer.

267
Q

Which of the following is a contraindication to resection of an adenocarcinoma of the bile duct?

A. Tumor location in the distal CBD

B. Tumor location at the bifurcation of the bile duct

C. Invasion of the right and left portal vein

D. Invasion of the right portal vein and right hepatic artery

E. None of the above

A

ANSWER: C
COMMENTS: Cancers of the extrahepatic bile ducts usually carry a poor prognosis because these tumors are frequently beyond the confines of surgical resection at the time of diagnosis.

Substantial palliation can often be achieved with therapy directed at the relief of biliary obstruction.

The prognosis is related to tumor location, resectability, and histologic pattern.

Proximal lesions at or near the hepatic bifurcation are most common but are also least often resectable and therefore have a less favorable prognosis.

Aggressive resection of proximal lesions, usually including hepatic resection, can improve survival.

Hilar cholangiocarcinoma (Klatskin tumor) is considered unresectable if there is metastatic disease, bilateral involvement of the portal vein or hepatic artery, or bilateral extension of the tumor to second-order biliary radicles.

Hepatic transplantation for otherwise unresectable tumors is performed at highly specialized centers, but is not the standard of care.

Distal lesions resectable by pancreaticoduodenectomy have the best prognosis, with a 5-year survival rate of approximately 30%–50%.

Palliative decompression can be achieved by surgical anastomosis, surgical intubation, or endoscopic or percutaneous catheter placement.

The most appropriate method of palliative decompression for a particular patient depends on tumor location and extent, the patient’s underlying condition, the expertise of the surgeon, and the anticipated complications associated with each technique.

Nonoperative decompression is preferred for patients who are demonstrated to have metastasis or otherwise unresectable disease before surgery.

268
Q

With regard to the hepatic anatomy, the falciform ligament divides the ___ from the ___?

A. Caudate lobe, quadrate lobe

B. Right lobe, left lobe

C. Left medial section, left lateral section

D. Left medial section, right lobe

A

Answer: C
The Falciform ligament divides the left lateral section from the left medial section.

The plane between the gallbladder fossa and the inferior vena cava (IVC—referred to as Cantlie’s line) divides the right and left lobes.

The falciform ligament, along with the round, triangular, and coronary ligaments may be divided in a bloodless plane during liver resection (Figs. 31-1 to 31-3). (See Schwartz 10th ed., p. 1265.)

269
Q

The most common variant of normal hepatic artery anatomy is

A. Replaced left hepatic artery from the left gastric artery

B. Completely replaced common hepatic artery from the superior mesenteric artery (SMA)

C. Replaced right and left hepatic arteries

D. Replaced right hepatic artery from the SMA

A

Answer: D

Understanding the anatomic variants of the hepatic arterial supply is important to avoid complications during liver surgery.

The standard arterial anatomy is as follows: the common hepatic artery arises from the celiac trunk, and then divides into the gastroduodenal and proper hepatic artery.

In a standard configuration, the proper hepatic artery gives rise to the right gastric artery, but this is variable.

The proper hepatic artery then divides into the right and left hepatic artery (Fig. 31-4). However, this standard arterial configuration only occurs in 76% of patients.

The most common variants include replaced right hepatic artery from the SMA (10–15%), replaced left hepatic artery from the left gastric artery (3–10%), replaced right and left hepatic arteries (1–2%), and the completely replaced common hepatic artery from the SMA (1–2%) (Fig. 31-5). (See Schwartz 10th ed., p. 1266.)

270
Q

Which of the following correctly pairs the segments of the liver and their associated systemic venous drainage?

A. Segments I, II, III: Right hepatic vein

B. Segment IV: Right hepatic vein

C. Segment I: IVC

D. Segment V, VI, VII, VIII: Left hepatic vein

A

Answer: C

There are three hepatic veins (right, middle and left) that serve as the outflow for the hepatic circulation and drain into the suprahepatic IVC.

The right hepatic vein drains segments V to VIII;

The middle hepatic vein drains segment IV, as well as segments V and VIII; and

The left hepatic vein drains segments II and III.

The caudate lobe drains directly into the IVC.

271
Q

With respect to the enterohepatic circulation of bile, where are the majority of bile salts reabsorbed?

A. Duodenum

B. Proximal jejunum

C. Terminal ileum

D. Colon

A

Answer: C

Bile salts are sodium and potassium salts of bile acids derived from cholesterol by hepatocytes.

After synthesis, the primary bile acids cholic and chenodeoxycholic acid are conjugated to either taurine or glycine and then secreted into the biliary system.

Approximately 90 to 95% of these primary bile salts and acids are absorbed by active transport at the terminal ileum, while the remainder enter the colon and are converted to secondary bile acids (deoxycholic and lithocholic acids) and their associated salts by resident bacteria.

Bile acids and salts reabsorbed in the terminal ileum are reabsorbed through the portal circulation, while those lost in the stool are replaced by hepatic synthesis. (See Schwartz 10th ed., p. 1270.)

272
Q

Which of the following compounds is not synthesized predominantly by the liver?

A. Albumin
B. Factor VIII
C. Factor VII
D. Factor II

A

Answer: B
The liver is the largest gland in the body, and responsible for synthesis of the majority of plasma proteins.

The liver produces approximately 10g of albumin per day, and albumin measurement can therefore be used as a surrogate or liver synthetic function.

This must be interpreted with caution, as albumin levels can be influenced by a host of factors unrelated to hepatic function, and albumin’s longhalf-life (15–20days) makes it a poor marker for acute hepatic dysfunction.

Most clotting factors are synthesized predominantly in the liver, except for factor VIII.

Due to this fact, the prothrombin time (PT) and international normalized ratio (INR) may also be used as markers of hepatic synthetic function. However, these too should be interpreted with caution as other conditions, including vitamin K deficiency and warfarin use, may prolong a patient’s PT/INR.

(SeeSchwartz10thed.,p.1271.)

273
Q

The gold standard for identifying liver lesions by imaging is

A. Intraoperative ultrasound

B. Computed tomography (CT) with triple-phase contrast

C. Magnetic resonance imaging (MRI) with gadoxetate-
based contrast

D. Positron emission tomography (PET) scan

A

Answer: A
The use of intraoperative ultrasound of the liver has rapidly expanded over the years with the increasing number and complexity of hepatic resections being performed.

It has the ability to provide the surgeon with real-time accurate information useful for surgical planning.

Intraoperative ultrasound is considered the gold standard for detecting liver lesions, and studies have shown that it can identify 20% to 30% more lesions than other preoperative imaging modalities.

Importantly, it has been shown to influence surgical management in almost 50% of planned liver resections or malignancies.

Applications for intraoperative ultrasound of the liver include tumor staging, visualization of intrahepatic vascular structures, and guidance of resections plane by assessment of the relationship of a mass to the vessels. In addition, biopsy of lesions and ablation of tumors can be guided by intraoperative ultrasound. (See Schwartz 10th ed., p. 1273.)

274
Q

The most common cause of acute liver failure (ALF) in the United States is

A. Acute viral hepatitis
B. Cardiogenic shock
C. Autoimmune hepatitis
D. Drug/toxin-induced (including acetaminophen)

A

Answer: D
Acute liver failure (ALF) is defined as development of hepatic encephalopathy within 26 weeks of severe liver injury in a patient with no history of liver disease or portal hypertension.

In developing countries, the most common etiology of ALF is viral infections, including hepatitis B, A, and E.

In the West (including the United States, Australia, United Kingdom, and most of Europe), 65% of ALF cases are related to drugs and toxins, especially acetaminophen.

(See Schwartz 10th ed., pp. 1275–1276.)

275
Q

A patient presents with painless jaundice and is found to have cirrhosis. They have no history of alcohol abuse, but do note a history of DM and pseudogout. They also mention that multiple members of their family have suffered from cirrhosis. What is the most likely etiology for their cirrhosis?

A. Alcohol abuse

B. Wilson disesase

C. Alpha-1-antitrypsin deficiency

D. Hemochromatosis

A

Answer: D
Chronic hepatitis C infection is the most common cause of chronic liver disease in the US.

Other etiologies include alcohol abuse, nonalcoholic steatohepatitis (NASH), and autoimmune diseases (primary biliary cirrhosis, primary sclerosing cholangitis, and autoimmune hepatitis).

Hereditary hemochromatosis is the most common metabolic cause of cirrhosis, and should be suspected if a patient presents with skin hyperpigmentation, DM, pseudogout, cardiomyopathy, or a family history of cirrhosis.

276
Q

Which of the following is not one of the physiologic changes noted in patients with cirrhosis?

A. Reduced resting energy expenditure

B. Reduced muscle and fat stores

C. Increased cardiac output

D. Decreased systemic vascular resistance

A

Answer: A
The clinical manifestations of cirrhosis are the result of numerous physiologic changes associated with a patient’s progressive liver failure.

Hypoalbuminemia results in finger clubbing, while spider angiomata and palmar erythema are thought to be caused by alterations in sex hormones. The physiologic basis for feminization of males (gyneco mastia, loss of chest/axillary hair, testicular atrophy) with cirrhosis is less well understood.

Portal hypertension manifests as caput medusa and varices.

Cirrhotic patients suffer from chronic malnutrition, which may be associated with weakness, weight loss, and decreased at and muscle stores.

Despite this fact, patients with cirrhosis have elevated resting energy expenditure. Also noted are increased cardiac output and heart rate with decreased systemic vascular resistance and blood pressure.

(See Schwartz 10th ed., p. 1279.)

277
Q

Clinically significant portal hypertension is evident when the _______ exceeds ______ mm Hg.

A. Wedged hepatic venous pressure, 10

B. Free hepatic venous pressure, 20

C. Hepatic venous pressure gradient, 10

D. Hepatic venous pressure gradient, 20

A

Answer: C
Portal hypertension occurs when the pressure in the portal system is increased due to factors that may be divided into three categories.

Presinusoidal causes of portal hypertension include sinistral/extrahepatic (splenic vein thrombosis, splenomegaly, splenic atrioventricular fistula) and intrahepatic (schistosomiasis, congenital hepatic fibrosis, idiopathic portal fibrosis, myeloproliferative disorder, sarcoid, graft-versus-host disease) etiologies.

Sinusoidal portal hypertension is a consequence of cirrhosis of any etiology.

Postsinusoidal hypertension can also be divided into intrahepatic (vascular occlusive disease) and posthepatic (Budd-Chiari, congestive heart failure [CHF], IVC webs) etiologies.

In evaluating patients with suspected portal hypertension, an enlarged portal vein on routine abdominal ultrasonography may suggest portal hypertension but this is not diagnostic.

Doppler ultrasound allows identification of vascular occlusion and the direction of portal venous low.

CT and MR angiography are useful for evaluating portal venous patency and anatomy.

The most accurate method or measuring portal hypertension is hepatic venography. This procedure introduces a balloon catheter directly into the hepatic vein where free hepatic venous pressure (FHVP) is measured. The hepatic vein is then occluded by inflation of the balloon allowing measurement of the wedged hepatic venous pressure (WHVP).

The hepatic venous pressure gradient (HVPG) may then be calculated by subtracting the FHVP rom the WHVP (HVPG = WHVP– FHVP).

Clinically significant portal hypertension is defined as HVPG greater than 10 mm Hg.

(See Schwartz 10th ed., pp. 1280–1281.)

278
Q

All of the following are considered appropriate during the management of an acute variceal hemorrhage EXCEPT

A. Endoscopy with variceal hemorrhage

B. Short-term antibiotic prophylaxis

C. Somatostatin analogues

D. Recombinant factor VIIa

A

Answer: D
Variceal bleeding is the leading cause of morbidity and mortality in those with portal venous hypertension. Approximately 30% of patients with compensated cirrhosis and 60% of those with decompensated cirrhosis will have varices, and one-third of these patients will experience a variceal bleed.

These episodes carry a 20-30% risk of mortality. Prevention of variceal bleeding may be accomplished through administration of non-selective beta-blockers (eg propanolol) and routine endoscopic surveillance and variceal band ligation.

In the case of acute variceal bleeding, patients should be admitted to an ICU for resuscitation and management.

While prompt resuscitation is critical, administration of both blood products and crystalloid should be done with care. A target hemoglobin of 8g/dL is appropriate, and administration of platelets and FFP may be considered for patients with thrombocytopenia or severe coagulopathy.

However, over-resuscitation with both blood products and crystalloid solution has been associated with increased risk of rebleeding and morbidity. Use of recombinant factor VIIa has not been shown to be better than standard therapy, and is not recommended.

Patients with cirrhosis who experience a variceal bleed are at high risk for developing bacterial infection, including spontaneous bacterial peritonitis (~50% of infectious complications), pneumonia, and UTI. These carry not only their own risk of morbidity and mortality, but also are associated with increased risk of re-bleeding.

For this reason, short-term antibiotic prophylaxis (eg ceftriaxone) is recommended for patients with acute variceal bleeding.

Management of the bleeding can be accomplished with vasoactive meds, including vasopressin and somatostatin analogues (eg octreotide). These therapies cause splanchnic vasoconstriction and slow the flow of blood to the varices. Though vasopressin is the most potent vasoconstrictor, it is limited by its systemic effects. This somatostatin analogues are the preferred agent.

Further therapy for bleeding varices should include endoscopy with variceal band ligation.

279
Q

A cirrhotic patient is admitted with variceal bleeding. The bleeding is controlled with pharmacologic therapy, and the patient recovers rom the acute episode. Assuming they receive no other therapies to treat their varices or their underlying cause, what is the likelihood that they will experience a recurrent variceal bleed within 2 years?

A. 20%
B. 40%
C. 70%
D. 100%

A

Answer: C

The risk of re-bleed for patients is 70% over 2 years if they receive no further treatment. Patients who recover from an episode of variceal bleeding should be treated with follow-up endoscopy and variceal band ligation.

In appropriate patients, transjugular intrahepatic portosystemic shunt (TIPS) or orthotopic liver transplant should be considered.

(See Schwartz 10th ed., p. 1282.)

280
Q

Which of the following INCORRECTLY matches a grading scale for patients with liver disease and one of its components?

A. MELD: Serum creatinine
B. CTP: Bilirubin
C. CTP: INR
D. MELD: Albumin

A

Answer: D

The Child- Turcotte-Pugh (CTP) was initially derived for use in predicting the risk of portocaval shunt procedures, and comprises five components: bilirubin, albumin, INR, presence of encephalopathy, presence of ascites.

The Model or End-Stage Liver Disease (MELD) score was developed as a model to predict mortality after TIPS, but has been adapted and validated for use as the method of organ allocation for orthotopic liver transplantation (OLT) in the United States.

It is a linear regression model based on the serum creatinine, total bilirubin, and INR.

(See Schwartz 10th ed., p. 1280.)

281
Q

Which of the following INCORRECTLY pairs the CTP class with overall risk of mortality following an intra- abdominal operation?

A. Class A: 10%
B. Class B: 30%
C. Class C: 50%
D. Class C: 75%

A

Answer: C
The overall mortality for patients with cirrhosis undergoing intra-abdominal surgery has consistently been shown to correlate with the CTP classification.

The estimated mortality is 10%, 30% and 75 to 80% for those with CTP class A, B, and C cirrhosis, respectively.

MELD score also predicts postoperative mortality in cirrhotic patients, and has been shown to correlate well with estimates based on CTP classification. In general, those patients with a MELD below 10 should be considered appropriate for surgery, while those with scores above 15 should not be considered or elective procedures. (See Schwartz 10th ed., p. 1280.)

282
Q

What is the most common complication following TIPS?

A. Encephalopathy
B. Hepatic ischemia
C. Infection
D. Life-threatening hemorrhage

A

Answer: A

TIPS is a percutaneous procedure used for treatment of patients who have gastroesophageal varices in the setting of portal hypertension.

It has largely replaced surgical portosystemic shunts due to the act that it is both safe and effective while also providing a minimally invasive alternative to major abdominal surgery.

TIPS functions by creating an intrahepatic shunt between the portal and systemic circulation which causes a reduction in the portal pressure and ultimately in the blood flow through varices.

It is accomplished by endovascular access through the jugular vein to a hepatic vein radical and subsequent creation of a needle tract that connects it to a branch of the portal vein. After dilation of the tract, a metallic stent is deployed to hold the new portosystemic connection open.

Because this shunt reduces first pass metabolism of the liver, the most common complication of TIPS is encephalopathy which occurs in 25 to 30% of patients.

Other complications such as hepatic ischemia, infection, renal failure, and hemorrhage may occur, but are rare.

(See Schwartz 10th ed., p. 1282.)

283
Q

Initial management of a pyogenic liver may include all of the following EXCEPT?

A. Treatment of the underlying cause

B. Broad-spectrum IV antibiotics

C. Surgical drainage and/or resection

D. Percutaneous fine-needle aspiration and culture

A

Answer: A

Pyogenic abscesses are the most common liver abscesses seen in the United States. Though traditionally a result of intra- abdominal infections such as appendicitis and diverticulitis, earlier diagnosis of these conditions in patients has reduced prevalence of these conditions as causes for pyogenic liver abscess.

Other etiologies include impaired biliary drainage, subacute bacterial endocarditis, dental work, infected indwelling catheters, or direct extension from abscesses related to inflammatory bowel disease.

Pyogenic liver abscesses are most commonly seen in the right lobe of the liver, and Escherichia coli is the most commonly isolated pathogen.

Approximately 40% of abscesses are polymicrobial, while 20% of culture negative.

Treatment of pyogenic liver abscesses include correction of the underlying cause and intravenous antibiotics for at least 8 weeks, which is effective in approximately 80 to 90% of patients.

Empiric antibiotic coverage should include gram-negative and anaerobic organisms, with percutaneous aspiration and culture used to tailor long-term antibiotic therapy.

Placement of a percutaneous drainage catheter may be considered, though it is often ineffective due to the viscous nature of the collection.

Surgical drainage or resection is reserved for patients who fail nonoperative management.

(See Schwartz 10th ed., p. 1284.)

284
Q

The most common benign hepatic lesion is the

A. Hemangioma

B. Simple cyst

C. Adenoma

D. Bile duct hamartoma

A

Answer: B

While hemangiomas are the most common solid benign masses found in the liver, the simple hepatic cyst is still the most common overall.

Simple cysts have a prevalence of approximately 2.8 to 3.6%, and are more common in women by a ratio o 4:1.

Cysts are generally found incidentally during abdominal imaging, and small, asymptomatic cysts may be managed conservatively.

Large cysts may begin to cause abdominal pain, epigastric fullness, and early satiety.

These patients may be treated with percutaneous cyst aspiration and sclerotherapy which is effective in approximately 90% of patients. For those who fail percutaneous treatment, or where percutaneous treatment is not available, surgical cyst fenestration may be considered. If surgical fenestration is performed, the cyst wall should be sent for pathologic analysis to exclude carcinoma. (See Schwartz 10th ed., p. 1288.)

285
Q

Which of the following liver lesions carry a significant risk of spontaneous rupture?

A. Hemangioma

B. Hepatic cyst

C. Adenoma

D. Bile duct hamartoma

A

Answer: C

Hemangiomas are congenital vascular lesions that may range in size from less than 1 to 25 cm or greater. They are predominantly found in women, and are generally asymptomatic.

Large lesions may result in discomfort from compression of nearby organs. Though hemangiomas are at risk for bleeding if they are biopsied, spontaneous rupture is rare.

Adenomas, on the other hand, carry a significant risk for spontaneous rupture with intraperitoneal bleeding.

For this reason, along with their potential for malignant degeneration, it is generally recommended that hepatic adenomas be resected once discovered.

(See Schwartz 10th ed., pp. 1290–1291.)

286
Q

A patient presents with results from a CT scan that describe a well-circumscribed lesion that demonstrates homogenous enhancement during arterial phase, isodensity on the venous phase, and a central scar. In general, what would be the recommended treatment?

A. Reassurance and observation

B. Percutaneous radio frequency ablation

C. Resection

D. Transarterial chemoembolization

A

Answer: A

On contrast-enhanced imaging, a focal nodular hyperplasia (FNH) can be recognized as a well-circumscribed mass that demonstrates enhancement on the arterial phase and isodensity on the venous phase.

FNH also demonstrates a characteristic central scar.

FNH are solid benign lesions, are similar to adenomas, and are more common in women of child- bearing age.

Unlike adenomas, however, they are not prone to malignant degeneration or spontaneous rupture.

For this reason, asymptomatic FNHs may be managed conservatively unless adenoma or HCC cannot be definitively excluded.

Gadolinium-enhanced MRI may allow better visualization of the fibrous septa extending from the FNH’s central scar.

While FNH and adenomas may appear similar on CT or standard MRI, new MRI contrast agents, such as gadobenate dimeglumine (MultiHance), allow superior discrimination between these two lesions.

(See Schwartz 10th ed., p. 1291.)

287
Q

What is the annual conversion rate to HCC for patients with cirrhosis?

A. Less than 1%
B. 1–2%
C. 2–6%
D. 6–10%

A

Answer: C

HCC is the fifth most common malignancy worldwide, and its risk factors include viral hepatitis, alcoholic cirrhosis, hemochromatosis, and NASH.

Cirrhosis is present in 70 to 90% of patients who develop HCC, and the annual conversion rate from cirrhosis is 2 to 6%.

(See Schwartz 10th ed., p. 1291.)

288
Q

Patient’s eligible for the Mayo Clinical protocol to treat hilar cholangiocarcinoma do NOT include

A. Patients with hilar cholangiocarcinoma and primary sclerosing cholangitis (PSC).

B. Patients with unresectable cholangiocarcinoma.

C. Patients with tumors less than 3 cm.

D. Patients who have had prior radiotherapy.

A

Answer: D

Cholangiocarcinoma is an adenocarcinoma of the bile ducts, and represents the second most common primary liver malignancy.

Cholangiocarcinoma may be intra- or extrahepatic, and the latter may be divided into proximal or distal.

Proximal cholangiocarcinoma is also known as hilar cholangiocarcinoma or Klatskin tumor. The only curative treatment option for hilar cholangiocarcinoma is surgical resection, for which the reported 5-year survival rates range from 25 to 40%.

However, in the presence of primary sclerosing cholangitis (PSC, ~10% of patients with cholangiocarcinoma), the results of surgical resection are poor due to associated liver dysfunction and portal hypertension.

For this reason, the Mayo Clinic protocol was developed to treat patients with hilar cholangiocarcinoma and PSC.

This treatment comprises external beam radiation, 5-FU-based chemotherapy, and iridium-192 brachytherapy followed by operative staging and OL in patients without metastatic disease.

The 5-year survival rate for patients completing this protocol is 70%. Current eligibility criteria for this protocol include patients with hilar cholangiocarcinoma with PSC or patients with unresectable hilar cholangiocarcinoma who have not received prior radiotherapy.

Furthermore, the patient must have a primary tumor less than 3 cm in radial dimension and no evidence of intrahepatic or extrahepatic metastases.

(See Schwartz 10th ed., pp. 1291–1292.)

289
Q

Which of the following is considered a primary determinant of suitability for resection when evaluating a patient with hepatic colorectal metastases?

A. Number of metastatic tumors
B. Size of metastatic tumors
C. Predicted volume of hepatic remnant
D. Prior therapy

A

Answer: C

The liver is a common site for metastatic disease in patients with colorectal disease, and approximately 50 to 60% of patients diagnosed with colorectal cancer will develop liver metastases within their lifetime.

With the advent of more aggressive strategies or the management of metastatic colorectal cancer, including improved chemotherapeutic regimens and expanded use of metastasectomy, the 5-year survival or patients with isolated metastases to the liver may exceed 30%.

Given these encouraging results, the paradigm for surgical evaluation and treatment of these patients has shifted to primarily consider the health of the background liver and volume of the hepatic remnant, and not tumor characteristics such as size and number.

(See Schwartz 10th ed., pp. 1293–1294.)

290
Q

Based on the standard Milan criteria, which of the following patients with HCC would be eligible for transplantation?

A. One 4.5-cm lesion in segment VI with invasion of the right portal vein.

B. Three lesions confined to the right lobe, with the largest being 2.5 cm.

C. A single, 5.5-cm lesion in segment II.

D. Three lesions spread throughout the liver, with the largest being 3.5 cm.

A

Answer: B

OTL was first attempted in the 1980s and 1990s, with initial series reporting 5-year survival rates of 20 to 50%.

This led to the introduction of the Milan criteria which limited eligibility to patients with one tumor less than 5 cm or up to three tumors less than 3 cm and no evidence of gross intravascular or extrahepatic spread.

Adoption of these guidelines resulted in significant improvement in 5-year survival or patients with HCC treated with OTL.

(See Schwartz 10th ed., p. 1295.)

291
Q

The only FDA-approved systemic chemotherapeutic agent for HCC is

A. Epirubicin
B. Cisplatin
C. 5-Fluorouracil
D. Sorafenib

A

Answer: D

Though systemic chemotherapy has not proven very effective in the treatment of HCC, the multikinase inhibitor sorafenib has been approved for use specifically in these patients.

Based on results of the SHARP trial, the sorafenib demonstrated a 3-month survival bene it versus placebo.

Though these results are modest, it remains a treatment option or patients with advanced, unresectable HCC.

(See Schwartz 10th ed., p. 1296.)

292
Q

Which of the following correctly pairs the Brisbane 2000 hepatic resection terminology with appropriate liver segments?

A. Right posterior sectionectomy: Segments IV and IV
B. Left hepatectomy: Segments I, II, III, and IV
C. Right Hepatectomy: Segments VI, VII, and VIII
D. Left lateral sectionectomy: Segments II and III

A

Answer: D

293
Q

A 40-year-old female undergoes a lap cholecystectomy and is found to have large CBD stones at the time of the operation. A CBD exploration was performed. After the stones were removed, the ductotomy was closed over a t-tube. She now presents two weeks postoperatively to your office. A follow-up cholangiography confirms the presence of a retained CBD stone. What is the next BEST step of management of this patient?

A. Endoscopic retrograde cholangiopancreatography (ERCP) via the T-tube to extract the stone

B. Choledochoscopy to extract the stone

C. Clamp T-tube and remove it

D. Wait an additional 2-4 weeks prior to any instrumentation

E. Open common bile duct exploration

A

D. Wait an additional 2-4 weeks prior to any instrumentation

294
Q

A 63-year-old male presents to the ER after several weeks of abdominal pain and malaise. The patient appears jaundiced on exam. A CT scan is performed that shows a mass in the biliary tree consistent with cholangiocarcinoma. Which of the following is true concerning this diagnosis?

A. Type I tumors involve the CBD

B. Primary biliary cirrhosis is a factor

C. Chemotherapy has significant survival benefit in unresectable disease

D. Type III tumors involve the secondary hepatic ducts

E. The major prognostic factor is the location of the tumor

A

D. Type III tumors involve the secondary hepatic ducts

BISMUTH-CORLETTE
Type I: Tumor below the confluence

Type II: Tumor reaching the confluence

Type IIIA: Tumor occluding the CHD + right hepatic duct

Type IIIB: Tumor occluding the CHD + left hepatic duct

Type IVa: Tumor involving the confluence and both ducts

Type IVb: Multicentric involvement of tumors

295
Q

A 61-year-old male comes to the emergency room complaining of persistent abdominal pain over the past week. On exam, the patient is jaundiced and has a palpable mass at his RUQ. CT scan demonstrates a dilated, cystic structure associated with the biliary tree. Which of the folllowing is true regarding this disease?

A. Type I cysts can be treated with drainage and cystectomy

B. Type II cysts are fusiform dilatations of the CBD

C. Type III cysts require cholecystectomy, cyst resection and hepaticojejunostomy

D. Type IV cysts may require a liver resection

E. Type V cysts have both intrahepatic and extrahepatic involvement

A

D. Type IV cysts may require a liver resection

296
Q

An 88-year-old female presents from the nursing home with nausea, vomiting, and abdominal pain. One exam, she has abdominal distension and mild diffuse abdominal tenderness. An abdominal x-ray shows air in the biliary tree and air fluid levels. She has a history of a hysterectomy in the past. What is the MOST likely etiology?

A. Fistula between the gallbladder and duodenum

B. SBO caused by adhesions

C. Sigmoid volvulus

D. Fistula between gallbladder and jejunum

E. Ascending cholangitis

A

A. Fistula between the gallbladder and duodenum

297
Q

A 43-year-old female comes to your office for a follow-up visit 6 months after a laparoscopic cholecystectomy for symptomatic gallstones complaining of right upper quadrant pain, similar to what she experienced prior to surgery. On exam, the patient is jaundiced and has tenderness to palpation in the right upper quadrant. An ultrasound shows a dilated CBD, but no fluid in the RUQ. What is the next best step in management?

A. ERCP for removal of recurrent stone in the CBD

B. ERCP for removal of retained stone in the CBD

C. ERCP for stenting of cystic duct leak

D. HIDA scan

E. Re-exploration for possible CBD injury

A

B. ERCP for removal of retained stone in the CBD

298
Q

A 95-year-old female with a history of rheumatoid arthritis on high dose prednisone and known history of congestive heart failure with an ejection fraction of 15% has been in the medical intensive care unit for 10 days with ventilator-dependent respirator failure following CAP. An abdominal sonogram was obtained during workup for fever of unknown origin, and is consistent with cholecystomegaly, thickening of the GB wall, and pericholecystic fluid. Which of the following would represent the BEST management strategy?

A. Continued observation

B. Initiation of IV antibiotics and expectant management

C. Urgent laparoscopic cholecystectomy

D. Urgent open cholecystectomy

E. Insertion of a percutaneous cholecystostomy tube for decompression

A

E. Insertion of a percutaneous cholecystostomy tube for decompression

299
Q

6 days s/p lap cholecystectomy, a 44-year-old female complains of RUQ pain, fever and chills. Her bilirubin is 4.1. Which of the following is true regarding bile leaks after lap cholecystectomy?

A. Surgical correction is the recommended management

B. Most common etiology is technical

C. Operative drainage is preferred method for draining biloma

D. Duct of Luschka leak should be managed differently from cystic duct leak

E. Most leaks will resolve with endoscopic stenting

A

E. Most leaks will resolve with endoscopic stenting

300
Q

A 68-year-old male with a history of multiple episodes of pancreatitis presents to the ER complaining of fevers and abdominal pain. On exam the patient is febrile, jaundiced, and has right upper quadrant tenderness. His VS are stable, but he has an elevated WBC. An UTS shows no signs of cholelithiasis and a CBD that measures 10mm. What is the next best step in management?

A. NPO with NG tube placement

B. CT scan of abdomen and pelvis with contrast

C. ERCP

D. MRCP

E. Laparoscopic cholecystectomy with IOC

A

C. ERCP

301
Q

A patient with a history of chronic cholecystitis is taken to the OR for a lap cholecystectomy. During the operation, there appears to be an aberrant duct that comes off of the cystic duct. Which of the following best describes the next best option in management?

A. Convert to an open cholecystectomy

B. Perform an intraoperative cholangiogram

C. Abort the operation and preform an MRCP

D. Place the clip distal to the aberrant duct and proceed with operation

E. Place clip proximal to the aberrant duct and proceed with the operation

A

B. Perform an intraoperative cholangiogram

302
Q

A 42-year-old male presents to the ER complaining of hematemesis, jaundice and abdominal pain. The patient recently had a motor vehicle accident and was found to have a hepatic hematoma that was treated nonoperatively. The patients BP is 84/52mmHg and his pulse is 115bpm. Crystalloid resuscitation has been initiated. What is the next best step?

A. Upper GI endoscopy

B. CT scan of abdomen and pelvis

C. ERCP with stent placement

D. Angiography

E. Laparoscopic cholecystectomy

F. OR for emergent laparotomy

A

F. OR for emergent laparotomy

303
Q

A 51-year old male presents to the ER complaining of severe abdominal pain that radiates to the back. Lab data shows an elevated amylase and lipase. On UTS, the patient is found to have cholelithiasis and a dilated CBD. What is the best course of management for the patient?

A. NPO, NG tube, IV antibiotics

B. Lap cholecystectomy

C. Lap chole with IOC

D. ERCP with stone extraction and lap chole during same admission

E. ERCP with stone extraction and lap chole in 6 weeks

A

D. ERCP with stone extraction and lap chole during same admission

304
Q

A 63-year-old male presents to the ER with complaints of blood in his stool. The patient has an EGD performed that shows isolated gastric varices. Splenic vein thrombosis is suspected. Which of the ff is TRUE concerning this disease?

A. It is associated with pancreatic cancer.

B. Caused by retrograde blood flow through the left gastric vein.

C. Bleeding is a complication in the majority of patients.

D. Asymptomatic patients benefit from a prophylactic splenectomy

E. The patient should have an abdominal UTZ.

A

E. The patient should have an abdominal UTZ

305
Q

A 67-year-old male with a history of colon cancer resected two years ago presents with metastatic disease to the liver. He is being considered for chemoembolization. What is the rationale for chemoembolization in hepatic mets?

A. Despite the higher incidence of side effects, the response rates are better than systemic chemotherapy.

B. Hepatic mets derive their blood supply from the hepatic artery.

C. It allows high local concentrations with hepatic extraction rates of 75%

D. It has better rates of survival and less morbidity than surgical resection.

E. It can potentially cure patients of disease.

A

B. Hepatic mets derive their blood supply from the hepatic artery.

306
Q

A 52-year old male was recently diagnosed with early stage hepatocellular cancer. Which is a contraindication for hepatic transplantation in this patient?

A. Hepatic encepalopathy

B. Refractory ascites

C. Single tumor with a size of 4cm

D. Two tumor sites measuring 2cm and 4cm

E. Three tumor site measuring 2cm in size each

A

D. Two tumor sites measuring 2cm and 4cm

307
Q

Which of the following is the best test of liver function?

A. Alanine aminotransferase (ALT)

B. Aspartate aminotransferase (AST)

C. Alkaline phosphatase

D. Prothrombin time

A

D. Prothrombin time

308
Q

Which of the following concerning a pyogenic abscess is TRUE:

A. The most common etiology is from ineffective treatment of abdominal infections

B. Most abscesses are found in the left liver

C. The most common pathogen is Bacteroides

D. Abnormal liver function tests are more commonly seen in a pyogenic abscess compared to an amebic abscess

E. CXR and ultrasound have similar sensitivities for diagnosis

A

D. Abnormal liver function tests are more commonly seen in a pyogenic abscess compared to an amebic abscess

309
Q

Which of the following is true?

A. Obstruction of the liver’s blood supply can lead to formation of hemorrhoids

B. Caudate lobe is directly enclosing the main portal vein

C. Liver dysfunction does not alter the bacteriologic flora of the small bowels

D. The liver’s left lateral segment consists of segments 3 and 4

A

A. Obstruction of the liver’s blood supply can lead to formation of hemorrhoids

310
Q

A 50-year-old female with chronic active hepatitis B presented with 1 month history of jaundice accompanied by abdominal distension and sudden onset of upper GI bleeding. An abdominal ultrasound revealed massive ascites, coarsened liver surface and small Liver. Which of the ff is true?

A. The patient is high risk for developing renal failure

B. Recurrence of the upper GI bleed is best prevented by surgery (devascularization procedures)

C. The ascites is best managed by giving Furosemide and albumin

D. An ERCP should be performed to rule out obstructive jaundice

A

A. The patient is high risk for developing renal failure

311
Q

A 35yo female with intermittent right upper quadrant pain for 2 months came to the ER because of jaundice and fever. PE revealed that the patient had an altered sensorium with yellowish discoloration of the skin and sclera. BP was 80/40 with a mean arterial pressure of 50. Heart rate was 110bpm. A previous UTZ showed dilated intrahepatic and proximal bile duct. The distal CBD was not visualized due to bowel gas. Which of the ff is true?

A. Highly potent broad spectrum antibiotics such as clindamycin should be started immediately

B. An abdominal CT scan should be performed immediately prior to treatment to determine the definite diagnosis

C. ERCP and/or PTBD is necessary

D. Surgery is the best approach to treat the patient since it has the least complication

A

C. ERCP and/or PTBD is necessary

312
Q

A 45-year-old male presented with intermittent RUQ pain accompanied by jaundice, weight loss, anorexia and vomiting. A CT scan revealed a mass measuring 8x5cm occupying the periampullary region with 4 hypodense nodules In the right and left lobes of the liver. An EGD revealed a fungating mass completely obstructing the second part of the duodenum. Biopsy revealed adenocarcinoma. Which of the ff is true?

A. Chemotherapy is needed to downsize the tumor prior to surgery

B. Surgery is required even with the presence of a Liver metastasis

C. Chemotherapy and Radiotherapy is the primary treatment of choice for this patient

D. ERCP and PTBD are needed to palliate symptoms

A

B. Surgery is required even with the presence of a Liver metastasis

313
Q

A 40-year-old male with 5cm HCC in his right lobe presented with the ff: moderate ascites, 2.5mg/dL of bilirubin, 2.7g/dL of albumin, Prothrombin time is 4 seconds longer, no encephalopathy, ALT of 100 and normal creatinine.

A. Child score is 9 and would require hepatic resection.

B. Child score is 8 and would require transarterial embolization.

C. Child score is 10 and would require Liver transplantation

D. Child score is 11 and would require Liver transplantation

A

B. Child score is 8 and would require transarterial embolization.

314
Q

True about hepatorenal syndrome:

A. Type 1 is the chronic type whereas type 2 is rapidly progressing

B. Type 1 has poorer prognosis compared to type 2

C. Hepatorenal syndrome results in renal arterial vasodilation and opening of small intrarenal arteriovenous communications

D. Hepatorenal syndrome is frequently noted in alcoholic patients with fatty liver.

A

B. Type 1 has poorer prognosis compared to type 2

Hepatorenal syndrome (HRS) is the development of renal failure in patients with advanced chronic liver disease [1] and, occasionally, fulminant hepatitis, who have portal hypertension and ascites.

Estimates indicate that at least 40% of patients with cirrhosis and ascites will develop HRS during the natural history of their disease.

The hallmark of HRS is renal vasoconstriction, although the pathogenesis is not fully understood. Multiple mechanisms are probably involved and include an interplay between disturbances in systemic hemodynamics, activation of the vasoconstrictor systems, and a reduction in the activity of the vasodilator systems.

The hemodynamic pattern of patients with HRS is characterized by increased cardiac output, low arterial pressure, and reduced systemic vascular resistance.

Renal vasoconstriction occurs in the absence of reduced cardiac output and blood volume, which is in contrast to most clinical conditions associated with renal hypoperfusion.

Type 1: Cirrhosis with rapidly progressive acute renal failure. (ARF)

Type 2: Cirrhosis with subacute renal failure. (Refractory ascites, gradual and progressive)

Type 3: Cirrhosis with types 1 or 2 HRS superimposed on chronic kidney disease or acute renal injury.

Type 4: Fulminant liver failure with HRS.

315
Q

A 65yo male presents with intermittent episodes of bowel obstruction. PMH revealed a recurrent tolerable RUQ pain and occasional fever. The patient was previously diagnosed with calculous cholecystitis. PE revealed a Slightly distended nontender abdomen with hypoactive bowel sounds. Abdominal xray revealed pneumobilia, dilated small bowels with a hyperdense rounded structure at the hypogastric region.

A. Colonoscopy is necessary to establish the diagnosis.

B. Metastatic workup is required.

C. The disease predominates in elderly females

D. Surgical treatment would include enteroromy and the creation of a roux-en-y hepaticojejunostomy.

A

C. The disease predominates in elderly females

316
Q

A 55-year-old patient with obstructive jaundice due to hilar cholangiocarcinoma with 3 bilobar metastatic lesions In the liver measuring 3cm each. Treatment would include:

A. Liver transplant to remove both the primary lesion as well as the metastatic nodules

B. Resection of the primary tumor as well as enucleation of the metastatic lesions.

C. Percutaneous transhepatic biliary drainage.

D. Transarterial embolization

A

C. Percutaneous transhepatic biliary drainage

317
Q

A 56 year old male presented with jaundice. CT showed a pancreatic head mass measuring 4.5cm partially encroaching a segment of the portal vein. The celiac axis is free of tumor. Biopsy of the mass revealed a pancreatic adenocarcinoma. The patient is fit enough to go for surgery. True regarding treatment:

A. The patient needs PTHBD prior to whipples to reduce the risk of complications

B. Chemoradiotherapy is the treatment of choice

C. The patient needs to undergo whipples resection coupled with portal vein reconstruction to improve survival

D. Whipples resection will not offer any survival benefit because of portal vein involvement

A

C. The patient needs to undergo whipples resection coupled with portal vein reconstruction to improve survival

318
Q

A 27 year old female presenting with jaundice and a palpable abdominal mass. CT revealed a mass at the head of the pancreas with solid and cystic component. There was also a 5cm mass occupying segment 6 of the Liver. Biopsy of the Liver mass revealed solid and pseudo papillary tumor of the pancreas. True regarding treatment:

A. Because of the liver mets, only whipples resection should be done to palliate the jaundice. Resection of Liver mass not needed

B. Because of the Liver mets, the patient only needs percutaneous biliary drainage to palliate the jaundice.

C. Chemoradiation is needed

D. The patient should undergo whipples resection together with Liver resection to provide long survival

A

D. The patient should undergo whipples resection together with Liver resection to provide long survival

319
Q

A 55 year old female presented with vague RUQ pain. Abdominal CT scan revealed a complex cystic mass occupying the right lobe of the liver measuring 6cm in diameter. The cyst was further described as multilocular with enhancing wall and papillary projections. Aspiration biopsy of the fluid revealed cystadenoma. True regarding treatment?

A. Cystadenoma can be treated by cyst aspiration followed by sclerotherapy to relieve the discomfort

B. Cystadenoma can be treated by creation of cystjejunostomy for internal drainage of the cyst content

C. Cystadenoma should be treated by unroofing of the cystic wall to prevent recurrence

D. Cystadenoma can be treated with hepatic resection

A

D. Cystadenoma can be treated with hepatic resection

320
Q

A 48 year old male is undergoing liver transplantation. Which of the following has emerged as a useful tool for estimating the mortality of patients awaiting liver transplantation?

A. Model for End Stage Liver Disease

B. Child Turcotte Pugh

C. UCSF modification

D. Milan

A

A. Model for End Stage Liver Disease

321
Q

UNOS Criteria does not include:

A. Tumor of more than 5cm

B. No macrovascular involvement

C. 2 tumors 2.5cm each

D. No macrovascular involvement

A

A. Tumor of more than 5cm

322
Q

A cirrhotic patient was noted to have 3cm segment V mass on ultrasound. Triphasic CT scan showed hepatic arterial enhancement and venous phase washout. You would

A. Observe

B. Assess patient for comorbidity, liver reserve and staging for possible resection +/- transplant

C. Do core biopsy if 2nd type of contrast study shows 0 or 1 classic enhancement

D. Do repeat imaging in 3 months

A

B. Assess patient for comorbidity, liver reserve and staging for possible resection +/- transplant

323
Q

A 58 year old patient who has cirrhosis and liver cancer is being contemplated for hepatic resection. His future liver remnant (FLR) volume can be assessed with ____ volumetry.

A. Ultrasound

B. CT

C. MRI

D. Operative

A

B. CT

324
Q

If MS has an sFLR of 20%, he is best treated by:

A. Supportive treatment only as he will not tolerate any surgery

B. Resection only, sFLR is acceptable to this patient

C. Portal vein embolization prior to resection

D. Resection and transplantation

A

C. Portal vein embolization prior to resection

325
Q

A patient was admitted due to hematemesis. He underwent endoscopy and had variceal ligation. He also underwent TIPS. This procedure might aggravate:

A. Ascites

B. Recurrence of variceal bleeding

C. Portal hypertension

D. Hepatic encepalopathy

A

D. Hepatic encepalopathy

326
Q

Test for determining true liver function:

A. Serum bilirubin

B. Alkaline phosphatase

C. Indocyanine Green clearance

D. AST and ALT

A

C. Indocyanine Green clearance?

327
Q

A 45-year-old male underwent laparoscopic cholecystectomy. On histopath, she was diagnosed to have adenocarcinoma invading the muscularis propria. Treatment of choice?

A. Observation

B. Port site resection

C. Do hepatic resection with negative margins and portal lymphadenectomy

D. Do extended right hepatectomy with portal lymphadenectomy

A

C. Do hepatic resection with negative margins and portal lymphadenectomy

328
Q

One of the following is NOT true for hepatic adenomas:

A. Hepatic adenomas have a risk of malignant transformation

B. 90% of patients who developed hepatic adenomas are OCP users

C. Triphasic contrast usually displays arterial enhancement and washout on delayed venous phase

D. Liver function tests, CEA and AFP are usually normal in patients with Has

A

C. Triphasic contrast usually displays arterial enhancement and washout on delayed venous phase

329
Q

The involvement of the portal vein or SMV by pancreatic carcinoma renders the patient at least

A. Resectable

B. Borderline resectable

C. Unresectable

D. None of the above

A

B. Borderline resectable

330
Q

A 55 year old female presented with chronic RUQ and low grade fever. Initial workup revealed an elevated liver enzymes and alkaline phosphatase. Ultrasound showed 2cm stone in the GB neck and a normal sized CBD. What is the best management for this patient?

A. Schedule for ERCP followed by lap cholecystectomy

B. Schedule for MRCP

C. Schedule for open cholecystectomy with IOC

D. Schedule for laparoscopic cholecystectomy with choledochoscopy

A

A. Schedule for ERCP followed by lap cholecystectomy

331
Q

A 65-year-old male presented with a history of yellowish discoloration of the skin 2 weeks prior to consult. Other symptoms noted was occasional feeling of bloatedness. Initial workup revealed an elevated liver enzymes and alkaline phosphatase, ultrasound showed markedly distended GB with a dilated CBD. PE: Positive for jaundice and icteresia, a palpable mass in the RUQ, with no tenderness upon palpation. What is the best management for this patient?

A. Sched for ERCP followed by lap cholecystectomy

B. Sched for MRCP

C. Sched for open cholecystectomy with IOC

D. Sched for laparoscopic cholecystectomy with choledochoscopy

A

B. Sched for MRCP

332
Q

A 45-year-old male who underwent a difficult laparoscopic cholecystectomy a week ago came in for follow up complaining of abdominal pain and febrile episodes. Initial exam showed a jaundiced patient with tenderness over the RUQ. Patient had UTZ showing a collection of peri hepatic fluid. PE: +slight jaundice. No abdominal tenderness. What is the best management for this patient?

A. Exploratory laparotomy and evacuation of the fluid collection

B. Schedule for a Diagnostic Laparoscopy

C. Do percutaneous drainage and ERCP

D. Schedule for MRCP

A

C. Do percutaneous drainage and ERCP

333
Q

A 67 year old male who initially underwent ERCP and stone clearance for primary CBD stones came in due to frequent episodes of RUQ pain and fever with associated yellowish discoloration of the skin. Patient claimed several episodes of RUQ pain. Initial ultrasound showed a 6cm hypoechoic mass with surrounding edema in segment 5 of the liver. Additional diagnostic workup were unremarkable. What is the most likely diagnosis?

A. Amoebic liver abscess

B. Acute cholangitis

C. Pyogenic liver abscess

D. Hepatocellular carcinoma

What is the most appropriate management for the above patient?

A. Antimicrobial coverage and repeat ERCP

B. Antimicrobial coverage + percutaneous catheter drainage of abscess

C. Percutaneous drainage only

D. Antimicrobial coverage with CT guided needle aspiration of abscess

A

C. Pyogenic liver abscess

——

B. Antimicrobial coverage + percutaneous catheter drainage of abscess

334
Q

Which of the following patient would least likely benefit from a prophylactic/incidental cholecystectomy?

A. Patient with long standing diabetes mellitus with gallstones

B. Morbidly obese patients undergoing bariatric surgery with asymptomatic GB stones

C. 65 year old patients with porcelain gallbladder

D. Heart transplant patient with asymptomatic gallstones

A

A. Patient with long standing diabetes mellitus with gallstones

335
Q

The arterial supply of the CBD is derived from

A. The left hepatic artery

B. The right hepatic artery

C. The gastroduodenal artery

D. The right hepatic and gastroduodenal arteries

E. The left hepatic and gastroduodenal arteries

A

D. The right hepatic and gastroduodenal arteries

The majority of the blood flow to the human CBD originated from the right hepatic artery and gastroduodenal arteries, with major trunks running along the medial and lateral aspects of the common duct (3 and 9 o’clock positions).

336
Q

Anomalies of the hepatic artery and cystic artery are present in what percent of individuals?

A. 15%

B. 25%

C. 35%

D. 50%

E. 75%

A

Answer D

Variations in the anatomy of the cystic and hepatic arteries are exceedingly common, the classical anatomy only appearing in 50-60% of the population.

The cystic artery is a branch of the right hepatic artery in 90% of individuals. The most common arterial anomaly of the portal arterial system is a replaced right hepatic artery originating from the superior mesenteric artery; this happens in 20% of persons.

337
Q

The treatment of choice for a type I choledochal cyst is

A. Observation

B. Cyst resection and primary reanastomosis of the CBD

C. Resection of the CBD, cholecystectomy, and hepaticojejunostomy

D. Resection of the cyst and choledocho-duodenostomy.

A

Answer C

Choledochal cysts are rare congenital cystic dilatations of the extrahepatic and I intrahepatic biliary tree. Females are affected 3 to 8 times more commonly than men. Though they are commonly diagnosed in childhood, as many as one half of patients are not diagnosed until adulthood.

The most common presentations in adulthood are jaundice and cholangitis, and less than one-half of patients present with the classic clinical triad of abdominal pain, jaundice and a mass.

Ultrasonography or CT scanning will confirm the diagnosis, but a more definitive imaging technique such as ERCP, PTC, or MRCP is required to assess the biliary anatomy and plan the appropriate surgical treatment.

The risk of cancer development in these patients is up to 15% and can largely be mitigated by excision of the biliary tree.

Types I, II and IV cysts are treated with excision of the extrahepatic biliary tree with a roux-en-y hepaticojejunostomy.

Type IV may also require a liver resection.

Sphincterotomy is recommended for type III cysts.

338
Q

Relaxation of the sphincter if Oddi in response to a meal is largely under the control of which hormone?

A. Gastrin

B. CCK

C. Motilin

D. Secretin

E. Ghrelin

A

Answer B

The sphincter of Oddi is a complex structure that is functionally independent from the duodenal musculature and creates a high pressure zone between the bile duct and the duodenum.

The sphincter of Oddi is about 4 to 6mm in length and has a basal resting pressure of 13mmHg above the duodenal pressure.

On manometry, the sphincter shows phasic contractions with a frequency of about 4/min and an amplitude of 12 to 140mmHg.

The spontaneous motility of the sphincter of Oddi is regulated by the interstitial cells of Cajal through intrinsic and extrinsic inputs from hormones and neurons acting on the smooth muscle cells.

Relaxation occurs with a rise in CCK, leading to diminished amplitude of phasic contractions and reduced nasal pressure, allowing increased flow of bile into the duodenum.

During fasting, the sphincter of Oddi activity is coordinated with the periodic partial GB emptying and an increase in bile flow that occurs during phase II of the migrating myoelectric motor complexes.

339
Q

What percentage of the bile acid pool is reabsorbed in the ileum through the enterohepatic circulation?

A. 25%
B. 50%
C. 75%
D. 90%
E. 95%
A

E

Bile is mainly composed of water, electrolytes, bile salts, proteins, lipids, and bile pigments. Sodium, potassium, calcium and chlorine have the same concentration in bile as in plasma or extracellular fluid.

The pH of hepatic bile is usually neutral or slightly alkaline, but varies with diet; an increase in protein shifts the bile to a more acidic pH.

The primary bile salts, cholate and chenodeoxycholate, are synthesized in the liver from cholesterol. They are conjugated there with taurine and glycine and act within the bile as anions (bile acids) that are balanced by sodium.

Bile salts are excreted into the bile by the hepatocyte and aid in the digestion and absorption of fats in the intestines. In the intestines, about 80% of the conjugated bile acids are absorbed in the terminal ileum.

The remainder is dehydroxylated (deconjugated) by gut bacteria, forming secondary bile acids deoxycholate and lithocholate. These are absorbed in the colon, transported to the liver, conjugated, and secreted into the bile.

Eventually, about 95% of the bile acid pool is reabsorbed and returned via the portal venous system to the liver, the so-called enterohepatic circulation.

Five percent is excreted in the stool, leaving the relatively small amount of bile acids to have maximum effect.

340
Q

Three parts of the caudate lobe?

A

Spiegel lobe
Paracaval portion
Caudate process

341
Q

Venous drainage of the Liver?

A

3 hepatic veins pass obliquely through the Liver to drain the blood to the suprahepatic IVC and eventually the right atrium.

Right hepatic vein: Drains segments V to VIII

Left hepatic vein: Drains segments II and III

Middle hepatic vein: Drains segments IV, V, VIII

IVC: Drains caudate lobe (I)

342
Q

What is a common source of referred pain to the right shoulder and scapula as well as right side or back, usually stimulated by tumors that stretch Glisson’s capsule or by diaphragmatic irritation?

A

Right phrenic nerve

343
Q

What is a common source of referred pain to the right shoulder and scapula as well as right side or back, usually stimulated by tumors that stretch Glisson’s capsule or by diaphragmatic irritation?

A

Right phrenic nerve

344
Q

Which is more specific between AST and ALT?

A

Hepatocellular injury of the liver is usually indicated by abnormalities in levels of the liver aminotransferases AST and ALT.

These enzymes participate in gluconeogenesis by catalyzing the transfer of amino groups from aspartic acid or alanine to ketoglutaric acid to produce oxaloacetic acid and pyruvic acid, respectively (these enzymes are also referred to as serum glutamic-oxaloacetic transaminase [SGOT] and serum glutamicpyruvic transaminase [SGPT]).

AST is found in liver, cardiac muscle, skeletal muscle, kidney, brain, pancreas, lungs, and red blood cells and thus is less specific for disorders of the liver. ALT is predominately found in the liver and thus is more specific for liver disease.

Hepatocellular injury is the trigger for release of these enzymes into the circulation. Common causes of elevated aminotransferase levels include viral hepatitis, alcohol abuse, medications, genetic disorders (Wilson’s disease, hemochromatosis, α1-antitrypsin deficiency), and autoimmune diseases.

345
Q

What is alkaline phosphatase?

A

AP is an enzyme with a wide tissue distribution but is found primarily in the liver and bones.

In the liver, it is expressed by the bile duct epithelium.

In conditions of biliary obstruction, levels rise as a result of increased synthesis and release into the serum.

Because the half-life of serum AP is approximately 7 days, it may take several days for levels to normalize even after resolution of the biliary obstruction.

346
Q

What is GGT?

A

Gamma-glutamyltranspeptidase (GGT) is another enzyme found in hepatocytes and released from the bile duct epithelium.

Elevation of GGT is an early marker and also a sensitive test for hepatobiliary disease. Like AP elevation, however, it is nonspecific and can be produced by a variety of disorders in the absence of liver disease.

Increased levels of GGT can be induced by certain medications, alcohol abuse, pancreatic disease, myocardial infarction, renal failure, and obstructive pulmonary disease.

For this reason, elevated GGT levels are often interpreted in conjunction with other enzyme abnormalities. For example, a raised GGT level with increased AP level supports a liver source.

347
Q

When does hyperbilirubinemia become detectable as jaundice?

A

Jaundice refers to the yellowish staining of the skin, sclera, and mucous membranes with the pigment bilirubin.

Hyperbilirubinemia usually is detectable as jaundice when blood levels rise above 2.5 to 3 mg/dL.

348
Q

What are the 3 phases in which bilirubin metabolism can take place, and cause jaundice?

A

As mentioned previously, bilirubin metabolism can take place in three phases: prehepatic, intrahepatic, and posthepatic.

The prehepatic phase includes the production of bilirubin from the breakdown of heme products and its transport to the liver. The majority of the heme results from red blood cell metabolism and the rest from other heme-containing organic compounds such as myoglobin and cytochromes.

In the liver, the insoluble unconjugated bilirubin is then conjugated to glucuronic acid to allow for solubility in bile and excretion.

The posthepatic phase of bilirubin metabolism consists of excretion of soluble bilirubin through the biliary system into the duodenum.

Dysfunction in any of these phases can lead to jaundice.

349
Q

Causes of pre hepatic jaundice?

A

Causes of hemolysis include inherited and acquired hemolytic anemias.

Inherited hemolytic anemias include genetic disorders of the red blood cell membrane (hereditary spherocytosis and eliptocytosis), enzyme defects (glucose-6-phosphate dehydrogenase deficiency), and defects in hemoglobin structure (sickle cell anemia and thalassemias).

Hemolytic anemias can also be acquired, and these can be further divided into those with immune-mediated and those with non–immune-mediated causes.

  • Immune-mediated hemolytic anemias result in a positive finding on a direct Coombs test and have a variety of autoimmune and drug-induced causes. In contrast, direct Coombs test results are negative in nonimmune hemolytic anemias. The causes in this latter category are varied and include drugs and toxins that directly damage red blood cells, mechanical trauma (heart valves), microangiopathy, and infections.
  • Prehepatic dysfunction of bilirubin metabolism also can result from failure in the transport of unconjugated bilirubin to the liver by albumin in any condition that leads to plasma protein loss. A poor nutritional state or excess protein loss as seen in burn patients can lead to elevated levels of unconjugated bilirubin in the circulation and jaundice.
350
Q

Causes of intrahepatic jaundice?

A

Intrahepatic causes of jaundice involve the intracellular mechanisms for conjugation and excretion of bile from the hepatocyte.

The enzymatic processes in hepatocytes can be affected by any condition that impairs hepatic blood flow and subsequent function of the liver (ischemic or hypoxic events).

Furthermore, there are multiple inherited disorders of enzyme metabolism that can result in either unconjugated or conjugated hyperbilirubinemia. Gilbert’s syndrome is a genetic variant characterized by diminished activity of the enzyme glucuronyltransferase, which results in decreased conjugation of bilirubin to glucuronide. It is a benign condition that affects approximately 4% to 7% of the population. Typically, the disease results in transient mild increases in unconjugated bilirubin levels and jaundice during episodes of fasting, stress, or illness. These episodes are self-limited and usually do not require further treatment.

Another inherited disorder of bilirubin conjugation is Crigler-Najjar syndrome. It is a rare disease found in neonates and can result in neurotoxic sequelae from bilirubin encephalopathy.

In addition to defects in conjugation, disorders in bilirubin excretion in hepatocytes can also lead to jaundice. Rotor’s syndrome and Dubin-Johnson syndrome are two uncommon genetic disorders that disrupt secretion of conjugated bilirubin from the hepatocyte into the bile and result in conjugated hyperbilirubinemia.

There are also multiple acquired conditions that result in inflammation and intrahepatic cholestasis by affecting hepatocyte mechanisms for conjugation and excretion of bile. Viruses, alcohol abuse, sepsis, and autoimmune disorders all can result in inflammation in the liver with subsequent disruption of bilirubin transport in the liver.

In addition, jaundice can also occur from the cytotoxic effects of many medications, including acetaminophen, oral contraceptives, and anabolic steroids.

351
Q

Causes of post hepatic jaundice?

A

Posthepatic causes of jaundice are usually the result of intrinsic or extrinsic obstruction of the biliary duct system that prevents the flow of bile into the duodenum. There is a wide spectrum of pathologies that may present with obstructive jaundice.

Intrinsic obstruction can occur from biliary diseases, including cholelithiasis, choledocholithiasis, benign and malignant biliary strictures, cholangiocarcinoma, cholangitis, and disorders of the papilla of Vater.

Extrinsic compression of the biliary tree is commonly due to pancreatic disorders. Patients with pancreatitis, pseudocysts, and malignancies can present with jaundice due to external compression of the biliary system.

Finally, with the growing armamentarium of endoscopic tools and minimally invasive surgical approaches, surgical complications are becoming more frequent causes of extrahepatic cholestasis. Misadventures with surgical clips, retained stones, and inadvertent ischemic insults to the biliary system can result in obstructive jaundice recognized at any time from immediately postoperatively to many years later.

352
Q

A 49-year-old man with cirrhosis from hepatitis C is seen for evaluation. The patient contracted hepatitis C approximately 30 years ago from a blood transfusion but had never been treated. He reports a 15-lb weight loss over 1 year. Physical exam reveals tenderness in the RUQ and stigmata of chronic liver disease. Ultrasonography shows cirrhosis and 2cm lesion in the left hepatic lobe, indicating possible HCC. Serum alpha fetoprotein level is normal. Which of the ff is the best management option?

A. Check the patient’s serum carbohydrate antigen 19-9 level

B. Obtain a CT scan

C. Perform a left hepatectomy

D. Perform surgical resection of the tumor alone

E. Repeat the test for AFP in 3-6 months

A

B. Obtain a CT scan

353
Q

A 22-year-old women develops fullness of the right upper quadrant but otherwise feels well. Results on PE are normal. She has no history of liver disease. She takes a PPI for GERD and an oral contraceptive. A CT scan reveals a 6cm lesion in the right hepatic lobe that is initially hypotenuse, but becomes irregularly enhancing after contrast administration. Which of the following is the lesion most likely to be?

A. Ampullary cancer

B. Cholangiocarcinoma

C. Gallbladder cancer

D. Hepatic adenoma

E. Hepatocellular carcinoma

A

D. Hepatic adenoma

354
Q

A 55-year-old man is evaluated for painless jaundice and 15lb weight loss. He has had intermittent bloody diarrhea for several years. A magnetic resonance study suggests a mass within the CBD. Endoscopic biopsy indicates cholangiocarcinoma. Which of the following is the most likely cause of the patient’s blood diarrhea?

A. Chronic duodenal ulcer

B. Hemorrhoids

C. Shigella enteritis

D. Tumor bleeding into the GI tract

E. Ulcerative colitis

A

E. Ulcerative colitis

355
Q

A 48-year-old woman undergoes abdominal and pelvic CT scanning during a gynecologic evaluation. The scan detects a 3.5cm lesion in the right hepatic lobe. The lesion is hypodense on precontrast images and shows early peripheral enhancement and then complete opacification with IV contrast, consistent with a cavernous hemangioma. The patient has normal liver chemistries and has no other symptoms. Which of the following is the most appropriate next step in management?

A. Measure the patient’s serum AFP level

B. Perform a right hepatectomy

C. Perform chemoembolization of the tumor

D. Perform surgical enucleation of the tumor, preserving the right hepatic lobe

E. Perform no further evaluation or therapy

A

E. Perform no further evaluation or therapy

356
Q

A 40-year-old woman with diabetes and chronic kidney disease presents to the clinic for follow-up after an abdominal ultrasound revealed incidental cholelithiasis without inflammation. The ultrasound revealed two stones, which were less than 5mm in size. The kidneys were normal. The patient denies experiencing abdominal pain, nausea or vomiting. She has not had cholecystitis in the past.

A. Cholecystectomy

B. ERCP

C. Observation

D. Percutaneous transhepatic cholangiogram

E. Alkaline phosphatase and SGPT determination

A

E. Alkaline phosphatase and SGPT determination

357
Q

Which of the following statements most accurately describes the current therapy for pyogenic hepatic abscess?

A. Antibiotics alone are adequate for treatment

B. All patients require open surgical drainage for optimal management.

C. Optimal treatment involves treatment of not only the abscess but the underlying source as well.

D. Percutaneous drainage is more successful for multiple lesions than for solitary ones.

E. ERCP with sphincterotomy is always required.

A

C. Optimal treatment involves treatment of not only the abscess but the underlying source as well.

358
Q

The most common benign hepatic tumor is:

A. Hemangioma

B. Hamartoma

C. Hepatic adenoma

D. Focal nodular hyperplasia

A

A. Hemangioma

359
Q

The most common etiologic factor for HCC worldwide:

A. Non-alcoholic steatohepatitis

B. Hepatitis A virus

C. Hepatitis B virus

D. Alcoholic cirrhosis

A

C. Hepatitis B virus

360
Q

The following statement is/are true regarding vitamin K:

A. Required for factor VIII synthesis

B. Water soluble vitamin

C. Is absorbed in the large intestine

D. Requires bile salts for absorption

A

D. Requires bile salts for absorption

361
Q

A 60 year old male patient came to your clinic for evaluation for liver mass on CT scan done last week. What tumor markers would you normally not request to evaluate the liver mass?

A. AFP
B. CA 19-9
C. CEA
D. CA 15-3

A

D. CA 15-3

362
Q

A central scar in a hepatic lesions is characteristic of which disease entity?

A. Hepatic adenoma

B. Focal nodular hyperplasia

C. Hemangioma

D. Hamartoma

A

B. Focal nodular hyperplasia

363
Q

Which hepatic lesion is most likely to undergo malignant transformation into HCC?

A. Hemangioma

B. Hamartoma

C. Adenoma

D. Focal nodular hyperplasia

A

C. Adenoma

364
Q

Which of the following is true regarding Hepatitis C virus?

A. RNA virus

B. Less commonly results in persistent viremia

C. Can be prevented with effective vaccination

D. Can be transmitted via fecal-oral route

A

A. RNA virus

365
Q

During liver trauma surgery, persistent bleeding was noted even after Pringles clamping was applied. Where could the bleeding be coming from?

A. Hepatic artery

B. Hepatic vein

C. Portal vein

D. Inferior vena cava

A

B. Hepatic vein

366
Q

The strongest risk factor for development of HCC is

A. Hepatitis B

B. Cirrhosis

C. Smoking

D. Alcohol

A

B. Cirrhosis

367
Q

The most common risk factor for Klatskin tumor is

A. Choledocholithiasis

B. Alcohol consumption

C. Choledochal cyst

D. Primary sclerosis cholangitis

A

D. Primary sclerosis cholangitis

368
Q

Management of Esophageal varices?

A

1) Prevention of bleeding
- Alcohol abstinence
- Aspirin and NSAID avoidance
- Propanolol
- Prophylactic Endoscopic Variceal Ligation (EVL)

2) Acute variceal bleeding
- Blood resuscitation
- FFP and platelet
- Prophylactic antibiotics
- Vasopressin 0.2-0.8 units/min
- Somatostatin (initial bolus 50 microunits/IV)
- Surgical shunt/TIPS (refractory variceal bleed)
- Balloon tamponade (<24h)

369
Q

Management of gastric varices?

A

1) Shunt indications
- MELD score <15
- Not candidates for hepatic transplant
- Limited access to TIPS

2) Aim
- Reduce portal venous pressure
- Maintain total hepatic and portal blood flow
- Avoid a high incidence of complication, hepatic encephalopathy

3) Portocaval shunt
- Higher incidence of shunt thrombosis and rebleeding

4) Warren shunt (distal splenorenal)
- Lower rate of hepatic encephalopathy and decompensation

5) Transjugular intrahepatic portosystemic shunt (TIPS)
- >90% of cases refractory to medical treatment

370
Q

What is the SUGUIRA procedure?

A

Indicated for: Extrahepatic portal vein thrombosis and refractory bleeding

  • Extensive devascularization of stomach
  • Transection of esophagus
  • Splenectomy
  • Truncal vagotomy
  • Pyloroplasty
371
Q

Indications for hepatic transplantation in variceal bleeding?

A
  • Only chance of definitive therapy and long term survival
  • Patient with variceal bleeding refractory to all forms of management
  • Reverses most of the hemodynamic and humoral changes associated with cirrhosis
  • Not affected by previous EVL, TIPS, splenorenal, mesocaval shunts
372
Q

Indications and contraindications for laparoscopic liver resection?

A

INDICATIONS

  • Small, superficial, peripheral
  • Segments 2, 3, 4b, 5, 6

CONTRAINDICATIONS

  • Large tumor, deeply seated, posteriorly located in right lobe
  • Tumor close to portal bifurcation or suprahepatic junction
373
Q

Management of multiple bilobar synchronous metastases?

A

1) Chemotherapy (if colonic tumor is not obstructed or symptomatic)

2) After 6 cycles:
- Colon resection
- Clearance of the left lobe of the liver
- Right portal vein ligation

3) After 2 months and 2 cycles of chemotherapy:
- Right hepatectomy

374
Q
  1. A 1-week-old infant is brought to the hospital because of vomiting. An upper gastrointestinal (GI) series reveals duodenal obstruction. On laparotomy, annular pancreas is found. Which of the following statements about annular pancreas is TRUE?

(A) Resection is the treatment of choice.

(B) It is associated with Down’s syndrome.

(C) Symptoms usually begin with back pain.

(D) It is most likely due to abnormal rotation encircling the third part of the duodenum.

(E) Symptoms begin in childhood.

A
  1. (B) Annular pancreas is a congenital anomaly; a band of pancreatic tissue encircles the second part of the duodenum.

Annular pancreas is associated with Down syndrome as well as duodenal stenosis or atresia.

Duodenojejunostomy and gastrojejunostomy are acceptable treatments.

Resection is not an acceptable choice due to the high incidence of fistula In adults, annular pancreas usually presents with abdominal pain, nausea, and vomiting.

375
Q
  1. A 60-year-old alcoholic is admitted to the hospital with a diagnosis of acute pancreatitis. Upon admission, his white blood cell (WBC) count is 21,000. His lipase is 500, blood glucose is 180 mg/dL, lactate dehydrogenase (LDH) is 400 IU/L, and aspartate aminotransferase (AST) is 240 IU/dL. Which of the following is TRUE?

(A) This patient is expected to have a mortality rate of less than 5%.

(B) The patient’s lipase level is an important indication of prognosis.

(C) This patient requires immediate surgery.

(D) A venous blood gas would be helpful in assessing the severity of illness in this patient.

(E) A serum calcium level of 6.5 mg/dL on the second hospital day is a bad prognostic sign.

A
  1. (E) The patient has three Ranson’s criteria at the
    time of admission.

The expected mortality rate is 15% with 3–4 Ranson’s criteria.

Amylase and lipase levels are not prognostic factors in acute pancreatitis.

Calcium level <8 mg/dL within the first 48 hours is one of Ranson’s criteria, as is arterial PO2 <60 mm Hg.

<2 points: 5% mortality
3-4: 15-20%
≤6: 40%
>7: 99%

376
Q
  1. A 19-year-old man is brought to the emergency
    department by emergency medical service (EMS) with a stab-wound to the right upper quadrant (RUQ) of the abdomen. A FAST scan shows free fluid, and the patient is taken to the operating room for an exploratory laparotomy. The findings are a nonbleeding laceration of the right lobe of the liver and a gallbladder laceration. Which of the following is TRUE?

(A) The gallbladder injury can be treated with cholecystectomy.

(B) Isolated gallbladder injuries are uncommon.

(C) Bile is usually sterile.

(D) The liver laceration does not require closed suction drainage.

(E) A thorough exploration is not necessary if the bleeding is confined to the RUQ.

A
  1. (B) Most gallbladder injuries are associated
    with other injuries, most often to the liver, large
    intestine, and/or small intestine.

Isolated gallbladder injuries are rare.

A gallbladder injury can be treated with cholecystectomy or cholecystostomy.

A nonbleeding liver laceration does not need further treatment.

A careful search for injuries should be made during laparotomy.

377
Q
  1. A 15-year-old female presents with RUQ abdominal pain. Workup reveals a choledochal cyst. Which of the following statements is TRUE?

(A) Choledochal cysts are more common in men.

(B) Laparoscopic cholecystectomy is the recommended treatment.

(C) Patients with a choledochal cyst have an increased risk of cholangiocarcinoma.

(D) All patients with a choledochal cyst have abdominal pain, a RUQ mass, and jaundice.

(E) The etiology is infectious.

A
  1. (C) Choledochal cysts can involve the intrahepatic and/or extrahepatic biliary tree (see Fig. 7–3 for classification).

Choledochal cysts present more commonly in infants and children, but may present in adults.

They are more common in females.

The classic triad of jaundice, RUQ mass and abdominal pain is found in less than a third of patients.

There is an association between choledochal cysts and hepatobiliary cancers, most commonly cholangiocarcinoma.

For most types of choledochal cyst, excision of the cyst with a Roux-en-Y biliary enteric anastomosis is
recommended.

Laparoscopic cholecystectomy alone is not sufficient.

378
Q
  1. A 13-year-old female presenting with RUQ abdominal pain is suspected of having a choledochal cyst. Which of the following studies would be least helpful in confirming the diagnosis in this case?

(A) Computed tomography (CT) scan

(B) Percutaneous transhepatic cholangiography

(C) Endoscopic retrograde cholangiopancreatography

(D) Magnetic resonance cholangiopancreatography (MRCP)

(E) Upper GI series

A
  1. (E) An upper GI series would not visualize the cyst.

Ultrasound may diagnose a choledochal cyst, showing size and location, but is not always diagnostic.

CAT scan and MRCP can show size, location, and extent of disease.

ERCP visualizes the distal duct anatomy well, while PTC is better at visualizing the proximal ductal anatomy.

379
Q
  1. An intraoperative cholangiogram is performed during an elective laparoscopic cholecystectomy on a 30-year-old woman. She has no previous surgical history. There is a 0.8-cm filling defect in the distal common bile duct (CBD). The surgeon should:

(A) Complete the laparoscopic cholecystectomy and check liver function tests (LFTs) postoperatively. If they are normal, no further treatment is needed.

(B) Complete the laparoscopic cholecystectomy and repeat an ultrasound postoperatively. Observe the patient if no CBD stone is visualized.

(C) Perform a CBD exploration either laparoscopically or open along with a cholecystectomy.

(D) Complete the laparoscopic cholecystectomy, no further treatment is necessary.

(E) Complete the laparoscopic cholecystectomy and plan for a postoperative hydroxy iminodiacetic
acid (HIDA) scan.

A
  1. (C) The intraoperative cholangiogram is suggestive of a CBD stone.

Normal LFTs do not rule out choledocholithiasis.

Patients with choledocholithiasis often have dilatation of the CBD on ultrasound; the stones may be visualized, but a normal ultrasound does not rule out CBD stones.

A HIDA scan is unlikely to be helpful.

An exploration of the CBD is indicated (either laparoscopically or open) along with a cholecystectomy.

380
Q
  1. An 85-year-old man is brought to the hospital
    with a 2-day history of nausea and vomiting. He has not passed gas or moved his bowels for the last 5 days. Abdominal films show dilated small bowel, no air in the rectum and air in the biliary tree. Which of the following statements is TRUE?

(A) Air in the biliary tree associated with small-bowel obstruction suggests a diagnosis of gallstone ileus.

(B) An enterotomy should be distal to the site of obstruction and the stone should be removed.

(C) Gallstone ileus is more common in the young adults.

(D) Cholecystectomy is contraindicated.

(E) Small-bowel obstruction usually occurs in the distal jejunum.

A
  1. (A) Gallstone ileus usually results from formation of a cholecystoenteric fistula and is seen more often in elderly patients.

Obstruction occurs most often at the terminal ileum.

Treatment is laparotomy and removal of the stone through an enterotomy proximal to the obstruction; cholecystectomy should be done if the patient can tolerate the additional surgery.

381
Q
  1. A 45-year-old man with hepatitis C undergoes an uneventful percutaneous liver biopsy. About 6-weeks later, he complains of RUQ pain, is clinically jaundiced, with a hemoglobin of 9.2 mg/dL and is fecal occult blood positive. Which diagnosis best explains this patient’s symptoms?

(A) Hepatocellular carcinoma

(B) Chronic hepatitis C

(C) Colon carcinoma with liver metastasis

(D) Hemobilia

(E) Symptomatic cholelithiasis

A
  1. (D) Hemobilia should be suspected in a patient
    with a history of liver trauma who later develops GI bleeding and abdominal pain.

Hemobilia usually appears weeks after the injury; pain is often intermittent and melena or hematemesis may occur.

In this case, the injury is the result of the melena and hematemesis may occur.

In this case, the injury is the result of the biopsy.

The other diseases listed are less likely to explain all of the findings listed.

382
Q
  1. A 40-year-old patient with a history of trauma to the RUQ presents with RUQ pain, clinical jaundice, and guaiac positive stools. Which one of the following studies would be most useful to confirm the patient’s diagnosis?

(A) Abdominal ultrasound

(B) CT of the abdomen

(C) Angiography

(D) HIDA scan

(E) Diagnostic laparoscopy

A
  1. (C) Once again hemobilia should be suspected,
    with the history of trauma to the liver and guaiac positive stools and RUQ abdominal pain.

Angiography can be diagnostic as well as therapeutic.

The source of bleeding can be identified and embolized.

Ultrasound is unlikely to identify the bleeding source; it would identify cholelithiasis or a liver tumor.

A CAT scan likewise would identify a tumor.

A HIDA scan documents patency of the cystic duct and would not be useful in this case.

Laparoscopy would be unlikely to identify the communication between a hepatic vessel and the biliary tree.

383
Q
  1. A 40-year-old female alcoholic is suspected of having a hepatic mass. Percutaneous ultrasound-guided liver biopsy is contraindicated in which of the following?

(A) Hepatocellular carcinoma

(B) Metastatic carcinoma

(C) Cirrhosis

(D) Hepatitis C

(E) Hepatic adenoma

A
  1. (E) Tru-cut needle liver biopsy allows pathologic diagnosis of liver lesions.

Needle biopsy is contraindicated if hemangioma is suspected and in adenomas, because of the risk of bleeding.

Other potential complications of percutaneous needle biopsy are pain, pneumothorax, and bile peritonitis.

Needle biopsy can diagnose posthepatic and postnecrotic cirrhosis, malignant tumors, and hepatitis, and can determine the need for treatment in hepatitis C.

384
Q
  1. A 20-year-old man is brought to the emergency
    department with a gunshot wound to the abdomen. His blood pressure is 70 systolic and his heart rate is 140 beats per minute (bpm). He is taken directly to the operating room for an exploratory laparotomy. A large, actively bleeding liver laceration is found. A pringle maneuver is performed as part of the procedure to control his bleeding. The pringle maneuver compresses which structures?

(A) Portal vein, hepatic vein, and hepatic artery

(B) Portal vein, hepatic artery, and cystic artery

(C) Portal vein and hepatic artery

(D) Portal vein, hepatic artery, and CBD

(E) Cystic artery, cystic duct, and CBD

A
  1. (D) Pringle’s maneuver is occlusion of the porta
    hepatis. The portal vein, hepatic artery, and the
    CBD are the structures of the porta hepatis.
385
Q
  1. A 22-year-old medical student is seen by the
    student health service prior to beginning school. Routine labs are drawn. The medical student immunized against hepatitis B in childhood will have which hepatitis profile?

(A) HbsAb+, HbsAg+, HbcAb+

(B) HbsAb+, HbsAg+, HbcAb−

(C) HbsAb−, HbsAg−, HbcAb−

(D) HbsAb+, HbsAg−, HbcAb−

(E) HbsAb−, HbsAg+, HcbAb−

A
  1. (D) Hepatitis B vaccine is made from genetically engineered hepatitis B surface antigen particles. Vaccination produces hepatitis B surface
    antibodies but not hepatitis B core antibodies.
    Hepatitis B surface antigen will not be present.
386
Q
  1. A 36-year-old man presents to the emergency department after a motor vehicle crash. He is complaining of left-sided chest pain and abdominal pain. His blood pressure is 130/80 mm Hg. An electrocardiogram shows sinus rhythm with a heart rate of 95 bpm. A chest x-ray shows left 8, 9, and 10 rib fractures. An abdominal computed axial tomography (CAT) scan is obtained. It shows a 3-cm laceration in the upper pole of the spleen with a small amount of blood around the spleen. No other injury is identified. Which of the following statements is TRUE?

(A) This is a class I injury and it may be treated nonoperatively.

(B) This is a class II injury and it may be treated nonoperatively.

(C) This is a class II injury and it requires immediate laparotomy.

(D) The patient should be prophylactically transfused in anticipation of continued blood loss.

(E) Delayed splenic rupture is not possible with this injury.

A
13. (B) A laceration to the parenchyma of the spleen
1–3 cm deep is a class II injury. 

A class I injury is a nonexpanding subcapsular hematoma involving less than 10% of the surface area of the spleen.

Nonoperative management may be attempted for both of these injuries.

This patient is hemodynamically stable and does not require emergency laparotomy.

A class III injury is a major parenchymal injury.

Prophylactic transfusion is not indicated.

Delayed splenic rupture may occur within 2 weeks or more of a blunt splenic injury in 10–15% of patients.

387
Q
  1. A 38-year-old man undergoes excisional biopsy
    of a cervical lymph node. Pathology reveals Hodgkin’s lymphoma. Which of the following statements about Hodgkin’s disease is TRUE?

(A) Splenectomy is always required for accurate staging.

(B) Staging laparotomy involves liver biopsy, biopsy of the spleen, and periaortic lymph node dissection.

(C) Stage II disease involves disease on both sides of the diaphragm.

(D) If the spleen is involved, the patient has stage IV disease.

(E) Splenectomy is sometimes indicated for thrombocytopenia.

A
  1. (E) Staging of Hodgkin’s disease is often done
    nonoperatively.

Staging laparotomy consists of wedge biopsy of the liver, splenectomy, examination and biopsy of peraortic lymph nodes, as well as biopsy of mesenteric and hepatoduodenal nodes.

Stage I Hodgkin’s lymphoma is limited to one anatomic area, while Stage II involves 2 or more areas of the same side of the diaphragm.

Stage III disease involves both sides of the diaphragm limited to lymph nodes, Waldyer’s ring or the spleen.

Stage IV disease involves organs other than lymph nodes, Waldyer’s ring, or the spleen.

Splenectomy may improve thrombocytopenia and allow chemotherapy to be administered.

388
Q
  1. A 50-year-old woman complains of weakness,
    profuse watery diarrhea, and crampy abdominal pain. She reports a 10-lb weight loss. Her serum potassium is 2.8 mEq/L. Select the most likely diagnosis.

(A) Watery diarrhea, hypokalemia, and achlorhydria (WDHA) syndrome

(B) Somatostatinoma

(C) Glucagonoma

(D) Insulinoma

(E) Multiple endocrine neoplasia type 1 (MEN-1)

A
  1. (A) WDHA or vasoactive intestinal polypeptide (VIPoma) is characterized by voluminous diarrhea, 5 L or more daily, rich in potassium, which looks like watery tea.

The diarrhea is secretory and if refractory to antidiarrheal agents.

Patients are weak, with metabolic acidosis and hypokalemia.

Octreotide decreases diarrhea volume.

The pancreatic tumor should be excised.

Secretory diarrhea also occurs in some patients with Zolliger-Ellison syndrome (ZES), and is the only complaint in less than 10% of ZES patients.

More than 90% of ZES patients have peptic ulcer disease.

Unlike the diarrhea associated with ZES, the diarrhea of WHDA continues with fasting and continuous nasogastric tube suctioning.

15% of patients with glucagonoma have diarrhea; glucagonoma is associated with migratory necrotizing dermatitis.

389
Q
  1. A 45-year-old man presents with an upper GI bleed. An upper endoscopy reveals multiple duodenal ulcers and an enlarged stomach. Select the most likely diagnosis.

(A) WDHA syndrome

(B) Glucagonoma

(C) Zollinger-Ellison syndrome

(D) Insulinoma

(E) Somatostatinoma

A
  1. (C) Gastrinoma, or Zollinger-Ellison syndrome,
    should be suspected in patients with peptic ulcer
    disease refractory to medical treatment, or in
    patients with multiple ulcers or ulcers in uncommon locations.

Gastrin secretion by the tumor, most commonly found in the pancreas, results in hypersecretion of gastric acid.

Common patient complaints are epigastric pain, melena, hematemesis, diarrhea, and weight loss.

ZES may occur as part of the MEN-1 syndrome; this
patient presents with only atypical peptic ulcer disease.

The treatment of choice involves the identification and resection of the gastrinoma.

Preoperative treatment may include treatment of the ulcers with omeprazole.

390
Q
  1. A 35-year-old woman with epigastric pain, which did not improve on ranitidine, is found to have a nonhealing pyloric channel ulcer on upper endoscopy. Her serum calcium level is 12 mg/dL. Select the most likely diagnosis.

(A) WDHA syndrome

(B) MEN-1

(C) MEN-2A

(D) MEN-2B

(E) Zollinger-Ellison syndrome

A
  1. (B) Multiple endocrine neoplasia syndrome type 1 (MEN-1, or Werner’s syndrome), and autosomal dominant disorder, involves tumors or hyperplasia of two or more glands, most commonly parathyroid, pancreas, and pituitary glands.

Hyperparathyroidism is most common, followed by various pancreatic isle cell tumors and pituitary adenomas.

MEN-2A (Sipple’s syndrome) consists of pheochromocytoma, medullary carcinoma of the thyroid, and often hyperparathyroidism.

MEN-2B is characterized by medullary carcinoma of the thyroid, pheochromocytoma, neuromas, and marfinoid body habitus.

MEN 2-A and 2-B are also autosomal dominant.

391
Q
  1. A 30-year-old man is noted to be anemic, with clinical jaundice and a palpable spleen on abdominal exam. Splenectomy is the only treatment for this patient’s autosomal dominant disorder. Select the most likely diagnosis.

(A) Thalassemia

(B) Hereditary spherocytosis

(C) Sickle cell disease

(D) Idiopathic autoimmune hemolytic anemia

(E) Thrombotic thrombocytopenic purpura (TPP)

A
  1. (B) Hereditary spherocytosis is the most common symptomatic familial hemolytic anemia, and is transmitted as an autosomal dominant trait.

A defect in the red cell membrane causes increased trapping in the spleen and hemolysis.

Anemia, jaundice, and splenomegaly are clinical findings.

Splenectomy is the only treatment.

Thalassemia is transmitted as a dominant trait; anemia is the result of a defect in hemoglobin synthesis.

Thalassemia major, or homozygous thalassemia, is associated with anemia, icterus, splenomegaly, and early death.

Transfusions are usually required. Splenectomy may reduce hemolysis and transfusion requirements.

Sickle cell anemia is hereditary hemolytic anemia.
Serum bilirubin may be mildly elevated. Splenomegaly often precedes autoinfarction. Splenectomy may be indicated for chronic hypersplenism or acute splenic sequestration.

392
Q
  1. The peripheral smear of a child with anemia shows hypochromic microcytic anemia with target cells. What is the child’s diagnosis?

(A) Thalassemia

(B) Hereditary spherocytosis

(C) Sickle cell disease

(D) Idiopathic autoimmune hemolytic anemia

(E) TTP

A
  1. (A) In thalassemia, intracellular hemoglobin precipitates, or Heinz bodies, damage red cells and contribute to early destruction.

Cells are small, thin, misshapen, and resistant to osmotic lysis.

Diagnosis is made by peripheral smear.

Nucleated red cells, or target cells, are present.

Distorted red cells, or target cells, are present.

Distorted red cells of different shapes and sizes are found.

In sickle cell disease, characteristic sickle cells are seen on peripheral smear.

In hereditary spherocytosis, the peripheral smear shows small, thick, nearly spherical red cells.

Cells have increased osmotic fragility.

393
Q
  1. A woman with longstanding rheumatoid arthritis has neutropenia on routine labs and splenomegaly is noted on physical examination. Which is the most likely diagnosis?

(A) Thalassemia

(B) Hereditary spherocytosis

(C) Sickle cell disease

(D) Idiopathic autoimmune hemolytic anemia

(E) Felty’s syndrome

A
  1. (E) The triad of rheumatoid arthritis, splenomegaly, and neutropenia is known as Felty’s
    syndrome.

Gastic achlorhydria is common.

Thrombocytopenia and mild anemia are sometimes seen.

Splenectomy is sometimes used to treat the neutropenia in patients with serious infections, anemia requiring transfusions, or severe thrombocytopenia.

394
Q
  1. A 50-year-old woman underwent wide excision of a 2.5-cm infiltrating ductal carcinoma of the breast with axillary lymph node dissection followed by radiation and chemotherapy 2 years ago. The patient now complains of RUQ abdominal pain. A CAT scan reveals two masses in the right lobe of the liver. Select the most likely diagnosis.

(A) Adenoma

(B) Focal nodular hyperplasia

(C) Hemangioma

(D) Hepatocellular carcinoma

(E) Metastatic carcinoma

A
  1. (E) Breast cancer commonly metastasizes to bone, lung, soft tissues, liver, and brain.

The patient should be worked up for local recurrence as well as other distant metastasis.

The presence of masses in the liver should lead to the diagnosis of metastatic cancer.

395
Q
  1. A 35-year-old woman complains of RUQ pain
    after meals with nausea and vomiting. An ultrasound reveals cholelithiasis and an anechoic 3cm mass on the inferior surface of the right lobe of the liver. Select the most likely diagnosis.

(A) Nonparasitic cyst

(B) Hydatid cyst

(C) Hamartoma

(D) Adenoma

(E) Focal nodular hyperplasia

A
  1. (A) Benign liver cysts can be single or multiple.

Solitary nonparasitic cysts usually contain clear,
watery fluid.

These cysts are more common in the right lobe.

They are most likely congenital and most are asymptomatic; many are found incidentally.

An anechoic area on ultrasound is suggestive.

Hydatid cysts, caused by Echinococcus, are also more common in the right lobe.

The colorless fluid in the cyst is under high pressure, unlike parasitic cysts.

Ultrasound will show internal echoes.

Hemangiomas can have a variable echogenic pattern on ultrasound; focal nodular hyperplasia is often hypodense.

Hepatocellular carcinoma and metastasis have a characteristic sonographic appearance different from benign nonparasitic cysts.

396
Q
  1. A 42-year-old man who consumed more than 3 bottles of vodka weekly over the past 20 years is admitted with upper abdominal pain radiating to the back, nausea, and vomiting. Serum amylase and lipase are elevated, and a diagnosis of pancreatitis is made. In determining his prognosis, which of the following factors would cause the greatest concern?

(A) Hypercalcemia (Ca >12 mg/dL)

(B) Age over 40 years

(C) Hypoxemia

(D) Hyperamylasemia (>600 U)

(E) Elevated lipase

A
  1. (C) Ranson’s criteria allow for early identification
    of patients who have severe pancreatitis.

Mortality increases with increasing number of Ranson’s criteria score.

The five criteria of poor prognosis at the time of admission are age >55, WBC >16,000, blood glucose >200 mg/dL, AST >250, LDH >350.

During the following 48 hours, six additional criteria may develop. These include hypoxemia with arterial PO2 <60 mm on room air, base deficit >4, fluid requirement >6 L, hematocrit fall >10%, blood urea nitrogen (BUN) increase >8 mg/dL, and serum Ca <8 mg/dL.

Amylase and lipase elevation may focus attention on the appropriate diagnosis, but amylase levels fail to correlate with prognosis.

397
Q
  1. A 24-year-old college student recovers from a bout of severe pancreatitis. He has mild epigastric discomfort, sensation of bloating, and loss of appetite. Examination reveals an epigastric fullness that on ultrasound is confirmed to be a pseudocyst. The swelling increases in size over a 3-week period of observation. What should be the next step in management?

(A) Percutaneous drainage of the cyst

(B) Laparotomy and internal drainage of the cyst

(C) Excision of pseudocyst

(D) Total pancreatectomy

(E) Administration of pancreatic enzymes

A
  1. (A) Pseudocysts frequently are encountered on ultrasound examination early after an acute attack of pancreatitis.

In most cases, the pseudocyst resolves, but if it enlarges, it may compress the stomach anteriorly.

An enlarging pseudocyst is an indication to attempt percutaneous drainage.

If percutaneous drainage is unsuccessful, internal drainage into the stomach should be performed at an appropriate interval to allow the pseudocyst wall to mature (Fig. 7–4).

398
Q
  1. A 40-year-old alcoholic male is admitted with severe epigastric pain radiating to the back. Serum amylase level is reported as normal, but serum lipase is elevated. The serum is noted to be milky in appearance. A diagnosis of pancreatitis is made. The serum amylase is normal because

(A) The patient has chronic renal failure.

(B) The patient has hyperlipidemia.

(C) The patient has alcoholic cirrhosis.

(D) The patient has alcoholic hepatitis.

(E) The diagnosis of pancreatitis is incorrect.

A
  1. (B) In pancreatitis, the serum amylase level may
    be normal.

The causes include: (a) hyperlipidemia, which interferes with chemical determination of amylase; (b) increased urinary excretion of amylase; and (c) near complete destruction of pancreatic parenchyma as a result of chronic pancreatitis.

On the other hand, the serum amylase level may be elevated in the absence of pancreatitis (for example, perforated peptic ulcer, gangrenous cholecystitis, small bowel strangulation or chronic renal failure.)

399
Q
  1. A 52-year-old woman is admitted to the hospital with abdominal pain. She reports that she drinks alcohol only at social occasions. The amylase is elevated to 340 U. Which following x-ray finding would support a diagnosis of idiopathic pancreatitis?

(A) Hepatic lesion on CT scan

(B) Choledocholithiasis on ultrasound

(C) Anterior displacement of the stomach on barium upper GI series

(D) Large loop of colon in the RUQ

(E) Irregular cutoff of the CBD on cholangiogram

A
  1. (C) If a large pseudocyst is present, it may cause displacement of the transverse colon, duodenum, or stomach (anteriorly).

Other radiologic signs in pancreatitis include pseudocyst on ultrasound or CT scan, downward displacement of transverse colon, dilated pancreatic duct on pancreatogram, and smooth tapering of the CBD on cholangiogram (if the head of the pancreas is diseased).

The irregular tapering of the common duct is suggestive of neoplasm.

The looping of the colon in the RUQ is seen with sigmoid volvulus.

400
Q
  1. A 67-year-old woman is noted to have a gradual increase in the size of the abdomen. A CT scan reveals a large pancreatic mass. The lesion was excised; on pathology examination, it is shown to be a TRUE cyst. Which statement is correct regarding true cysts?

(A) They are commonly seen in alcoholic pancreatitis.

(B) They commonly occur after trauma.

(C) They are frequently malignant.

(D) They are associated commonly with choledochocele.

(E) They have an epithelial lining.

A
  1. (E) True epithelial-lined cysts in the pancreas are
    extremely rare.

They should not be confused with the more common pseudocyst (no epithelial lining), benign cystadenoma, or malignant cystadenoma of the pancreas.

Pseudocysts are more common in men, but cystadenocarcinoma occurs more frequently in women.

401
Q
  1. A 40-year-old man with a history of alcohol consumption of 25-year duration is admitted with a history of a 6-lb weight loss and upper abdominal pain of 3-weeks duration. Examination reveals fullness in the epigastrium. His temperature is 99°F, and his WBC count is 10,000. Which is the most likely diagnosis?

(A) Pancreatic pseudocyst

(B) Subhepatic abscess

(C) Biliary pancreatitis

(D) Cirrhosis

(E) Splenic vein thrombosis

A
  1. (A) The presence of an epigastric mass 2–3 weeks after the onset of acute pancreatitis strongly favors a pancreatic pseudocyst.

The history of alcoholism points to pancreatitis as a possible etiologic factor in the differential diagnosis.

Pseudocysts develop in 10% of patients following acute pancreatitis. Most of these, however, resolve spontaneously.

They may also develop in patients with chronic pancreatitis or after pancreatic trauma.

402
Q
  1. A 58-year-old man with a 30-year history of alcoholism and pancreatitis is admitted to the hospital with an elevated bilirubin level of 5 mg/dL, acholic stools, and an amylase level of 600 U. Obstructive jaundice in chronic pancreatitis usually results from which of the following?

(A) Sclerosing cholangitis

(B) CBD compression caused by inflammation

(C) Alcoholic hepatitis

(D) Biliary dyskinesia

(E) Splenic vein thrombosis

A
  1. (B) Fibrosis in the head of the pancreas as a result of chronic inflammation may lead to compression of the CBD.

In pancreatitis, the narrowing of the CBD is smooth on x-ray studies.

There is no association with pancreatitis and sclerosing cholangitis.

Alcoholic hepatitis is the most common cause of jaundice, but it most frequently is not of an obstructive nature.

Pseudocysts and carcinoma of the head of the pancreas are other recognized causes of obstructive jaundice in patients with chronic pancreatitis.

403
Q
  1. A 48-year-old woman is admitted with acute cholecystitis. The bilirubin level is elevated, as are the serum and urinary amylase levels. Which radiologic sign indicates biliary obstruction in pancreatitis?

(A) Pancreatic intraductal calcification

(B) Smooth narrowing of the distal CBD

(C) Stomach displaced anteriorly

(D) Calcified gallstone

(E) Air in the biliary tree

A
  1. (B) The passage of small stones through Vater’s
    ampulla often results in pancreatitis.

It is important to perform cholecystectomy after pancreatitis has subsided but during the same hospital stay in patients with documented gallstone pancreatitis (to avoid recurrence of symptoms).

Smooth tapering of the common duct is usually seen with stones obstructing the common duct.

Pancreatic intraductal calcification is consistent with chronic pancreatitis, and air in the biliary tree is consistent with gallstone ileus.

404
Q
  1. A 62-year-old man is admitted with abdominal pain and weight loss of 5 lb over the past month. He has continued to consume large amounts of rum. Examination reveals icteric sclera. The indirect bilirubin level is 5.6 mg/dL with a total bilirubin of 6 mg/dL. An ultrasound shows a 4-cm pseudocyst. What is the most likely cause of jaundice in a patient with alcoholic pancreatitis?

(A) Alcoholic hepatitis

(B) Carcinoma of pancreas

(C) Intrahepatic cyst

(D) Pancreatic pseudocyst

(E) Hemolytic anemia

A
  1. (A) A recent increase in alcohol consumption
    explains the jaundice secondary to alcoholic hepatitis in the majority of such patients.

Carcinoma of the pancreas is relatively rare but often causes difficulty in the differentiation from pancreatitis.

A pseudocyst measuring 4 cm is not likely to be associated with nonobstructive jaundice in this patient.

405
Q
  1. A 42-year-old woman with a history of chronic alcoholism is admitted to the hospital because of acute pancreatitis. The bilirubin and amylase levels are in the normal range. An ultrasound reveals cholelithiasis. The symptoms abate on the fifth day after admission. What should she be advised?

(A) To start on a low-fat diet.

(B) To increase the fat content of her diet.

(C) To undergo immediate cholecystectomy.

(D) To undergo cholecystectomy during the same hospital stay as well as an assessment of her bile ducts.

(E) That she will be discharged and now should undergo elective cholecystectomy after 3 months.

A
  1. (D) Patients who develop acute pancreatitis as a
    result of cholelithiasis should have gallbladder
    surgery performed during the same hospital stay
    to avoid recurrence.

An assessment of the bile ducts should be performed either preoperatively or intraoperatively after the resolution of the pancreatitis.

Elective cholecystectomy should be avoided during the actual phase of pancreatitis.

406
Q
  1. Following a motor vehicle accident a truck driver complains of severe abdominal pain. Serum amylase level is markedly increased to 800 U. Grey Turner’s sign is seen in the flanks. Pancreatic trauma is suspected. Which statement is true of pancreatic trauma?

(A) It is mainly caused by blunt injuries.

(B) It is usually an isolated single-organ injury.

(C) It often requires a total pancreatectomy.

(D) It may easily be overlooked at operation.

(E) It is proved by the elevated amylase level.

A
  1. (D) Because of its protected retroperitoneal location, pancreatic injury occurs with deep penetrating wounds or with significant blunt
    trauma to upper abdomen.

Blunt trauma accounts for less than 20–30% of all pancreatic injuries.

The most common site of injury is at the neck of the pancreas where the pancreatic tissue is compressed against the spine.

Associated visceral and vascular injuries occur commonly and together with the delay in diagnosis account for the high morbidity and mortality.

Fistulae, pseudocyst, infection, and secondary (delayed) hemorrhage are common complications.

Pancreatic injuries frequently are overlooked initially, and their detection requires a high index of suspicion.

Elevation of amylase after trauma is nonspecific.

407
Q
  1. A 40-year-old woman with severe chronic pancreatitis is scheduled to undergo an operation,
    because other forms of treatment have failed. The ultrasound shows no evidence of pseudocyst formation or cholelithiasis and endoscopic retrograde cholangiopancreatogram (ERCP) demonstrates dilated pancreatic ducts with
    multiple stricture formation. Which operation is suitable to treat this condition?

(A) Pancreaticojejunostomy (Puestow procedure)

(B) Gastrojejunostomy

(C) Cholecystectomy

(D) Splenectomy

(E) Subtotal pancreatectomy

A
  1. (A) If the pancreatic duct is dilated and symptoms persist, a longitudinal pancreaticojejunostomy (Puestow) is performed (Fig. 7–5).

In this operation, the pancreatic duct is slit open and anastomosed side-to-side to the cut end of the divided jejunum with a Roux-en-Y anastomosis.

Resection of the pancreas is reserved for patients without a dilated duct (<6 mm).

In these cases, a distal pancreatectomy is performed when the disease primarily involves the body and tail of the pancreas; whereas, a Whipple operation is performed when the disease is confined to the head.

408
Q
  1. A 26-year-old woman with a known history of
    chronic alcoholism is admitted to the hospital
    with severe abdominal pain due to acute pancreatitis. The serum and urinary amylase levels
    are normal. On the day following admission to
    the hospital, there is no improvement, and she
    has a mild cough and and slight dyspnea. What
    is the most likely complication?
    (A) Pulmonary atelectasis
    (B) Bronchitis
    (C) Pulmonary embolus
    (D) Afferent loop syndrome
    (E) Pneumonia
A
  1. (A) Atelectasis is partly due to a factor released
    from the pancreas that alters pulmonary surfactant. The other conditions listed are not
    specifically related to pancreatitis.
409
Q
  1. A 30-year-old male is admitted with frequent
    episodes of hypoglycemia. Biochemical investigations confirmed an insulinoma. Localization
    studies were carried out. ACT scan and magnetic
    resonance imaging (MRI) of the abdomen failed
    to reveal a tumor in the pancreas. An endoscopic
    ultrasound, however, localized a 2-cm insulinoma in the tail of the pancreas. What should be
    the next step in the management of this patient?
    (A) Somatostatin receptor scintigraphy
    (SRS) to confirm the insulinoma
    (B) Exploratory laparotomy and total
    pancreatectomy
    (C) Distal pancreatectomy
    (D) Whipple pancreaticoduodenectomy
    (E) Enucleation of the tumor
A
  1. (E) Most insulinomas are small (<2 cm), solitary,
    and benign. Therefore, simple enucleation is adequate. Less than 10% of cases are malignant and
    require resection in the form of either pancreaticoduodenectomy or distal pancreatectomy
    (depending upon the location of the tumor). Ten
    percent of insulinomas are associated with MEN
    I syndrome, and in these cases, the tumors are
    multiple. Partial pancreatic resection may be
    required for these patients. Total pancreatectomy
    is almost never required for the removal of insulinomas. Somatostatin receptors are not always
    present on insulinoma cells, and, therefore, SRS is
    less useful for localization of this tumor.
410
Q
  1. A 66-year-old man with obstructive jaundice is
    found on ERCP to have periampullary carcinoma. He is otherwise in excellent physical
    shape and there is no evidence of metastasis.
    What is the most appropriate treatment?
    (A) Radical excision (Whipple procedure)
    where possible
    (B) Local excision and radiotherapy
    (C) External radiotherapy
    (D) Internal radiation seeds via catheter
    (E) Stent and chemotherapy
A
  1. (A) Carcinoma of the head of the pancreas is
    treated with radical excision of the head of the
    pancreas along with the duodenum. Continuity
    of the biliary and GI tract is established by performing hepaticojejunostomy, pancreaticojejunostomy, and gastrojejunostomy (Fig. 7–6).
    The 5-year survival rate is higher for periampullary carcinoma (30%) than that for pancreatic head lesions (10%). Most centers do not
    give irradiation routinely before or after surgery, because pancreatic cancers do not respond
    well to radiotherapy. Endoscopically placed
    stents alone are used only in palliative circumstances in patients with limited life expectancy.
411
Q
  1. A 74-year-old man complains of epigastric discomfort. There is no jaundice evident, but an
    enlarged gallbladder is palpated. The bilirubin
    level is 13 mg/dL, the alkaline phosphatase
    level is 410 U, and the hematocrit is 35%. CT
    scan and MRI findings are shown in Fig. 7–1.
    What is the most likely malignant tumor causing extrahepatic obstructive jaundice?
    (A) Gallbladder
    (B) Common hepatic duct
    (C) Cystic duct
    (D) Periampullary area
    (E) Head of the pancreas
A
  1. (E) Cancer of the head of the pancreas is the
    most common cause of obstructive jaundice.
    In cholangiocarcinoma of the common hepatic
    duct, the gallbladder will be empty and not
    distended. Anemia may occur as a result of
    bleeding into the duodenum in periampullary
    cancer, but this is relatively rare. Carcinoma of
    the gallbladder results in jaundice only after
    the tumor invades the adjacent biliary tree.
412
Q
  1. A 25-year-old female presents with episodes of
    bizarre behavior, memory lapse, and unconsciousness. She also demonstrated previously
    episodes of extreme hunger, sweating, and tachycardia. During one of these episodes, her blood
    sugar was tested and was found to be 40 mg/dL.
    Which of the following would most appropriately indicate a diagnosis of insulinoma?
    (A) Demonstration of insulin antibodies in
    blood
    (B) Abnormal glucagon level
    (C) CT of the pancreas showing a mass
    (D) Hypoglycemia during a symptomatic
    episode with relief of symptoms by
    intravenous glucose
    (E) Decreased circulating C peptide in the
    blood
A
  1. (D) The characteristic features of insulinomas
    include: (a) hypoglycemic symptoms; (b) blood glucose <50 mg/dL during the symptomatic episodes; and (c) relief of symptoms by
    intravenous injection of glucose (Whipple’s
    triad). Diagnosis is confirmed by demonstration of fasting hypoglycemia in the presence of
    inappropriately elevated levels of insulin in
    the blood. A ratio of plasma insulin/glucose
    >0.3 is diagnostic. Circulating levels of Cpeptide are usually elevated in patients with
    insulinoma but not in patients with such other
    causes of hypoglycemia as tumors of mesenchymal origin and liver tumors. Patients
    who surreptitiously administer insulin develop
    insulin antibodies.
413
Q
  1. A 41-year-old woman is known to have multiple endocrine neoplasia syndrome. She has
    multiple family members who have had adenoma tumors removed from the parathyroid,
    pancreas, and/or pituitary glands. She has
    severe diarrhea associated with low gastric acid
    secretion and a normal gastrin level. Which of
    the following serum assays would be best to
    evaluate the possible cause of the diarrhea?
    (A) Glucagon
    (B) Vasoactive intestinal peptide (VIP)
    (C) Cholecystokinin
    (D) Serotonin
    (E) Norepinephrine
A
  1. (B) VIP producing tumors (VIPomas) are usually
    malignant, although benign tumors and hyperplasia may also occur. Increase of VIP results in
    the WDHA syndrome. Diarrhea is severe and
    results in fluid and electrolyte disturbances.
    Treatment is directed to removal of the pancreatic
    tumor. Gastrinoma is more common but is associated with increased gastrin level in the blood.
414
Q
  1. A 45-year-old patient with chronic pancreatitis is
    suffering from malnutrition and weight loss secondary to inadequate pancreatic exocrine secretions. Which is TRUE regarding pancreatic
    secretions?
    (A) Secretin releases fluid rich in enzymes.
    (B) Secretin releases fluid rich mainly in
    electrolytes and bicarbonate.
    (C) Cholecystokinin releases fluid,
    predominantly rich in electrolytes, and
    bicarbonate.
    (D) All pancreatic enzymes are secreted in
    an inactive form.
    (E) The pancreas produces proteolytic
    enzymes only.
A
  1. (B) Secretin releases fluid rich mainly in electrolytes and bicarbonate. Both cholecystokinin
    and vagal stimulation result in fluid with a
    high content of enzymes. Among the pancreatic
    enzymes, amylase and lipase are released in
    their active forms; whereas, the proteolytic
    enzymes (trypsinogen, chymotrypsinogen) are
    secreted as inactive zymogens. Their activation
    occurs in the duodenum, where the zymogens
    are exposed to enterokinase.
415
Q
  1. A48-year-old woman presents with severe recurrent peptic ulcer located in the proximal jejunum.
    Five years previously she underwent parathyroidectomy for hypercalcemia. Her brother was
    previously diagnosed as having Zollinger-Ellison
    syndrome. To confirm the diagnosis of ZollingerEllison syndrome, blood should be tested for
    levels of which of the following?
    (A) Parathyroid hormone
    (B) Histamine
    (C) Pepsin
    (D) Gastrin
    (E) Secretin
A
  1. (D) Zollinger-Ellison syndrome is caused by
    secretion of excessive amounts of gastrin by
    islet cells of the pancreas (gastrinoma). It
    should always be thought of in patients with
    peptic ulcer disease, whose ulcers are severe,
    refractory to management, recurrent or located
    distally, beyond the first part of the duodenum.
    Gastrin levels in the blood are increased
    markedly and can be raised further by secretin
    injection (paradoxical response). The source of
    gastrin level in the blood may arise from hyperplasia, adenoma, or most commonly carcinoma
    of the islets. Most gastrinomas are sporadic,
    but 25% of patients have a family history of
    multiple endocrine neoplasia.
416
Q
  1. A 50-year-old patient develops severe peptic
    ulcer disease that recurs despite gastric resection
    and vagotomy operations. She now presents
    with melena from a peptic ulcer located in the
    third part of the duodenum. To localize the
    gastrin-producing tumor, she should have which
    of the following?
    (A) CT scan of the abdomen
    (B) Ultrasound of the abdomen
    (C) SRS
    (D) MRI of the abdomen
    (E) Barium meal and follow through
A
43. (C) Because most gastrinomas are small, preoperative localization of the tumor may be difficult. A nuclear scan may be performed using
radiolabeled somatostatin (octreotide) analogue. This binds with the somatostatin receptors present on the gastrin-producing cells which
identifies the tumor. Endoscopic (not transcutaneous) ultrasound is also useful in localizing
these lesions in the pancreas and in the duodenum. The combined accuracy of SRS and
endoscopic ultrasound in preoperative localization of gastrinomas is 93%.
417
Q
  1. A 42-year-old accountant presents with recurrent RUQ pain of 3-year duration. He had
    undergone a laparoscopic cholecystectomy
    2-years ago for presumed symptomatic
    cholelithiasis, but the pain persisted. An upper
    GI endoscopy is normal. A sonogram and CT
    scan of the abdomen are normal. An ERCP is
    performed, and the pressure in the CBD is 45-
    cm saline (normal bile duct pressure is 10–18-
    cm saline). What is the most likely diagnosis?
    (A) Acalculous cholecystitis
    (B) Emphysematous cholecystitis
    (C) Biliary dyskinesia
    (D) Cancer of the gallbladder
    (E) Myasthenia gravis
A
  1. (C) Patients with biliary dyskinesia present
    with typical symptoms of gallstone disease,
    but investigations fail to reveal cholelithiasis
    or choledocholithiasis. Ironically, many patients
    will have undergone cholecystectomy for incidentally found gallstones but without relief of
    pain. ERCP with measurement of sphincter
    pressure will reveal basal sphincter pressure
    above 40 cm of water. Calcium channel blockers may be tried initially to relieve the spasm of
    the sphincter of Oddi, but many patients will
    require an endoscopic sphincterotomy.
418
Q
  1. In the emergency department, blood is taken
    from a 42-year-old man who presents with central abdominal pain of 12-hour duration. There
    is no history of alcohol abuse or gallstones. The
    serum is noted to be lactescent (milky appearance). To help elucidate the significance of the
    abdominal pain, which of the following tests
    should be requested?
    (A) Amylase
    (B) Hemoglobin electropheresis
    (C) Creatinine kinase MB (CK-MB)
    (D) Lipase
    (E) Calcium
A
  1. (D) Lactescent serum is sometimes seen soon
    after an acute attack of pancreatitis. Lipase level
    should be elevated to show pancreatitis as the
    cause of the abdominal pain. Hypertriglyceridemia artificially lowers serum amylase
    levels. If the blood specimen appears milky,
    the serum should be diluted; after dilution,
    serum amylase levels may become elevated.
    Other uncommon causes of pancreatitis
    include steroids, thiazide diuretics, lasix, sulfonamides, protein deficiency, hypercalcemia,
    familial, traumatic, idiopathic, and anatomic
    anomalies such as stricture, or pancreas divisum of the pancreatic duct.
419
Q
  1. A 67-year-old woman is evaluated for obstructive jaundice. The cholangiographic findings
    indicate that she has a cancer of the lower end
    of the CBD. Clinical examination would most
    likely reveal which of the following?
    (A) Enlarged gallbladder
    (B) Shrunken gallbladder
    (C) Enlarged pancreas
    (D) Shrunken pancreas
    (E) Palpable tumor
A
  1. (A) The gallbladder is enlarged (Courvoisier’s
    sign) in most cases of obstructive jaundice attributable to malignancy. In obstructive jaundice
    attributable to gallstones, the gallbladder is usually shrunken, owing to the previous inflammatory condition affecting the gallbladder.
420
Q
  1. A 73-year-old woman is evaluated for obstructive jaundice after an injury to the CBD, 7 months
    previously at laparoscopic cholecystectomy. The
    alkaline phosphatase is elevated. In obstructive
    jaundice, which of the following statements is
    true regarding alkaline phosphatase?
    (A) Its level increases before that of bilirubin.
    (B) Its level is unlikely to be increased in
    pancreatic malignancy.
    (C) Its elevation indicates bone metastasis.
    (D) Its elevation excludes hepatic metastasis.
    (E) Its level falls after that of the bilirubin,
    following surgical intervention.
A
  1. (A) Alkaline phosphatase level usually is more
    sensitive than the bilirubin level for indicating
    cholestatic jaundice. It also is more likely to fall
    before the bilirubin level when the obstruction
    has been relieved. If an unexplained alkaline
    phosphatase elevation exists (even in the presence of a normal bilirubin), biliary pathology
    must be excluded. Elevation of the alkaline
    phosphatase from a possible source in bone disease can be excluded by measuring isoenzymes.
421
Q
  1. A recently arrived emigrant from China
    develops jaundice, rigors, and high fever.
    Investigations revealed that he is suffering from
    oriental cholangiohepatitis. This condition is
    confirmed by detecting which of the following?
    (A) Schistosomiasis (Bilharzia) parasite
    (B) Ameba
    (C) Opisthorchis (Clonorchis) sinensis
    (D) Hydatid cyst (Echinococcus)
    (E) Hookworm
A
  1. (C) Oriental cholangiohepatitis is thought to be
    caused by the Chinese liver fluke (C. sinensis). It
    is encountered mainly in China (Canton) and
    Hong Kong, and among Chinese who have emigrated elsewhere. There are multiple strictures
    in the biliary tree, and the intrahepatic ducts
    are dilated. Secondary infection supervenes.
    Schistosomiasis causes liver fibrosis, ameba
    causes liver abscess. Echinococcus, causes hydatid
    liver cysts, and hookworm causes anemia.
422
Q
  1. A 48-year-old female travel agent presents with
    jaundice. Radiological findings confirm the
    presence of sclerosing cholangitis. She gives a
    long history of diarrhea for which she has
    received steroids on several occasions. She is
    likely to suffer from which of the following?
    (A) Pernicious anemia
    (B) Ulcerative colitis
    (C) Celiac disease
    (D) Liver cirrhosis
    (E) Crohn’s disease
A
  1. (B) Sclerosing cholangitis is rare and occurs
    mainly in the third and fourth decades of life.
    Unlike most autoimmune disorders, it affects
    men more commonly. It may occur without
    any other abnormal pathology or may be associated with ulcerative colitis or retroperitoneal
    fibrosis. The CBD is converted to a thickened
    cord whose lumen is almost completely obliterated. The prognosis is guarded, and the mean
    survival is only 5–6 years.
423
Q
  1. A 40-year-old man underwent laparoscopic
    cholecystectomy 2 years earlier. He remains
    asymptomatic until 1 week before admission,
    when he complains of RUQ pain and jaundice.
    He develops a fever and has several rigor
    attacks on the day of admission. An ultrasound
    confirms the presence of gallstones in the distal
    CBD. The patient is given antibiotics. Which
    of the following should be undertaken as the
    next step in therapy?
    (A) Should be discharged home under
    observation
    (B) Should be observed in the hospital
    (C) Undergo surgical exploration of the CBD
    (D) ERCP with sphincterotomy and stone
    removal
    (E) Anticoagulants
A
  1. (D) The patient described has the features of
    Charcot’s triad-jaundice, abdominal pain, and
    rigors, which indicates the presence of ascending
    cholangitis in a patient with obstructive jaundice. The patient should be treated with broad
    spectrum IV antibiotics and undergo ERCP,
    sphincterotomy, and stone extraction. If this fails,
    surgical exploration of the CBD will be required.
424
Q
  1. A 43-year-old woman undergoes open cholecystectomy. Intraoperative cholangiogram
    revealed multiple stones in the CBD. Exploration
    of the CBD was performed to extract gallstones.
    The CBD was drained with a #18 T-tube. After 10
    days, a T-tube cholangiogram reveals a retained
    CBD stone. This should be treated by which of
    the following?
    (A) Laparotomy and CBD exploration
    (B) Subcutaneous heparinization
    (C) Antibiotic therapy for 6 months and
    then reevaluation
    (D) Extraction of the stone through the
    pathway created by the T-tube (after
    6 weeks)
    (E) Ultrasound crushing of the CBD stone
A
  1. (D) If a stone is detected, the T tube should be left
    in place for 6 weeks to allow the tract to mature.
    At this time, the T tube can be removed, and the
    stone can be extracted by using a Dormia basket
    under fluoroscopy. This approach is indicated
    only when a T-tube larger than 16 has been
    inserted. If this approach is not feasible, the stone
    can be extracted by retrograde endoscopic techniques or CBD exploration.
425
Q
  1. A 62-year-old woman who underwent cholecystectomy and choledochoduodenostomy
    (CBD duodenal anastomosis) 5 years previously is admitted to the hospital with a 3-day
    history of upper abdominal pain, chills, fever,
    and dark urine. These symptoms are suggestive of ascending cholangitis.What is the laboratory finding that supports a diagnosis of
    ascending cholangitis?
    (A) Amylase elevation with normal findings
    on liver studies
    (B) Alkaline phosphatase elevation with
    normal or elevated normal bilirubin
    levels
    (C) Elevated serum glutamic oxaloacetic
    transaminase (SGOT) levels
    (D) Altered urea/creatinine ratio
    (E) Urobilin in urine
A
  1. (B) In the presence of previous gallbladder surgery, the possibility of cholestatic jaundice must
    be excluded. Elevation of alkaline phosphatase
    (with normal or elevated bilirubin level)
    strongly supports this diagnosis. The dark urine results from increase in conjugated bilirubin
    (regurgitated jaundice). Urobilin is excreted in
    the urine in hepatocellular jaundice but is
    absent in the urine in obstructive jaundice,
    because this pigment forms only if bile reaches
    the small intestine.
426
Q
  1. A 70-year-old male underwent a choledochoduodenostomy for multiple common duct
    stones. The patient now presents with RUQ
    abdominal pain. What should be the initial test
    (least invasive with the best yield) to determine
    patency of the choledochoduodenostomy?
    (A) ERCP
    (B) Percutaneous transhepatic
    cholangiogram (PTC)
    (C) HIDA scan
    (D) CT scan of the abdomen
    (E) Ultrasound of the abdomen
A
  1. (C) A HIDA scan will show excretion of the
    radiolabeled isotope into the biliary tree, but
    there will be no flow into the duodenum, indicating that the biliary-enteric anastomosis is
    occluded. If an upper GI study with barium is
    performed, visualization of the common bile
    duct would indicate patency of the choledochoduodenal anastomosis.
427
Q
54. An 70-year-old male presents with a clinical
diagnosis of acute cholangitis. Which organism is most likely involved in the pathogenesis
of ascending cholangitis?
(A) Clonorchis sinensis
(B) Escherichia coli
(C) Salmonella
(D) Staphylococcus aureus
(E) Clostridia
A
  1. (B) Gram-negative bacilli including E. coli,
    Klebsiella, and Proteus are the organisms most
    commonly involved in ascending cholangitis.
    Anaerobic bacteroids should also be excluded,
    especially in elderly patients. Intravenous
    hydration and early institution of appropriate
    antibiotics is indicated. The antibiotics selected
    should be effective against the isolated organisms. Combined therapy with an aminoglycoside, penicillin, and an antibiotic targeted
    specifically against anaerobic organisms should
    be administered initially until blood culture
    results are available.
428
Q
  1. Following admission to the hospital for intestinal obstruction, a 48-year-old woman states
    that she previously had undergone cholecystectomy and choledochoduodenostomy. The
    most likely indication for the performance of
    the choledochoduodenostomy was:
    (A) Hepatic metastasis were present.
    (B) Multiple stones were present in the
    gallbladder at the previous operation.
    (C) Multiple stones were present in the CBD
    at the previous operation.
    (D) The common hepatic duct had a
    stricture.
    (E) The small intestine was occluded.
A
  1. (C) Multiple stones were present in the CBD at
    the previous operation. During exploration of
    the CBD, most stones can be removed by using
    Desjardin’s forceps or under direct vision using
    a choledochoscope and Dormia basket.
    However, if there are multiple stones impacted
    in the lower part of the CBD, a drainage procedure may be indicated. The CBD must be
    dilated before considering performing a choledochoduodenostomy at the time of gallbladder
    surgery (Fig. 7–7). If a stone is present in a
    dilated CBD after previous cholecystectomy, a
    choledochoduodenostomy is performed,
    because the rate of recurrent jaundice is high
    (>20%). Alternatively ERCP and sphincterotomy could be considered.
429
Q
  1. In attempting to minimize complications
    during cholecystectomy, the surgeon defines
    the triangle of Calot during the operation. The
    boundaries of the triangle of Calot (modified)
    are the common hepatic duct medially, the
    cystic duct inferiorly, and the liver superiorly.
    Which structure courses through this triangle ?
    (A) Left hepatic artery
    (B) Right renal vein
    (C) Right hepatic artery
    (D) Cystic artery
    (E) Superior mesenteric vein
A
  1. (D) The cystic artery courses through the triangle of Calot. The identification of the triangle
    is therefore important in the performance of a
    cholecystectomy.
430
Q
  1. A 64-year-old man complains of abdominal
    pain, pruritus, 4-lb weight loss, and anorexia.
    There are multiple scratch marks on the skin
    of the extremities and flank. The bilirubin is
    1.0 mg/dL. To determine if the condition is
    due to cholestasis, blood should be tested for
    which of the following?
    (A) Direct and indirect bilirubin
    (B) Alkaline phosphatase
    (C) Serum glutamic-oxaloacetic
    transaminase (SGOT)
    (D) Serum glutamic-pyruvic transaminase
    (SGPT)
    (E) Bile pigments
A
  1. (B) Pruritus occurs frequently in untreated
    obstructive jaundice. Bile salt elevation is a possible cause of pruritus. Patients with generalized pruritus should have alkaline phosphatase
    levels determined; if levels are elevated, the
    possibility of cholestatic jaundice should be
    considered. Bilirubin is not always elevated in
    obstructive jaundice.
431
Q
  1. A 49-year-old African American woman born
    in New York is admitted with RUQ pain, fever,
    and jaundice (Charcot’s triad.) A diagnosis of
    ascending cholangitis is made. With regard to
    the etiology of ascending cholangitis, which of
    the following is TRUE?
    166 7: Pancreas, Biliary Tract, Liver, and Spleen
    (A) It usually occurs in the absence of
    jaundice.
    (B) It usually occurs secondary to CBD
    stones.
    (C) It occurs frequently after
    choledochoduodenostomy.
    (D) It does not occur in patients with
    cholangiocarcinoma.
    (E) It is mainly caused by the liver fluke.
A
  1. (B) Any obstruction to the biliary tree (stones
    and benign, malignant, or anastomotic strictures) can lead to infection and cholangitis. It
    may also occur after trauma to the biliary tree.
    In ascending cholangitis, there is fever, jaundice, and rigors (Charcot’s triad). Suppurative
    cholangitis is suspected when additional signs
    of deterioration in mental status and hypotension are present in addition (Reynold’s pentad).
    This entity requires immediate biliary decompression either endoscopically or surgically. C.
    sinensis, the liver fluke, causes suppurative
    cholangitis in the Far East.
432
Q
  1. A 43-year-old man is admitted with jaundice of
    6-week duration. An ultrasound shows multiple small stones in the gallbladder and the presence of a CBD stone. A preoperative ERCP
    followed by a laparoscopic cholecystectomy is
    planned. The international normalization ratio
    (INR) is elevated to 3.1 What is the next step in
    management?
    (A) Infusion of cryoprecipitate
    (B) Oral vitamin K tablets to decrease
    prolonged INR
    (C) Parenteral vitamin K to decrease
    prolonged INR
    (D) Demonstration that urobilinogen is
    increased in the urine
    (E) Demonstration that stercobilinogen is
    increased in the stool
A
  1. (C) Vitamin K requires bile salts for efficient
    absorption from the gut, as do the other fatsoluble vitamins—A, D, and E. Therefore, the
    oral route is not suitable to administer patients
    with obstructive jaundice. If intramuscular
    vitamin K is given, correction will occur if there
    has been no hepatocellular damage. When
    emergency surgery is required in this circumstance, the coagulation defect due to hepatic
    disease may be corrected with fresh-frozen
    plasma (FFP). Urobilinogen usually is absent in
    the urine in obstructive jaundice, because its
    presence depends on a patent biliary–enteric
    circulation. Stercobilinogen will be absent in
    fecal examination.
433
Q
  1. A 65-year-old woman is admitted with RUQ
    pain radiating to the right shoulder, accompanied by nausea and vomiting. Examination
    reveals tenderness in the RUQ and a positive
    Murphy’s sign. A diagnosis of acute cholecystitis is made. What is the most likely finding?
    (A) Serum bilirubin levels may be elevated.
    (B) Cholelithiasis is present in 40–60%.
    (C) Bacteria are rarely found at operation.
    (D) An elevated amylase level excludes this
    diagnosis.
    (E) A contracted gallbladder is noted on
    ultrasound.
A
  1. (A) Stones are found in the gallbladder in over
    90% of patients with cholecystitis. Bacteria are
    cultured in bile in about half the patients undergoing surgery; however, many patients have
    previously received antibiotics. The gallbladder
    is usually distended in patients with acute cholecystitis but contracted in chronic cholecystitis.
434
Q
  1. A surgeon is removing the gallbladder of a
    35-year-old obese man. One week previously
    the patient had recovered from obstructive
    jaundice and at operation, numerous small
    stones are present in the gallbladder. In addition to cholecystectomy, the surgeon should
    also perform which of the following?
    (A) Intraoperative cholangiogram
    (B) Liver biopsy
    (C) No further treatment
    (D) Removal of the head of the pancreas
    (E) CBD exploration
A
  1. (A) If there is a recent history of jaundice,
    although the CBD is not dilated, intraoperative
    cholangiography must be performed to exclude
    CBD stones. Other indications for intraoperative cholangiogram include a recent history of
    ascending cholangitis, dilated CBD on preoperative sonogram, or suspicion of a “missing”
    stone in the gallbladder (i.e., as detected by
    ultrasound or other observations). Elevated
    bilirubin and alkaline phosphatase are other
    indications that a CBD stone may be present.
435
Q
  1. A 42-year-old man presents with recurrent
    RUQ pain for 2 years. A sonogram is negative
    for gallstones, and the CBD is normal. An
    upper GI endoscopy is also normal, and there
    is no peptic ulcer disease. Biliary dyskinesia is
    suspected, and the patient undergoes further
    evaluation. Which of the following will stimulate contraction of the gallbladder?
    (A) Cholecystokinin
    (B) Vagal section
    (C) Secretin
    (D) Epinephrine
    (E) Gastrin
A
  1. (A) A cholecystokinin stimulated HIDA scan
    should be performed. Failure of the gallbladder
    to contract after stimulation by cholecystokinin
    may suggest dyskinesia. This is an indication
    for cholecystectomy, even though stones are
    not demonstrated. Secretin is the duodenal hormone that stimulates exocrine pancreatic secretion. Gastrin, released mainly from the antrum,
    increases gastric acid secretion that is high in
    bicarbonate and electrolytes.
436
Q
  1. A 57-year-old previously healthy business executive presents with gradually increasing obstructive jaundice. An ultrasound of the liver shows
    dilated intrahepatic ducts, but the CBD is
    normal. An ERCP shows a filling defect at the
    level of the common hepatic duct. Endoscopic
    brush biopsies are taken, and histology confirms
    cholangiocarcinoma. In discussing these findings, the surgeon should inform the patient that
    (A) This tumor affects men more commonly
    than women.
    (B) The tumor is a result of gallstones.
    (C) The tumor is best treated with a stent to
    relieve obstructive jaundice.
    (D) Weight loss is common in this condition.
    (E) The most common location of these
    tumors is at the ampulla of Vater.
A
  1. (A) Unlike most biliary disease conditions,
    cholangiocarcinoma condition affects men more
    commonly than women. Primary sclerosing
    cholangitis, C. sinensis, and choledochal cysts
    may play an etiological role in some cases, but
    gallstones are not involved in the pathogenesis
    of this tumor. Patients present with obstructive
    jaundice; pain, and weight loss are less common.
    Proximal tumors (Klatskin) are most common,
    and they require excision of hepatic duct bifurcation and reconstruction with a Roux-en-Y limb
    of jejunum. Tumors of the distal duct can be
    resected by performing a Whipple pancreatoduodenectomy. Patients who are not operative
    candidates (those with advanced disease or
    those who cannot withstand a major operation)
    should undergo palliative endoscopic stent
    placement to relieve the obstruction.
437
Q
  1. A 38-year-old male lawyer develops abdominal
    pain after having a fatty meal. Examination
    reveals tenderness in the right hypochondrium
    and a positive Murphy’s sign. Which test is
    most likely to reveal acute cholecystitis?
    (A) HIDA scan
    (B) Oral cholecystogram
    (C) Intravenous cholangiogram
    (D) CT scan of the abdomen
    (E) ERCP
A
  1. (A) The HIDA scan is most accurate in establishing a diagnosis of acute cholecystitis. After
    injection, the technetium-labeled imminodiacetic acid radioisotopes are taken up by the
    liver and excreted into the biliary tree. If the
    cystic duct is obstructed (as in patients with
    acute cholecystitis) the gallbladder will not
    be visualized. Ultrasound may show ductal
    dilation, the presence of wall thickening (<3
    mm), or pericholecystic fluid, which is highly
    suggestive of acute cholecystitis.
438
Q
  1. A 55-year-old white female undergoes a laparoscopic cholecystectomy for symptomatic
    cholelithiasis. The operation went well, and the
    patient was discharged home. One week later,
    she comes to your office for a routine postoperative follow-up. The final pathology report
    shows an incidental finding of a gallbladder carcinoma confined to the mucosa. In further advising the patient, you should inform her that
    (A) She should undergo radiation therapy.
    (B) She should undergo right hepatectomy
    to remove locally infiltrating disease.
    (C) She should undergo regional
    lymphadenectomy.
    (D) She requires systemic chemotherapy.
    (E) She does not require any further
    therapy.
A
  1. (E) She does not require any further therapy. In
    instances where gallbladder carcinoma is discovered incidentally during cholecystectomy
    and is shown to have only invaded the mucosa
    and submucosa it is classified as stage I. The
    5-year survival for these patients is 100% and
    no further treatment is required as for more
    advanced lesions, that is, those penetrating the
    muscular layer or with lymph node involvement (stages II & III). Here there is a higher
    incidence of local and regional spread to the
    liver and porta hepatis lymph nodes, respectively. For these patients an en bloc resection of
    segments 4 and 5 of the liver is performed
    along with dissection of celiac axis and porta
    hepatis lymph nodes. For more advanced
    lesions (stage IV), the prognosis is very poor,
    and further resection is not indicated.
    Gallbladder carcinoma responds poorly to
    radiotherapy or chemotherapy.
439
Q
  1. A 49-year-old man who recovered 7 years ago
    from acute viral hepatitis develops chronic
    active hepatitis and liver cirrhosis. He is seen in
    the office without any abdominal symptoms.
    An ultrasound reveals cholelithiasis and
    ascites. What treatment should be instituted?
    (A) He should undergo percutaneous
    dissolution of stones.
    (B) He should undergo cholecystectomy.
    (C) He should undergo cholecystostomy.
    (D) He should be placed on a diet that
    avoids fatty foods and discouraged
    from undergoing elective
    cholecystectomy.
    (E) He should be treated with
    ursodeoxycholic acid.
A
  1. (D) The morbidity and mortality of cholecystectomy is markedly increased in the presence
    of cirrhosis. The prognosis is particularly grave
    in patients with decompensated liver disease.
    Most gallstones in patients with cirrhosis are
    pigment stones, and hence dissolution with
    ursodeoxycholic acid is not an acceptable
    option of treatment.
440
Q
  1. A 48-year-old man is admitted to the hospital
    with severe abdominal pain, tenderness in the
    right hypochondrium, and a WBC count of
    12,000. A HIDA scan fails to show the gallbladder after 4 hours. Acute cholecystitis is
    established. After diagnosis, cholecystectomy
    should be performed within which of the
    following?
    (A) 3–60 minutes
    (B) The first 2–3 days following hospital
    admission
    (C) 8 days
    (D) 3 weeks
    (E) 3 months
A
  1. (B) Early after the onset of acute cholecystitis,
    the plane of dissection may be facilitated
    because of early inflammatory response.
    Between the seventh and fourteenth day after
    admission, surgery may be extremely difficult
    because of resolving infection and adhesions.
    Where possible surgery should be avoided
    during this period.
441
Q
  1. A 60-year-old diabetic man is admitted to the
    hospital with a diagnosis of acute cholecystitis.
    The WBC count is 28,000, and a plain film of
    the abdomen and CT scan show evidence of
    intramural gas in the gallbladder. What is the
    most likely diagnosis?
    (A) Emphysematous gallbladder
    (B) Acalculous cholecystitis
    (C) Cholangiohepatitis
    (D) Sclerosing cholangitis
    (E) Gallstone ileus
A
  1. (A) Emphysematous cholecystitis is caused by
    gas-forming organisms. On a plain x-ray of the
    abdomen, gas may be seen within the wall of the
    gallbladder. Clinically, the patient has rapidly
    progressive sepsis, RUQ pain, fever, and hemodynamic instability. The disease primarily affects
    diabetic men. Treatment with laparotomy and cholecystectomy is urgent to avoid complications. Air within the biliary tree (not gallbladder
    wall) may be seen in gallstone ileus, after biliaryenteric anastomosis or after sphincterotomy.
442
Q
  1. A60-year-old woman is recovering from a major
    pelvic cancer operation and develops severe
    abdominal pain and sepsis. Following a positive
    HIDA scan, laparotomy is performed. The gallbladder is severely inflamed and removed. There
    is no evidence of gallbladder stones (acalculous
    cholecystitis). Cholecystectomy is performed.
    Which is true of acalculous cholecystitis?
    (A) It is usually associated with stones in
    the CBD.
    (B) It occurs in 10–20% of cases of
    cholecystitis.
    (C) It has a more favorable prognosis than
    calculous cholecystitis.
    (D) It is increased in frequency after trauma
    or operation.
    (E) It is characterized on HIDA scan by
    filling of the gallbladder.
A
  1. (D) Acute acalculous cholecystitis is most commonly encountered in critically ill patients after
    trauma, other unrelated surgical operations,
    burns, sepsis, and multiorgan failure. The
    HIDA scan fails to visualize the gallbladder,
    and a sonogram may show a distended gallbladder with wall thickening and pericholecystic fluid. Acalculous cholecystitis carries a
    mortality rate of 10–30%. Delay in diagnosis
    and hence treatment is accompanied by severe
    complications, such as gangrene and perforation of the gallbladder in a patient who usually
    has other debilitating illnesses.
443
Q
  1. Following recovery in the hospital from a fracture of the femur, a 70-year-old nursing home
    female patient develops RUQ abdominal pain
    and fever. She has tenderness in the right subcostal region. There is evidence of progressive
    sepsis and hemodynamic instability. The WBC
    count is 24,000. A bedside sonogram confirms
    the presence of acalculous cholecystitis. What
    should treatment involve?
    (A) Intravenous antibiotics alone
    (B) ERCP
    (C) Percutaneous drainage of the
    gallbladder
    (D) Urgent cholecystectomy
    (E) Elective cholecystectomy after 3 months
A
  1. (C) Obstruction of the cystic duct may be caused
    by factors other than stones. Acalculous cholecystitis carries a high mortality, because it
    occurs in patients who are already critically ill.
    Furthermore, the establishment of the diagnosis
    is often delayed. Urgent cholecystostomy should
    be performed. In recent years, percutaneous
    cholecystostomy under CT or ultrasonography
    (US) guidance is performed more commonly
    than surgical cholecystectomy, because it carries
    a lower operative mortality rate. In a stable
    patient, cholecystectomy may be considered.
444
Q
  1. In designing a study related to gallbladder
    function, it should be noted that the healthy
    gallbladder mucosa selectively absorbs which
    of the following?
    (A) Bile pigment
    (B) Bile salts
    (C) Cholesterol
    (D) Sodium
    (E) Free fatty acids
A
  1. (D) Sodium chloride and water are selectively
    absorbed by the gallbladder mucosa. Bile salts
    and pigments are concentrated in the bile.
    Mucus also is secreted into the bile to function
    in a protective capacity. The presence of cholesterol crystals in biliary drainage material
    warrants further investigation, although the
    biliary system is normal.
445
Q
  1. On a recent safari in Africa, a 39-year-old male
    engineer developed an acute diarrhea state
    requiring hospitalization and treatment with
    Flagyl. Six weeks after his return, he developed
    RUQ pain, fever and chills. Achest x-ray showed
    elevation of the right hemidiaphragm, and sonogram showed a large abscess in the right lobe of
    the liver.Which of the following statements is
    TRUE regarding this disease process?
    (A) Satisfactory treatment is not readily
    available.
    (B) Diagnosis is easily made by finding
    Entamoeba histolytica in stools in nearly
    all patients.
    (C) Bloody diarrhea is always present.
    (D) Anchovy-paste pus is usually present in
    the abscess cavity.
    (E) Extensive surgical drainage is usually
    indicated.
A
  1. (D) This patient has an amebic liver abscess. In
    most patients, the antecedent intestinal phase
    has subsided by the time patient presents with
    fever, chills, and a painful, tender enlarged liver.
    Amoebae are found in examination of fresh
    stools in 15% of cases, but the indirect hemagglutination test is almost always positive.
    Amebic abscess responds rapidly to treatment
    with metronidazole (Flagyl). Surgery should be
    avoided when possible and is indicated only
    when medical treatment has failed or complications, such as perforation, have occurred.
446
Q
  1. A 45-year-old male is suspected of having an
    amebic abscess of the liver. Serum bilirubin is
    mildly elevated. The WBC is 11,000 but there is
    eosinophilia. The initial line of treatment
    involves which of the following?
    (A) Cortisone
    (B) Metronidazole (Flagyl)
    (C) Surgical excision
    (D) Sulfonamides and penicillin
    (E) Colon resection
A
  1. (B) Amebic liver abscess almost always
    responds to treatment with metronidazole
    (Flagyl). Occasionally, percutaneous aspiration
    is required when there is no response to Flagyl
    or if the abscess is secondarily infected. Amebic
    lever abscess affects mainly middle-aged men.
    Complications of amebic liver abscess include
    secondary infection in 20% and rupture into
    pleural, peritoneal, or pericardial cavity in 10%
    of cases.
447
Q
  1. In performing hepatic resection, a knowledge of
    the different lobes and segments of the liver is
    mandatory. The right and left lobes of the liver
    are separated by an imaginary plane (Cantlie’s
    line) that passes between the the inferior vena
    cava (IVC) and which of the following?
    (A) Portal vein
    (B) Falciform ligament
    (C) Left margin of the quadrate lobe
    (D) Gallbladder
    (E) Left margin of the caudate lobe
A
  1. (D) The hepatic artery, portal vein, and hepatic
    bile duct are distributed equally between both
    lobes of the liver divided by Cantlie’s line. This
    line passes between the inferior vena cava posteriorly and the gallbladder fossa anteroinferiorly. The falciform ligament does not divide
    the liver into a right and left lobe; it divides the
    true left lobe into medial and lateral segments.
    The caudate and quadrate lobes are part of the
    left lobe, and, thus, Cantlie’s line passes along
    their right (and not left) margins.
448
Q
  1. A 32-year-old diabetic woman who has taken
    contraceptive pills for 12 years develops RUQ
    pain. CT scan of the abdomen reveals a 5-cm
    hypodense lesion in the right lobe of the liver
    consistent with a hepatic adenoma. What
    should the patient be advised to do?
    (A) Undergo excision of the adenoma
    (B) Stop oral contraceptives only
    (C) Stop oral hypoglycemic medication
    (D) Undergo right hepatectomy
    (E) Have serial CT scans every 6 months
A
  1. (A) Hepatic adenomas are associated with an
    increased incidence in patients receiving oral
    contraceptives, diabetes, and pregnancy. Most
    patients are symptomatic with pain and bleeding. Because of the real risk of intraperitoneal or
    intratumoral bleeding as well as malignant transformation, excision of the adenoma is recommended. Tumors are removed by enucleation or
    with a narrow rim of normal parenchyma, and
    major liver resection is not required.
449
Q
  1. A 35-year-old woman is seen in the office with
    focal nodular hyperplasia. This condition is similar to hepatic adenoma, in that it does what?
    (A) Frequently causes symptoms
    (B) Tends to lead to liver rupture
    (C) LFT and alpha fetoprotein (AFP) are
    normal
    (D) Easily detected by CT scan of the liver
    (E) Tends to undergo malignant changes
A
  1. (C) Unlike hepatic adenomas, these lesions do
    not usually cause symptoms. Unlike hepatic
    adenomas, focal nodular hyperplasia does not
    tend to cause intramural bleeding with rupture into the peritoneal cavity. CT or US scan
    may frequently miss the lesion, because it is so
    dense. There is no definite relationship with
    oral contraceptives. Focal nodular hyperplasia
    lesions are not well encapsulated and have a
    central stellate scar. Malignant changes have
    not been reported. LFT and AFP are normal in
    both conditions.
450
Q
  1. A 64-year-old man has mild upper abdominal
    pain. On contrast CT scan, a 5-cm lesion in the
    left lobe of the liver enhances and then decreases
    over a 10-minute period from without to within.
    The most likely lesion is which of the following?
    (A) Congenital cyst
    (B) Hemangioma
    (C) Fungal abscess
    (D) Focal nodular hyperplasia
    (E) Hepatic adenoma
A
  1. (B) Hemangioma is the commonest nodule in
    the liver. On intravenous contrast CT or MRI, a
    liver hemangioma shows initial centripetal
    enhancement followed by decrease in dye
    over 10 minutes from without to within.
    Hemangiomas occur more frequently in women.
    Most lesions are asymptomatic, discovered incidentally, and require no treatment. Larger
    hemangiomas may cause pain because of stretching of liver capsule or thrombocytopenia due to
    platelet trapping. These tumors may occasionally
    require resection.
451
Q
  1. A 16-year-old previously healthy male fell off
    his bicycle while riding back home from school.
    On examination there was mild tenderness in
    the RUQ. No other abnormality was detected.
    A sonogram showed a large solitary hypoechogenic cyst in the liver. The LFTs are normal,
    and there is no family history of cystic disease
    involving solid organs. What is the most likely
    cause?
    (A) Fungal abscess
    (B) Trauma
    (C) Developmental
    (D) Neoplastic
    (E) Pyogenic abscess
A
  1. (C) Congenital cysts are more frequently
    encountered than those that are acquired,
    which are caused by trauma, inflammation, or
    parasitic disease. Most congenital cysts tend to
    be asymptomatic and require no treatment.
    Larger cysts may cause pain and occasionally
    require radiologically guided percutaneous
    drainage or operative unroofing to prevent
    recurrence. Fungal abscesses, encountered
    mainly in immunosupressed patients, tend to
    be multiple. Pyogenic abscesses tend to be
    symptomatic with fever and pain, whereas,
    tumors are generally not hypoechoic.
452
Q
  1. A healthy 64-year-old woman had a cancer of
    the left colon resected 4 years previously.
    During follow-up, an increased carcinoembryonic antigen (CEA) level lead to a CT scan of
    the abdomen, which revealed two discrete
    lesions in the left lateral lobe of the liver. Liver
    biopsy confirms that this is metastatic colon
    cancer. What is the most appropriate plan?
    (A) Inform the patient that there is no treatment, and that her expectation of life is
    limited.
    (B) Irradiation is recommended.
    (C) Local cauterization of the cancer is
    recommended.
    (D) Liver resection is recommended.
    (E) Chemotherapy is recommended.
A
  1. (D) Before performing the left hepatic lobectomy, any extrahepatic metastasis should be
    ruled out. If lung, bone, adrenal, or skin metastasis were present, then subjecting the patient
    to a major operation would not be warranted in
    most cases. Moreover, before proceeding with
    surgery, it must be ascertained that control of
    the primary tumor has been achieved and that
    the patient’s physical condition will allow such
    a major operation. Surgical excision of hepatic
    metastasis results in 25%, 5-year survival.
    Patients not treated by hepatic resection do not
    usually survive into the first year after clinical
    detection. Chemotherapy would be offered if
    resection were not indicated.
453
Q
  1. A 42-year-old man undergoes a liver transplantation. There is rapid deterioration after the completion of the graft, and the patient dies within
    12 hours. What is the most likely cause of death?
    (A) Massive pulmonary embolus
    (B) Graft rejection
    (C) Fat embolus
    (D) Massive hemorrhage
    (E) Subphrenic abscess
A
  1. (D) Massive hemorrhage. Hemorrhage is a
    major cause of death after liver transplantation. Subphrenic infection and other intraabdominal and intrahepatic infections may
    occur later in the postoperative period. Graft
    rejection is mainly a problem at a later period.
454
Q
  1. In discussing the treatment of a 42-year-old
    man with severe liver cirrhosis, the possibility
    of heterotopic transplantation is considered.
    Which statement about heterotopic liver transplantation is TRUE?
    (A) It implies removal of the recipient’s
    liver.
    (B) It is preferable to orthotopic liver
    transplantation.
    (C) It should be done in the iliac vessels.
    (D) It is rarely associated with long-term
    survival.
    (E) Heterotopic auxiliary liver transplants
    require high-out flow pressures.
A
  1. (D) Is rarely associated with long-term survival.
    Heterotopic (to a remote position) auxiliary
    transplantation is only occasionally indicated
    where orthotopic transplantation cannot be carried out. Long-term survival with this procedure is limited (2 of 69 cases). Hetrotopic
    auxiliary liver transplants require low outflow
    pressure and are, therefore, most likely to succeed if placed proximally as close to the heart as
    possible. One advantage of this procedure is
    that the procedure is technically easier, because
    the patient’s liver is not disturbed.
455
Q
  1. A 43-year-old man develops chronic hepatitis,
    which was attributed to a complication resulting from multiple blood transfusions for sickle
    cell anemia. He complains of chronic sweating,
    palpitation, and hunger attacks. What would
    be the most likely cause of these symptoms?
    (A) Hepatogenic hypoglycemia
    (B) Hemolytic anemia
    (C) Jaundice
    (D) Spontaneous hyperglycemia
    (E) Elevated bile salts in the blood
A
  1. (A) The liver plays a role in glucose formation
    from various glucogenic amino acids and other
    substances. Hepatic disease removes this
    source of glucose supply. In insulin hypoglycemia, there is enhanced rapid uptake of
    glucose by fat tissue and muscle.
456
Q
  1. A 42-year-old man is admitted with bleeding
    from esophageal varices. Investigation reveals
    that he has an occlusion of the portal vein.
    There is no evidence of liver cirrhosis. Which
    test will most likely reveal an underlying predisposing factor for this condition?
    (A) Hepatitis screening
    (B) Isoamylase
    (C) Intravenous pyelogram to exclude
    hydronephrosis
    (D) Coagulation tests to include
    antithrombin III
    (E) CT of abdomen
A
  1. (D) Antithrombin III counteracts excess of
    thrombin formation. The excess of thrombin
    facilitates conversion of fibrinogen to fibrin.
    Portal vein thrombosis will lead to portal
    hypertension but not hepatic congestion, as
    seen in Budd-Chiari syndrome. Portal vein
    thrombosis may occur in cirrhosis, trauma, in
    patients on contraceptive tablets, and in those
    who have an increased propensity for thrombus formation. It is also a direct complication of
    periumbilical infection in the neonate.
457
Q
  1. A9-year-old girl had multiple episodes of upper
    GI bleeding. Contrast enhanced CT scan showed
    multiple cavernous malformation surrounding
    the portal vein (Fig. 7–2). She is admitted with
    severe hematemesis and melena. At birth, she
    had developed an infection around the umbilicus. What is the most likely site of bleeding?
    (A) Meckel’s diverticulum
    (B) Esophageal varices
    (C) Peptic ulcer
    (D) Duodenal varices
    (E) Mallory-Weiss tear of the lower end of
    the esophagus
A
  1. (B) Umbilical infection at birth is associated with
    ascending infection along the remnant of the
    left umbilical vein in the round ligament. This
    vein communicates with the left portal vein.
    Portal hypertension occurs because of portal
    vein thrombosis. In general, LFTs are normal,
    because the site of portal obstruction is outside
    the liver. Other causes of portal vein thrombosis
    include chronic pancreatitis, carcinoma of the
    pancreas, surgical intervention in this region,
    and diseases associated with an increased tendency toward clot formation. (See Answer 65.)
458
Q
  1. A 49-year-old man with a history of cirrhosis is
    admitted with significant hematemesis. There
    is jaundice and clubbing of the fingers. His
    extremities are cold and clammy, and the systolic blood pressure drops to 84 mm Hg. The
    initial step in the management is to proceed
    with which of the following?
    (A) Urgent endoscopy and sclerotherapy
    (B) Sengstaken-Blakemore tube
    (C) Infusion of intravenous crystalloids
    (D) Intravenous pitressin
    (E) Surgery to stop bleeding
A
  1. (C) As in any patient with upper GI bleeding,
    the initial intervention following clinical evaluation requires appropriate resuscitation.
    Blood transfusion may be required. Liver functions must be assessed, and coagulopathy should be corrected with FFP or vitamin K
    injection. After resuscitation is completed,
    every attempt should be made to perform an
    upper GI endoscopy as soon as possible. These
    patients may be bleeding from varices, portal
    hypertensive gastropathy, peptic ulcer, or
    Mallory-Weiss tear, and early endoscopy will
    provide a higher diagnostic yield as to which
    lesion is actually bleeding.
459
Q
  1. A 42-year-old woman with a known history of
    esophageal varices secondary to hepatitis and
    cirrhosis is admitted with severe hematemesis
    from esophageal varices. Bleeding persists after
    pitressin therapy. What would the next step in
    management involve?
    (A) Emergency portacaval shunt
    (B) Emergency lienorenal shunt
    (C) Insertion of Sengstaken-Blakemore tube
    (D) Vagotomy
    (E) Transjugular intrahepatic portasystemic
    shunt (TIPS)
A
  1. (E) TIPS refers to an implantable, expandable
    metal stent placed radiologically through the
    hepatic parenchyma to establish a tract
    between the hepatic and portal vein. A portal
    systemic shunt is, therefore, created, and the
    varices are decompressed. Because of the high
    incidence of complications (esophageal perforation, aspiration, airway obstruction) associated with the Sengstaken-Blakemore tube, it is
    only used as a last ditch attempt to control
    exsanguination. In >50% of cases, bleeding
    recurs after the tube is deflated.
460
Q
  1. A 12-year-old boy who underwent a previous
    splenectomy for thalassemia presents to the
    emergency room with fever, chills, and septic
    shock. The parents give a history of seemingly
    minor sore throat, which started only a few
    hours previously. The child is hypotensive and
    appears moribund. A diagnosis of overwhelming postsplenectomy infection (OPSI) is made.
    Which of the following statements about OPSI
    is TRUE?
    (A) The condition is more common in
    children.
    (B) The condition is more common after
    splenectomy for trauma.
    (C) Prophylactic antibiotics have not been
    shown to improve outcome in children.
    (D) Prophylactic vaccination against
    Enterococcus should be performed.
    (E) The condition is very common after
    splenectomy.
A
  1. (A) The condition is more common in children.
    Splenectomy predisposes the patient to OPSI
    characterized by fulminant bacteremia, meningitis, or pneumonia. The mortality of this condition is high. The risk is greatest in children under
    4 and for those undergoing splenectomy for thalassemia or lymphoma. The risk is lower in
    adults and those undergoing splenectomy for
    trauma than for ITP. All patients undergoing
    elective splenectomy should receive vaccination
    against pneumococcus and H. Influenzae about
    2 weeks before surgery. Vaccination should be
    repeated every 5 years. In addition, children
    should be given penicillin prophylactically until
    they are 18 years of age. Postsplenectomy
    patients should seek medical attention at the first
    sign of even seemingly mild upper respiratory
    tract infection and should be advised to wear a
    Medic Alert tag indicating their asplenic state.
461
Q
  1. A 43-year-old man with chronic hepatitis and
    liver cirrhosis is admitted with upper GI bleeding. He has marked ascites and shows multiple
    telangiectasias, liver palmar erythema, and
    clubbing. A diagnosis of bleeding esophageal
    varices secondary to portal hypertension is
    made. Portal pressure is considered elevated
    when it is above which of the following?
    (A) 0.15 mm Hg
    (B) 1.5 mm Hg
    (C) 12 mm Hg
    (D) 40 mm Hg
    (E) 105 mm Hg
A
  1. (C) Portal hypertension is suspected clinically
    if esophageal varices are detected, hypersplenism occurs, or ascites develop. Normal
    portal venous pressure is 5–10 mm Hg.
    Pressure may be measured indirectly by using
    hepatic venous wedge pressure (occlusive
    hepatic wedge pressure). About two-thirds of
    patients with portal hypertension will develop
    varices of which one-third will bleed.
462
Q
  1. A 23-year-old male college student has a history of liver cirrhosis due to KimmelstielWilson syndrome (abnormality in copper
    metabolism). He should be treated with which
    of the following?
    (A) Penicillamine as soon as the diagnosis is
    established
    (B) Penicillamine after variceal bleeding has
    occurred
    (C) A portocaval shunt
    (D) Sclerosis of the esophageal varices as a
    prophylactic measure
    (E) Splenorenal shunt
A
  1. (A) Penicillamine counteracts the adverse
    effects of copper on the liver in patients with
    Kimmelstiel-Wilson syndrome. This has been
    demonstrated in both humans and animals
    afflicted with this disease. Portocaval shunt
    and esophageal varices are not indicated
    prophylactically.
463
Q
  1. A 24-year-old woman presents with menorrhagia, an easy tendency toward bruising, and a
    history of prolonged bleeding after extraction of
    an impacted molar several years previously. A
    diagnosis of idiopathic thrombocytopenic purpura (ITP) is made after appropriate investigations. Her disease has failed to respond to
    steroid and immunoglobin therapy. She is
    scheduled to undergo splenectomy in 1 week,
    but her platelet count is 22,000. What should be
    the treatment of choice?
    (A) She should be given platelets daily and
    be scheduled for splenectomy when her
    platelet count is more than 75,000.
    (B) She should undergo bone marrow
    transplantation.
    (C) She should be treated with steroids only,
    and the operation should be canceled.
    (D) She should receive transfusion with 3 U
    of packed cells.
    (E) She should not be given platelets routinely before surgery.
A
  1. (E) Platelets should not be given before splenectomy, but arrangements should be made to
    have them available immediately before the
    operation. They should be used only if bleeding occurs and after the spleen has been
    removed. ITP is a hemorrhagic disorder characterized by a low platelet count with bone
    marrow findings that show normal or
    increased megakaryocytes. The female/male
    ratio is 3:1. This diagnosis implies that no other
    systemic disease or past history of drug intake
    could account for these changes. Some cases
    may be caused by an autoimmune response.
464
Q
  1. Following a successful splenectomy, for thrombocytopenia, a 24-year-old patient notes that
    she was no longer prone to excessive bleeding.
    Her platelet count had become elevated.
    However, 2 years later, she developed further
    skin purpura, and her platelet count was
    reduced to 45,000. What should she undergo?
    (A) Radioactive technetium (99mTc) scan to
    see if a splenunculus is present
    (B) Radioactive (I135) to see if a splenunculus
    is present
    (C) Exploratory laparotomy
    (D) Platelet transfusion
    (E) Red blood cell (RBC) fragility test
A
  1. (A) Radioactive technetium (99mTc) scan to see if
    a retained accessory spleen (splenunculus) is
    present, which may account for postoperative
    thrombocytopenia. Radioactive technetium
    (not I135) is used to localize splenic tissue.
    Patients with ITP have petechiae, ecchymosis,
    and/or bleeding. Splenectomy performed initially for ITP is likely to be successful in 80% of
    patients, but is less effective in older patients.
465
Q
  1. A 28-year-old woman is diagnosed with TTP. In
    addition to purpura and thrombocytopenia,
    studies will show which of the following?
    (A) Normal arterioles on biopsy of the
    spleen
    (B) Absence of infarction on biopsy of the
    spleen
    (C) Leukopenia
    (D) Elevated urea and creatinine levels
    (E) Suppression of reticulocytes
A
  1. (D) Features of TTP include fever, thrombocytopenic purpura, hemolytic anemia, neurological manifestations, and renal disease. The exact
    cause of TTP has not been determined.
    Histologically, there is diffuse hyalinization of
    arterioles and capillaries, with occlusion and
    infarction. The disease may follow a rapid and
    fulminant course, with death occurring secondary to cerebral hemorrhage or renal failure.
    Treatment includes steroids, plasmapheresis,
    and splenectomy. Approximately, 1/20 cases
    occur in pregnancy, but unlike ITP, TTP is not
    improved by termination of pregnancy.
466
Q
  1. A 24-year-old African American man has sickle
    cell disease. He is admitted to the hospital
    because of a sickle cell crisis. His hemoglobin is
    10 g/dL, and he complains of pain in the lower
    chest wall and legs. His further course of management should include which of the following?
    (A) Emergency splenectomy
    (B) Elective splenectomy
    (C) Admission to the hospital for hydration
    and given dehydromorphine as required
    (D) Administer steroids
    (E) Exchange transfusions to keep his
    hemoglobin at a normal level
A
  1. (C) Sickle cell disease is relatively common in
    African-Americans and certain ethnic communities in the United States. Most patients
    with sickle cell disease respond favorably to
    rehydration and analgesia for each attack. The
    patient must avoid unnecessary exposure to
    infections, hypoxemia, and dehydration. In
    most patients with sickle cell disease, there is
    autoinfarction of the spleen. Splenectomy is
    rarely indicated, except for patients with sickle
    cell disease with a massively enlarge spleen,
    where trapping of RBCs is demonstrated.
467
Q
  1. A 24-year-old woman from the Caribbean is
    admitted to the hospital for severe lower chest
    and upper abdominal pain. Her hemoglobin
    is 9 g/dL. The findings on ultrasound of the
    abdomen and chest x-ray are normal. Her
    father has sickle cell disease. For her physician
    to establish the diagnosis of sickle cell trait or
    disease, she must undergo which procedure?
    (A) A bone marrow study
    (B) Injection of radioactive RBCs
    (C) Red cell fragility studies
    (D) Studies to determine her response to
    erythropoietin
    (E) Blood smear and electrophoresis
A
  1. (E) Sickle cell disease is diagnosed by peripheral smear showing sickle-shaped red cells and
    HbS on electrophoresis. The pathogenesis of
    the disease is characterized by microinfarction
    in different parts of the body. This can lead to
    serious (and in some instances fatal) outcome.
468
Q
95. Splenectomy is often indicated in the management of which of the following?
(A) Hereditary spherocytosis
(B) Hereditary neurofibromatosis
(C) Aplastic anemia
(D) Pheochromocytoma
(E) Hashimoto’s disease
A
  1. (A) In hereditary spherocytosis, the abnormally
    shaped erythrocytes fail to pass through the
    splenic pulp and are more prone to earlier
    destruction. In hereditary elliptocytosis, the erythrocyte membrane also is abnormal. Children
    with spherocytosis should undergo splenectomy
    around their fourth birthday. Other less common
    hematological indications for splenectomy are
    thalassemia, sickle cell anemia, autoimmune
    anemia, and an enlarged spleen that becomes a
    major site of red cell sequestration.
469
Q
  1. A 2-year-old African-American boy is diagnosed as having hereditary spherocytosis. His
    parents should be informed that this condition
    is which of the following?
    (A) It is not associated with a marked
    increase in gallstones.
    (B) It is transmitted as a recessive trait.
    (C) It is diagnosed by showing RBCs
    undergo lysis at a higher osmotic
    pressure.
    (D) It is characterized by a low reticulocyte
    count.
    (E) It is infrequently treated by splenectomy.
A
  1. (C) Characterized by RBCs that undergo lysis at
    a higher osmotic pressure. Gallstones are frequently encountered as a result of increased production of bilirubin. Hereditary spherocytosis
    is transmitted as an autosomal-dominant trait.
    Because of a fault in the RBC membrane, the
    cells are smaller and round and undergo lysis in
    a minor vessel, which results in a relative
    obstruction to flow.
470
Q
  1. A 67-year-old man is admitted to hospital with
    a diagnosis of polycythemia vera. He has considerable back pain and is diagnosed as having
    myeloid metaplasia. This condition is characterized by which of the following?
    (A) Decrease of the connective tissue in the
    spleen
    (B) Decrease in the blood elements of the
    spleen
    (C) Aplastic anemia
    (D) Deterioration after splenectomy
    (E) A favorable response to alkylating
    agents
A
  1. (E) Alkalating agents must be given cautiously,
    because patients with myeloid metaplasia are
    sensitive to these agents. The connective and
    hemopoietic tissues in the spleen and liver are
    increased. Polycythemia vera, myelogenous
    leukemia, and idiopathic thrombocytosis must
    be excluded. Splenectomy is often of value.
471
Q
  1. A 24-year-old woman with rheumatoid arthritis involving the sacroiliac joint and fingers is
    noted to have splenomegaly and neutropenia
    (Felty’s syndrome). She is advised to have
    splenectomy, but she should be informed that
    (A) Large-joint disease symptoms will
    lessen.
    (B) Small-joint disease symptoms will
    lessen.
    (C) Neutropenia responds to splenectomy.
    (D) The joint symptoms will become worse.
    (E) All symptoms will lessen.
A
  1. (C) Felty’s syndrome is characterized by
    splenomegaly, neutropenia, and rheumatoid
    arthritis. Steroids are used initially, but their
    effect usually is transient. Splenectomy favorably alters the leukocyte count; it does not alter
    the clinical course of rheumatoid arthritis. As
    with all patients undergoing elective splenectomy, this patient must be given pneumovax, as
    well as haemophilus and meningiococcal vaccines before surgery.
472
Q
  1. A 10-year-old boy is hit by a truck while riding
    his bicycle home from school. A CT scan shows
    a tear of the spleen. His hematocrit is 32%, and
    he is in pain, although fully alert and oriented.
    His blood pressure is 110/60 mm Hg, and his
    heart rate is 104 bpm. The next step in management should be which of the following?
    (A) Cross-match blood and transfuse
    appropriately
    (B) Perform splenectomy as soon as
    possible
    (C) Perform laparotomy, and suture the tear
    where possible
    (D) Perform angiographic embolization of
    the spleen
    (E) Avoid surgery, even if bleeding
    continues profusely after transfusion
A
  1. (A) The risk of infection after removal of the
    spleen as well as the good results of conservative treatment should encourage a nonoperative approach in children. In adults, surgery is
    usually recommended, but when possible, the
    spleen should be repaired and not removed. If
    the spleen is to be removed on an elective
    basis,pneumovax and prophylactic vaccine
    against H. influenza are given about 2 weeks
    before surgery. (See Answer 69.)
473
Q

Management of Acute Variceal Bleeding?

A

Child A: Surgical shunt

Child B and C: TIPS

474
Q

Nonselective portosystemic surgical shunts?

A

End-to-side portacaval shunt (Eck fistula)
Side-to-side portacaval shunt
Large diameter interposition shunts (eg mesocaval, Drapanas)
Central splenorenal shunt
Proximal splenorenal shunt

475
Q

Selective portosystemic surgical shunts?

A

Distal splenorenal shunt (Warren)
Left gastric vena caval shunt (Inokuchi)
Partial
Small diameter portacaval H graft shunt (Sarfeh)

476
Q

Indications for resection in HCC?

A

Non-cirrhotic patient
Child A
Single lesion
No metastasis

477
Q

Indications for liver transplantation in HCC?

A
One nodule <5cm
2 or 3 nodules <3cm
(-) vascular invasion
(-) extra hepatic spread
Child A, B, C
478
Q

Hepatocystic triangle? (Budde)

A

Formed by the

  • cystic duct (right)
  • common hepatic duct (left)
  • margin of the right liver lobe (superior)
479
Q

Triangle of Calot?

A

Formed by the

  • cystic duct
  • common hepatic duct
  • cystic artery
480
Q

Moosman area?

A
  • Circular area, 30mm in diameter, that fits into the hepatocystic duct angle
  • About 85% of all variations in the hepatic pedicle are found in Moosman area, and 50% are a potential hazard during cholecystectomy
481
Q

Elements of the Critical View of Safety?

A

1) The triangle of calot must be dissected free of fat (without exposing the common bile duct)
2) the base of the gallbladder must be dissected off the liver bed (or cystic plate)
3) two structures (and only two– cystic duct and artery) enter the GB and these can be seen circumferentially (360-degree view)